全部 1- 101- 201- 301- 401- 501- 601- 701- 801- 901- 1001- 最新50


面白い問題おしえて〜な 十問目
1 名前:132人目の素数さん :2005/05/31(火) 02:35:12
面白い問題、教えてください

2 名前:132人目の素数さん :2005/05/31(火) 02:35:56
過去ログ
http://www3.tokai.or.jp/meta/gokudo-/omoshi-log/

3 名前:132人目の素数さん :2005/05/31(火) 03:41:50
3を三つ使って式を作り、その答えをできるだけ大きくしてください。


3+33=

ガイシュツ?

4 名前:132人目の素数さん :2005/05/31(火) 04:07:41
3^(33)はどう? 3/(3-3)=∞では?

5 名前:132人目の素数さん :2005/05/31(火) 09:54:18
>3
などって・・・つかっていいシンボルをキチンと規程しないと問題にならん。

6 名前:竜太 :2005/05/31(火) 09:54:39
アホが1匹釣れたw

7 名前:132人目の素数さん :2005/05/31(火) 12:58:27
3次元空間にn(>=3)個の点がありこの中から任意の3点を
選ぶと二等辺三角形になる。nの最大値を求めよ。
 
↑これ解いて。

8 名前:132人目の素数さん :2005/05/31(火) 13:52:12
>>3
3+∞

9 名前:132人目の素数さん :2005/05/31(火) 13:54:19
>>8
ネ申

10 名前:竜太 :2005/05/31(火) 18:40:42
前レスの998と
>>6
俺の名前使ってひどい事言うなよ!!

11 名前:132人目の素数さん :2005/05/31(火) 21:01:56
ある正の整数がある。下記の操作をしたとき、どのような正の数でも操作が停止することを証明せよ。
・1なら操作を停止する。
・偶数なら2分の1にする
・奇数なら3倍して+1する。
例)
3→10→5→16→8→4→2→1



12 名前:竜太 :2005/05/31(火) 21:57:54
>>10
アホが1匹釣れたw

13 名前:132人目の素数さん :2005/06/01(水) 00:28:22
>>11
コラッツ予想
http://science3.2ch.net/test/read.cgi/math/1072595845/

↑でどうぞ
ただし今、基地外が降臨中

14 名前:132人目の素数さん :2005/06/01(水) 01:09:42
977 名前:132人目の素数さん[sage] 投稿日:2005/05/25(水) 18:48:18
△ABCはAB=ACたる二等辺三角形である。
Cから辺ABに垂線を下し,その足をMとし,Mから辺BCに垂線を下し,その足をNとする。
MN=3,AN=4のとき,ABの長さはいくらか。


未解決

15 名前:132人目の素数さん :2005/06/01(水) 18:22:18
各点の座標を
A=(u,v), B=(b,0), C=(c,0), M=(0,m), N=(0,0)
とおく.ただし u^2+v^2=AN^2=4^2, m=NM=3.

t=(v-m)/uとすると直線MBとMCは次のようにあらわされる
直線MB: y=tx+m, 直線MC: y=-(x/t)+m
したがって,b=-(m/t),c=mt=-m^2/b と書ける.

△ABCがAB=ACなる二等辺三角形である条件は
u = (b+c)/2 = (b-m^2/b)/2 ∴ -2bu + b^2 = m^2

すると
AB^2 = (u-b)^2 + v^2
= (u^2 + v^2) + (-2ub + b^2)
= AN^2 + AM^2
= 25

よってAB=5.


16 名前:132人目の素数さん :2005/06/01(水) 18:57:08
>>15
すげっっっ。
図形って座標入れちゃうと補助線とかなしに機械的に解けるって先生に言われたんですけど、こういうことなんですね。
でも幾何の味わいが薄れるみたいでちょっとタブーな気もしますー。
国公立の二次で平面幾何とか三角比があんまり出題されないのはこういうことがあるからですか?

17 名前:132人目の素数さん :2005/06/01(水) 23:20:56
>>15
そして暇だな

18 名前:132人目の素数さん :2005/06/01(水) 23:26:01
幾何のいやらしさは、もっと不定点が増えて次々と必殺技(=定理)を繰り出す
証明にある。

19 名前:132人目の素数さん :2005/06/01(水) 23:39:51
∀n,m a[n]+a[m]≧a[n+m]を満たす正の実数列a[1],a[2],a[3],…は
lim[n→∞]a[n]/n=inf{a[n]/n}を満たす事を証明せよ。(infは下限を表す)

20 名前:132人目の素数さん :2005/06/01(水) 23:55:51
>>19
>lim[n→∞]a[n]/n=inf{a[n]/n}を満たす事を証明せよ。(infは下限を表す)
 
なんでこんなまわりくどい設問なん?収束することを示せでいいじゃん。
結構有名なんだなこれ。

21 名前:132人目の素数さん :2005/06/02(木) 01:46:06
>>14
>>15お見事
おくればせながら>>14の別解
BCの中点をHとする
AB^2 = AH^2 + BH^2
4^2 = AH^2 + NH^2
上の式から下の式を引くと
AB^2 - 4^2 = BH^2 - NH^2 = (BH + NH )(BH - NH)
NはBC上にあるのでBH - NH > 0であり、
右辺は BN * CN(=CN * BN)に等しい
一方△BMN ∽ △MCNであるので
BN / 3 = 3/ CN よって BN * CN = 3^2
AB^2 - 4^2 = 3^2
∴AB = 5


22 名前:未解決 :2005/06/02(木) 03:26:42
自然数 n が次の性質を満たすとき、nを「良い数」と呼ぶことにする。

性質:nをいくつかの正整数の和にうまく分割すると、それらの逆数の和を1にできる。
    すなわち n=a1+a2+‥‥+ak 、 (1/a1)+(1/a2)+‥‥+(1/ak)=1 とできる。

例1:9は良い数である。9=3+3+3 であり、1/3+1/3+1/3=1。
例2:10は良い数ではない。実際どのような分割に対しても、逆数の和が1にはならない。
例3:11は良い数である。11=2+3+6、1/2+1/3+1/6=1。

問題:良くない数は高々有限個であることを示し、その最大値を求めよ。

23 名前:132人目の素数さん :2005/06/02(木) 07:36:42
>>22
突っ込んでやれ

「良くない数」が定義されてないので回答不能

24 名前:132人目の素数さん :2005/06/02(木) 09:30:23
>>23
突っ込んでやれ

アンケートじゃないので「回答」などいらぬ

25 名前:22 :2005/06/02(木) 10:21:14
訂正。10は良い数でした。10=4+4+2。

>>23
良くない数:=自然数であって良い数ではない数

26 名前:132人目の素数さん :2005/06/02(木) 13:13:41
>>22 すげー汚いし、有限性だけだけど

n が十分大きいとき、n を分割して逆数の和を 1 にするアルゴリズムを示す

(1) n≡0 (mod 2) なら a_1=8、そうでなければ a_1=9+72 として、
n_1=n-a_1 とする。n_1≡0 (mod 2) は明らか。
(2) n_1≡0 (mod 4) なら a_2=8、そうでなければ a_2=10+40 として、
n_2=n_1-a_2 とする。n_2≡0 (mod 4) は明らか。
(3) n_2≡0 (mod 8) なら a_3=8、そうでなければ a_3=12+24 として、
n_3=n_2-a_3 とする。n_3≡0 (mod 8) は明らか。
(4) n_3=8(29+n_4) と分割する。
任意の自然数は4個の(0を含む)平方数の和で表せるので、
n_4 = b_1^2 + b_2^2 + b_3^2 + b_4^2 とする。
(5) b_1〜b_4 に 0 が k 個 (0≦k≦3) 含まれているとする。
n_5=29-k, n_6=n_4+k とする。n_3=8(n_5+n_6)、
n_6 = c_1^2 + c_2^2 + c_3^2 + c_4^2 (c_i > 0) となる。

以上で n = a_1 + a_2 + a_3 + 8n_5 + 8Σc_i^2 と分割できた。

(6) a_1,a_2,a_3 は分割して逆数の和を 1/8 にできる。
(7) 26≦n_5≦29 で、26〜29 は良い数。
(26=4+4+6+6+6, 27=3+6+6+6+6, 28=4+4+4+8+8, 29=2+3+12+12)
n_5 = Σk_i, Σ1/k_i = 1 とできるので、
8n_5 = Σ8k_i, Σ1/(8k_i) = 1/8 のように
8n_5 を分割して逆数の和を 1/8 にできる。
(8) 8c_i^2 を c_i 個の 8c_i に分割すれば、逆数の和を 1/8 にできる。

以上で a_1, a_2, a_3, 8n_5, 8c_i (1≦i≦4) をそれぞれ分割して、
逆数の和を 1/8 にできたので、
n を分割して逆数の和を 1 にできる。

27 名前:132人目の素数さん :2005/06/03(金) 03:54:35
>>22
>>26の議論で、400以上の良くない数は存在しないことがわかる
もう少し修正すれば、上限を 250 程度まで下げられる

適当にプログラム書いて調べてみたら、
(完全探索じゃないし、あまりチェックしてないけど)
↓以外は良い数みたい

2,3,5,6,7,8,12,13,14,15,19,21,23,24,30,31,32,33,37,39,43,57

あとは 57 が本当に良くない数かどうか調べなきゃならない

28 名前:132人目の素数さん :2005/06/03(金) 04:10:38
33=3+3+9+9+9
57=3+9+9+9+9+9+9
だった

2,3,5,6,7,8,12,13,14,15,19,21,23,24,30,31,32,37,39,43

29 名前:132人目の素数さん :2005/06/03(金) 04:39:48
あわわ…
24=2+4+6+12
30=2+3+10+15
31=2+4+5+20
32=2+3+9+18
37=2+3+8+24
39=2+6+6+10+15
43=3+6+6+8+8+12

残り
2,3,5,6,7,8,12,13,14,15,19,21,23

30 名前:132人目の素数さん :2005/06/03(金) 05:48:06
24以下の数は4個以下の数の組合せで表される。

4個以下の組み合わせをリストアップ。
(1)(2,2)(2,3,6)(2,4,4)(3,3,3)
(2,3,7,42)(2,3,8,24)(2,3,9,18)(2,3,10,15)(2,3,12,12)
(2,4,5,20)(2,4,6,12)(2,4,8,8)(2,5,5,10)(2,6,6,6)
(3,3,4,12)(3,3,6,6)(3,4,4,6)(4,4,4,4)

和が24以下になるのは和が1,4,9,10,11,16,17,18,20,22,24の時。
>>29の残りと合わせて全部。

31 名前:132人目の素数さん :2005/06/03(金) 09:06:11
>>27
>>26の議論で、400以上の良くない数は存在しないことがわかる
>もう少し修正すれば、上限を 250 程度まで下げられる
 
ここわからんあげ。ホントにこれで400以上の良くない数は存在しないことが
しめせてるの?

32 名前:132人目の素数さん :2005/06/03(金) 09:20:50
>>22
平方数はつねによい数じゃないの?
n^2=n+n+・・・+n、1=1/n+1/n+・・・+1/n

33 名前:32 :2005/06/03(金) 09:29:49
しまった。問題よみまちがった。良くない数が有限であることを示せか。
>>27でしめせてる。orz

34 名前:132人目の素数さん :2005/06/03(金) 09:35:58
そうだ >>32 が全て正しい。”良い数”は無限に存在する。

35 名前:132人目の素数さん :2005/06/03(金) 09:49:32
>>31
>>26のアルゴリズムが動くには、(4)のところで
n_3 ≧ 8(29+1) = 240
であればよい。
n_3 = n - (a_1 + a_2 + a_3),
a_1 + a_2 + a_3 ≦ 81 + 50 + 36 = 167
だから、n ≧ 240 + 167 = 407 であれば十分。

あと、26〜29 が4個の連続する良い数であることを
使っていて、これは 24〜27 に置き換えられるから、
そのときは n ≧ 8(27+1) + 167 = 391 であれば十分。
つまり、391以上の良くない数は存在しない。

(5)の書き方おかしかったけど、
これは、4個の0を含む平方数の和だと具合が悪いから、
4個の0でない平方数の和になるように、
29 のほうから必要なぶんだけ(k個の)1をもらって来て、
b_i=0 のところを 1 で埋めてる。

4平方数の定理はこのへん
ttp://aozoragakuen.sakura.ne.jp/suuron/node40.html

36 名前:132人目の素数さん :2005/06/03(金) 20:51:52
щ(゚Д゚щ)カモォォォン オモシロイノ カモォォォン (屮゚Д゚)屮 オモシロイノ カモーン アゲ

37 名前:132人目の素数さん :2005/06/03(金) 22:15:31
>>22の「良い数」の定義を
「いくつかの正整数の和」から「いくつかの互いに異なる正整数の和」にするとどうなるか

38 名前:132人目の素数さん :2005/06/03(金) 23:19:19
各階が2.5mの高さの33階建てのマンションの高さを求めよ。


39 名前:39 :2005/06/04(土) 05:00:20
3 = √9


40 名前:132人目の素数さん :2005/06/04(土) 22:30:10
>>22をちょい一般化して
 
正の有理数rに対し自然数nが次の条件をみたすときrに関してよいと呼ぶとする。
 
 ある正の整数 a1、・・・、ak が存在して n=蚤i、r=1/ai
 
正の有理数rを任意に固定するときrに関して良くない自然数は有限個しかないことを示せ。
 
>>37
わからん。ヒントおながいします。

41 名前:132人目の素数さん :2005/06/05(日) 06:24:05
>>31 >>35
上限を落として 56 以上の良くない数が存在しないことが言える。

n = 6a + b (a>0, b∈{30,33,35,50,52,55}) と分割する。
a を4個以下の正の平方数の和で表せば、
6a を分割して逆数の和を 1/6,2/6,3/6,4/6 のいずれかにできる。
b を分割して逆数の和を 2/6,3/6,4/6,5/6 のいずれにもできる(*)ので、
結局 n を分割して逆数の和を 1 にできる。

(*)
30 = 6+12+12 = 3+9+18 = 3+9+9+9 = 2+8+8+12
33 = 6+9+18 = 6+9+9+9 = 3+6+12+12 = 2+6+10+15
35 = 5+15+15 = 3+8+24 = 4+4+9+18 = 2+6+9+18
50 = 10+10+15+15 = 5+5+20+20 = 4+6+8+16+16 = 2+12+12+12+12
52 = 4+24+24 = 3+7+42 = 3+10+12+12+15 = 2+10+10+15+15
55 = 6+7+42 = 5+5+15+30 = 3+4+24+24 = 2+4+21+28

42 名前:132人目の素数さん :2005/06/12(日) 16:47:42
定規のみを用いて、成す角が1/2005度以下である2本の直線を引くことはできるか。

…微妙な問題なので、あまり深入りしないで下さい。

43 名前:132人目の素数さん :2005/06/12(日) 17:27:49
できると思う

44 名前:132人目の素数さん :2005/06/13(月) 22:06:03
半径が10、18、35の三つの円があり、それぞれは他の二つと外側で接している。
この状態の三つの円を内側に含み、全てと接する円の半径はいくらか?

45 名前:132人目の素数さん :2005/06/15(水) 02:21:28
age

46 名前:132人目の素数さん :2005/06/15(水) 20:50:15
>>37
やっとできた・・・一週間かかった。
 
補題1
 
長さ12の3つの正の整数列a1i,a2i,a3iがあって以下をみたす。
・蚤1i=蚤2i=蚤3i=203904。
・1/(a1i)=(1/2)1/(a2i)=(1/3)1/(a3i)=256/3289
・aijの2'部分はすべて相異なる。特に
 aij2^u=akl2^v⇒u=v、i=k、j=l。
・aijは9の倍数であるか5の倍数。
・aijは15の倍数でない。
・aijは7の倍数でない。
(ここで正の整数nの2'部分とは2と互いに素なnの約数のなかで最大のものである。)
 
証明)
a1=(45 1035 99 55 117 65 6435 1287 715 148005 29601 16445)
a2=(9 207 495 55 585 65 6435 1287 715 148005 29601 16445)
a3=(5 115 495 99 585 117 6435 1287 715 148005 29601 16445)
とおけばよい。

47 名前:132人目の素数さん :2005/06/15(水) 20:50:36
補題2
 
任意の整数nについて長さ18の整数列aiが存在し以下をみたす。
・巴i2^(i-1)≡n (mod 203904)。
・bi=49 or 6。
 
証明)
まず203904=2^7・3^2・59なので(203904,43)=1。よって整数mを
43m+6(2^18-1)≡n (mod 203904)、0≦m<203904となるようにとれる。
mの2進展開をm=把i2^(i-1) (i:1〜18)とするときbi=43ci+6と定める。
明らかにbi=49 or 6。さらに
巴i2^(i-1)=(43ci+6)2^(i-1)=43m+6(2^18-1)≡n (mod 203904)。
 
補題3
 
任意の正の整数nについて長さがN=[log(2)n+1](=2進展開したときの桁数)の
数列ciが存在して以下を満たす。
・把i2^(i-1)=n。
・ci=1,3。
・ciの第N項は1。
 
証明)nの2進展開の桁数に関する帰納法。nの2進展開が1桁のときはn=1なのであきらか。
N桁未満で成立すると仮定してnがN桁の整数とする。nが2べきならあきらか。
nが2べきでないとする。n-2^Nは正の整数だから帰納法の仮定を適用して
長さMがN-1以下の数列diを
・播i2^(i-1)=n-2^N
・di=1,3
・diの第M項は1
と選ぶ。M=N-1ならaiをci=di(i<N)、cN=1とさだめればよい。M<N-1のときは
ci=di(i<M)、cM=3、ci=1(i>M)と定めればよい。

48 名前:132人目の素数さん :2005/06/15(水) 20:50:58
補題4
 
任意の正の整数nに対して正の整数の列diが存在し以下を満たす。
・diはすべて相異なる。
・播i=203904n。
・1/di=512/3289。
・diは9の倍数であるか5の倍数。
・diは15の倍数でない。
・diは7の倍数でない。
 
証明)a1i、a2i、a3iを補題1において構成した数列とする。
nにたいして補題3を適用してciを補題3でのべられた条件をみたすようにとる。
その長さをNとおく。長さ12Nの2重数列eij(1≦i≦N,1≦j≦12)を以下のようにさだめる。
eij=a1j2^(i-1) (if ci=1、i≠N)、eij=a3j2^(i-1) (if ci=3)、eNj=a2j2^(N-1)
この2重列eijを適当にならべてdiとする。これが要求された条件を満たすことは容易。
 
補題5
 
任意の整数nに対し正の整数列eiが存在し以下を満たす。
・eiはすべて相異なる。
・覇i≡n (mod 203904)、覇i≦49(2^18-1)。
・1/(ei)=1/6。
・eiの2'部分は3か7か21。
 
証明)biを補題2において構成した列とする。その長さをNとして
2重列fij(1≦i≦N,j=1,2)を以下でさだめる。
fi1=42・2^(i-1)、fi2=7・2^(i-1) (if bi=49)、fi1=6・2^(i-1)、fi2=0 (if bi=6)
fijの0でない項をならべたものをeiとすれば要求された条件をみたす。

49 名前:132人目の素数さん :2005/06/15(水) 20:51:33
補題6
長さ10の正の整数の列fiが存在して以下をみたす。
・fiはすべて相異なる。
・杷i=3696。
・1/fi=1-512/3289-1/6。
・fiは9の倍数でも5の倍数でもないか、15の倍数であるか、7の倍数である。
・fiの2'部分は3でも7でも21でもない。
 
証明)
fiを以下のように定めればよい。
fi=(92, 77, 1380, 420, 910, 345, 105, 91, 35, 70, 13, 156, 2)

定理7
 
49(2^18-1)+3696より大きい任意の整数nに対してxiが存在し以下を満たす。
・xiはすべて相異なる。
・肺i=n。
・1/xi=1。
 
証明)補題4、補題5、補題6より容易。

50 名前:132人目の素数さん :2005/06/15(水) 21:09:29
問題
 
任意の正の有理数aに対して相異なる正の整数x1・・・xnで1/xi=aを満たすものが
存在することを示せ。

51 名前:132人目の素数さん :2005/06/15(水) 21:26:10
>>46-49
うおお、すげえ
証明理解するのに1週間かかりそう

52 名前:132人目の素数さん :2005/06/16(木) 02:18:09
>>50の類題 良問なので貼っておこう

224 名前:132人目の素数さん[] 投稿日:2005/06/15(水) 01:16:23
自然数を並べ替えた数列{x(i)}で、
任意の自然数mに対して、あるnが存在して、
Σ[i=1,n]{1/x(i)}=m
をみたすものが存在することを示せ。

53 名前:42 :2005/06/16(木) 06:46:18
一応、答えを書いておきます。
異なるn本の直線を適当に引く。引き出し原理から、この中のある2本は成す角が180/n度以下となる。
よって、n=180*2005とすればよい。

…が、しかし、どの2本が該当する2本なのかは特定できません。特定できないのに作図したと言えるのかは
よく分からないので、問題文は「引けるか」という表現にしました。が、特定できないのに「引ける」と
言えるのかも結局 微妙な気が…

54 名前:132人目の素数さん :2005/06/16(木) 11:17:35
>>53
てきとうに引いたんじゃ平行になってるかもしんないじゃんかよ。
平行でない保証はどうやってするんだ?
それとも平行な確率は0だとでも言うのか?
まさか平行なら角度は0だから1/2005度以下だとか言うんじゃないだろうな。

55 名前:541 :2005/06/16(木) 11:23:34
いやまあどの2直線も平行でない作図の方法は簡単に見つかるだろうけどな。

56 名前:132人目の素数さん :2005/06/16(木) 11:23:49
>>54
(1)まず適当に1本引き、その上に点Pを取る。
(2)直線上にない1点を取り、そことPを通る直線を引く。
(3)どの直線上にもない1点を取り、そことPを通る直線を引く。
以下(3)の繰り返し。

57 名前:42 :2005/06/16(木) 15:41:11
>>54
ん?平行でない保証は要りませんよ。平行なら成す角は0度ではありませんか?そしてこれは1/2005度以下です。


58 名前:132人目の素数さん :2005/06/17(金) 08:19:18
>>52
>>50の結果をみとめれば簡単。
x(i),i(k)を帰納的に以下のようにさだめる。
(I)x(1)=1、i(1)=1。
(II)x(i)、i(k) (i≦i(k)、k≦K)が
 (i)納i≦i(k)]x(i)=K
 (ii)i(k) (1≦k≦i(K))は狭義単調増大、x(i) (1≦i≦i(K))はすべて相異なる自然数で
   そのなかに1〜Kがすべて入っている。
となるようにさだめられたときi(K)とx(i)(i(K)+1≦i≦i(K+1)を以下のようにさだめる。
まずK+1がx(i) (1≦i≦i(K))のなかにはいっているときj=i(K)、r=0、そうでないときは
j=i(K)+1、r=1/x(i(K))とする。整数mをx(i) (1≦i≦j)のいづれよりもおおきい整数とする。
>>50をみとめているので(1-r)/m=納1≦l≦L]y(l)となる正の整数の有限列がとれる。
そこでi(K+1)=j+L、x(j+l)=my(l)とさだめる。するとi(k) (1≦k≦K+1)、x(i) (1≦i≦i(K+1))
も(I)(II)をみたす。

59 名前:47 :2005/06/17(金) 08:22:18
>>47の補題3まずかった。証明したかったのは
各桁が1か3である2進展開みたいなもんで最高位が1であるものがとれる
といいたかった。まあそのようによんでちょ。

60 名前:132人目の素数さん :2005/06/18(土) 18:06:25
age

61 名前:132人目の素数さん :2005/06/20(月) 17:36:47
>>59
補題3が後でどう使われてるかまだ良く見てないから、
見当違いかもしれないけど、
把i2^(i-1) は 2k+1 か 2k+3 になるから、
偶数はできなくない?


62 名前:132人目の素数さん :2005/06/20(月) 19:45:16
>>50
高校時代に頑張って解いたな。当時の証明を引っ張り出してみたが、えらい分かりにくいので整理した。

まず、次の主張を証明する。
f:N→Nはf(n+1)−f(n)≧n+2 (n∈N)を満たす。X1={a} (2≦a∈N),X(n+1)=f(Xn)∪(1+Xn)とおくとき、
#Xn=2^(n−1)となる(元が全て異なるという意味)。…(i)

補題1:minX(n+1)=min(1+Xn)<minf(Xn)
x=minXn≧2とおくと、min(1+Xn)=1+x,minf(Xn)=f(x) このときf(x)≧f(x−1)+x+1≧f(1)+x+1≧x+2>x+1
=min(1+Xn)よりmin(1+Xn)<minf(Xn)また、X(n+1)=f(Xn)∪(1+Xn)であるから、minX(n+1)=min(1+Xn)

補題2:dn=minXnとおくとdn=a+n−1
d1=aは明らか。また、Lem1よりd(n+1)=min{dn+1,f(dn)}=dn+1 (n≧1)よってdn=a+n−1

(i)の証明:n=1のときは明らか。n=k (k≧1)のとき成り立つとすると、n=k+1のとき、#X(k+1)=#(1+Xk)∪f(Xk)
≦#(1+Xk)+#f(Xk)=2^(k−1)+2^(k−1)=2^k よって、等号が成り立つこと、すなわち(1+Xk)∩f(Xk)=φが成り
立つことを示せばよい。x∈(1+Xk)∩f(Xk)とすると、x=1+y=f(z) (y,z∈Xk)が成り立つ。まず、x=1+yより
x−1=y∈Xk=X(n−1)である。そこで、N∋m<nを、x−m∈X(n−m)を満たす最大の自然数とする。m=n−1のときは
x−m=aであるからx=m+a=a+n−1=dn=minXn<minf(X(n−1))=minf(Xk)(補題1より)となって、x∈f(Xk)に矛盾。
m≦n−2のときは、x−m∈f(X(n−m−1))である。なぜなら、もし そうでないとするとx−m∈(1+X(n−m−1))つまり
x−(m+1)∈X(n−m−1)となり、mの最大性に矛盾するから。さて、x−m∈f(X(n−m−1))よりx−m=f(p) とおける。
よってm=x−f(p)=f(z)−f(p)ここで、m>0であるから、f(z)>f(p) fは狭義単調増加よりz>pよってz≧p+1よって
m=f(z)−f(p)≧f(p+1)−f(p)≧p+2よってp+2≦m≦n−2=k−1よってf(dk)≧f(dk−1)+dk+1=f(a+k−2)+a+k
≧f(a+p+1)+a+k>f(p)+m=x 従って、やはりx<minf(Xk)となり、x∈f(Xk)に矛盾。以上より、(1+Xk)∩f(Xk)=φ
であることが分かり、数学的帰納法から(i)は成り立つ。

63 名前:132人目の素数さん :2005/06/20(月) 19:46:28
次に、f:N∋n→n(n+1)∈N とする。X1={a} (2≦a∈N),X(n+1)=(1+Xn)∪f(Xn)とおく。次の2つが成り立つ。

I Xnの元の逆数の総和は1/aになる。
1/x+1/f(x)=1/xであるから、Σ[x∈X(n+1)]1/x=Σ[x∈Xn](1/(x+1)+1/f(x))=Σ[x∈Xn]1/x=…=Σ[x∈X1]1/x=1/a

II #Xn=2^(n−1) (n≧1)である。(Xnの元は全て異なるという意味)
f(n+1)−f(n)=2(n+2)≧n+2より、(i)から明らか。

さて、IIより、任意のn∈Nに対して1/a=Σ[x∈Xn]1/x は異なる1/xの和になっている。正の有理数b/aに対して、まず
b/a=1/a+…+1/aと分解する。初めの1/aは1/a=Σ[x∈X1]1/xと分解し、次の1/aは1/a=Σ[x∈Xn]1/x (maxX1<minXnと
なるnを持って来る)と分解し、これを次々行うことでb/aは異なる1/xの有限個の和になることが言える。従って、任意の
有理数b/aは異なる有限個の1/xの和で表せる。

64 名前:132人目の素数さん :2005/06/22(水) 01:29:08
>>37
4731203 以上の整数は >>37 の意味で良い数と言える
(>>46-49見た後知恵 + PCで探索)

集合 S,T,U,C を
S={3,5,7,9,11,15,42,45,110}, T={3,5,7,9,11,26,33,42,45,143},
U={1,2,3,6}, C={3,114,247} とする。
Σ[x∈S]x = 247, Σ[x∈T]x = 324, Σ[x∈S]1/x = Σ[x∈T]1/x = 1,
Σ[x∈U]x = 12, Σ[x∈U]1/x = 2, Σ[x∈C]x = 364, Σ[x∈C]1/x = 9/26。

n を 4731203(=52*2^13+4305219) 以上の勝手な整数とする。
n = 52*2^p + m + 4305219 (p≧13, 0≦m<52*2^p) と書ける。
gcd(4*246,13*323)=1 なので、中国剰余定理より、適当な整数 a,b により
m + 4*246*13*323 = 4*246a + 13*323b (a≧0, 0≦b<4*246) とできる。
以上から n = 4(13*2^p-1+246a) + 13(13*2^10-1+323b) + 364 と書ける。
p≧13, m<52*2^p から a<2^p を言うのは簡単。

0≦k≦p-1 に対して
A[k] = { 4*2^k*x | x∈S } (a の2進展開に 2^k が現れるとき),
A[k] = {4*2^k} (それ以外)、A[p] = { 4*2^p*x | x∈U } とする。
A[k] (0≦k≦p) が全て互いに素であることはすぐ分かる。
A = ∪[k=0,p] A[k] とすれば、
Σ[x∈A]x = 4(13*2^p-1+246a), Σ[x∈A]1/x = 1/2。

同様に、0≦k≦9 に対して
B[k] = { 13*2^k*x | x∈T } (b の2進展開に 2^k が現れるとき),
B[k] = {13*2^k} (それ以外)、B[10] = { 13*2^10*x | x∈U } として、
B = ∪[k=0,10] B[k] とすれば、
Σ[x∈B]x = 13(13*2^10-1+323b), Σ[x∈B]1/x = 2/13。

A,B,C が互いに素であることはすぐ分かる。
X = A∪B∪C とすれば、Σ[x∈X]x = n, Σ[x∈X]1/x = 1。

65 名前:132人目の素数さん :2005/06/22(水) 02:10:45
age

66 名前:132人目の素数さん :2005/06/22(水) 14:09:39
>>44の問題に関連してこんな定理みつけた。なんかいかにも
初等幾何でありそうな定理なんだけど。もしかして有名だったりとか?
 
定理 半径Ra,Rb,Rcの円Ca,Cb,Ccが互いに外接をしておりその中心をA,B,Cとする。
    円Ca,Cb,Ccすべてに接する円の小さいほう(2つある)の半径をR、
    三角形ABCの内接円の半径をIとするとき次が成立
 
      1/R=1/Ra+1/Rb+1/Rc+2/I
 
ちなみにCa,Cb,Ccすべてと接する円C'がもう一つあってそいつの半径R'は
1/R'=|1/Ra+1/Rb+1/Rc-2/I|になるようだ。(C'はCa,Cb,Ccすべてと内接するときと
外接するときがあってそれぞれの場合で絶対値の中の符号がきまる。)
いまんとこ複素座標とって円円対応の原理とかつかって力技で証明した。
できた形みるとなんかいかにもスパっとしめせそうな悪寒なんだけど・・・

67 名前:66 :2005/06/22(水) 16:07:45
いま検索してみたらどうもデカルトの円定理とかいうやつらしい。
>>44の問題解くには>>66の“ちなみに〜”の部分を使うんだけどそれはデカルトの円定理
とはいわないようだ。まあオレの証明だと±をちょっと替えて場合分け一回やるだけなんだけど。
ぐぐっても証明みつからんということはそんなズバット瞬殺するような方法はないんだろうな。

68 名前:50 :2005/06/22(水) 21:31:49
せっかくなのでオレが用意してた解答かいときます。
有理数rが調和的というのをr=1/aiとなる相異なる正の整数があるときと
さだめておく。
(StepI) r=a/bが0<r<1でbが奇数とあらわせられるならrは調和的。
(証明)2^n≡1 (mod b)なるnをもってくる。2^n-1=bkとなる正の整数をとる。
このときr=ak/bk=ak/(bk+1)+ak/((bk)(bk+1))=ak/(2^n)+ak/((bk)(2^n))
akの2進展開をak=盃i2^(i-1)とするとak<bk<2^nその桁数はn以下。
つまりi≧nならui=0。よって
ak/(2^n)+ak/((bk)(2^n))=盃i/(2^(n-i-1))+盃i/((bk)(2^(n-i-1)))
の左辺の分母はすべて正の整数。相異なることは用意。
(StepII)r=a/b、b奇数とあらわせられる正の有理数rは調和的。
(証明)納i≦N]1/(2i-1)≦r<納i≦N+1]1/(2i-1)となるNをとる。
s=r-納i≦N]1/(2i-1)とおくときs≦1/3。よって2sは1未満であり
容易に2s=c/d、d:奇数とあらわせられることがわかるのでStepIにより
2s=1/aiなる相異なる正の整数aiをとれる。よってこのとき
r=納i≦N]1/(2i-1)+1/(2ai)となるのでrは調和的。
(StepII)すべての正の有理数は調和的。
(証明)r=a/(b・2^n)、b:奇数となる正の整数a,bと非負整数nをとる。
StepIIよりa/b=1/aiとなる相ことなる正の整数aiをとってくる。
するとr=1/(ai2^n)。

69 名前:132人目の素数さん :2005/06/23(木) 21:43:39
>>37解決したみたいなんだけどせっかくだし
だれか>>37の意味での“最大のよくない数”計算してみてくれん?

70 名前:132人目の素数さん :2005/06/23(木) 22:51:19
>>69
>>37 の意味での良くない数リスト
(探索範囲 n≦2000。自信ないから、誰か追試頼む)

2,3,5,6,7,8,9,12,13,14,15,16,17,18,19,20,21,23,25,26,27,28,
33,34,35,36,39,40,41,42,44,47,48,49,51,56,58,63,68,70,77

71 名前:132人目の素数さん :2005/06/23(木) 23:04:56
訂正 やっぱいきなりバグってた

2,3,4,5,6,7,8,9,10,12,13,14,15,16,17,18,19,20,21,22,23,25,26,27,28,29,
33,34,35,36,39,40,41,42,44,46,47,48,49,51,56,58,63,68,70,72,77

72 名前:132人目の素数さん :2005/06/23(木) 23:46:20
>>70-71
乙。いまんとこの最良評価4731203未満まではまだまだ差があるけどいけそう?

73 名前:132人目の素数さん :2005/06/24(金) 21:40:25
>>72
ちょっと卑怯だけど、下の補題(PCで確認した)を認めれば、
n≧155 なら n は >>37 の意味で良い数。

補題) 155≦n≦822 のとき n を 4 の倍数でない相異なる
自然数に分割してその逆数の和を 1 にできる

定理) n≧155 のとき n は >>37 の意味で良い数

まず n≧718 とする。
n = a[0] + 4b, a[0]∈{8,27,29,34} とできる。

155(4^(p-1)-1)/3 + 4*4^(p-1) ≦ b < 155(4^p-1)/3 + 4*4^p となる p(≧2) を選び、
c = b - {155(4^(p-1)-1)/3 + 4*4^(p-1)} とすれば、0≦c<167*4^(p-1) なので
c = Σ[k=1,p-1] c[k]*4^(k-1) (0≦c[k]<4*167) とできる。
a[k]=c[k]+155 (1≦k≦p-1), a[p]=4 とすれば結局
n = Σ[k=0,p] a[k]*4^k (155≦a[k]≦822=4*167-1+155 for 1≦k≦p-1)
と書ける。

a[0] を分割して逆数の和を 2/3 にできる
(8=2+6, 27=2+10+15, 29=2+9+18, 34=3+6+10+15)。
補題より、1≦k≦p-1 に対して、a[k]*4^k を分割して逆数の和を 1/4^k にできる。
a[p]*4^p を 4^p+3*4^p と分割すれば、逆数の和を 1/(3*4^(p-1)) にできる。
a[k]*4^k (0≦k≦p) は全て 4^k の倍数であって、
4^(k+1) の倍数でない自然数に分割されたので、
分割されてできた自然数には重複がない。
以上から、n(≧718) を相異なる自然数に分割して逆数の和を
2/3 + (Σ[k=1,p-1]1/4^k) + 1/(3*4^(p-1) = 1 にできる。

補題より、明らかに 155≦n≦717 の n は良い数なので、
上と併せて n≧155 のとき n は良い数。

74 名前:132人目の素数さん :2005/06/24(金) 23:41:16
>>73
>a[p]*4^p を 4^p+3*4^p と分割すれば、逆数の和を 1/(3*4^(p-1)) にできる。
 
これおかしくね?4^p=4^(p-1)+3・4^(p-1)と分割すると逆数和は
1/4^(p-1)+1/(3・4^(p-1))=(4/3)(1/4^(p-1))=1/(3・4^(p-2)
じゃね?

75 名前:132人目の素数さん :2005/06/25(土) 00:14:18
>>74
>a[p]=4 とすれば

例)
n=743
a[0]=27, p=2, b=179, c=8, c[1]=8, a[1]=163, a[p]=a[2]=4

743 = 27 + 4*163 + 4^2*4
= (2+10+15) + 4(2+6+9+10+15+22+99) + 4^2(1+3)
= 2+10+15 + 8+24+36+40+60+88+396 + 16+48

76 名前:132人目の素数さん :2005/06/25(土) 00:29:34
>>75
ああ、なるほどわかった。
>n = Σ[k=0,p] a[k]*4^k (155≦a[k]≦822=4*167-1+155 for 1≦k≦p-1)
この後ろの部分の条件でa[p]はまえの構成で4ときまってるんだな。
でその4を1+3と分割すると。ならあとは全部あってる気配が。
ということは結局+>>71で完全に終了?すばらしい。

77 名前:132人目の素数さん :2005/06/25(土) 01:12:57
>>76
>>70 とかもっと上のほうでミスってるのも自分だから、
プログラムに頼ってる部分はあまり…

しかしこの問題は>>37のオリジナル?

(あと、見ればわかるとおり>>73>>46-49の、
無限に大きくなる部分を等比数列に押し込むって
アイデアを借用してる)


78 名前:132人目の素数さん :2005/06/26(日) 10:54:14
1, 2, 3, .... , 19 の数字から任意に異なる 7 個の数を取ると、
そのうちから異なる 2 個からなる組を適当に 2 個選んで、
各組の数の和が等しくなるようにすることができる。

79 名前:132人目の素数さん :2005/06/27(月) 01:51:06
     .┌━┐    ┌━┐
      ┃┌╋──╋┐┃
      └╋┘    └╋┘
        ┃ ・   ・  ┃        ┌━━┐
    ●━╋┐    ┌╂━━━━╂┐  ┃
    └━┷┴━━╂┘        └╋━┘
同じスレにはコピペ ┌╋┐        ┌╋┐
できるけど、違う  ┃└╋╋━━╋╋┘┃
スレにはコピペでき ┃  ┃┃    ┃┃  ┃
ない不思議コピペ ┃  ┃┃    ┃┃  ┃
           └━┘┘   └└━┘


80 名前:132人目の素数さん :2005/06/27(月) 07:32:44
     .┌━┐    ┌━┐
      ┃┌╋──╋┐┃
      └╋┘    └╋┘
        ┃ ・   ・  ┃        ┌━━┐
    ●━╋┐    ┌╂━━━━╂┐  ┃
    └━┷┴━━╂┘        └╋━┘
同じスレにはコピペ ┌╋┐        ┌╋┐
できるけど、違う  ┃└╋╋━━╋╋┘┃
スレにはコピペでき ┃  ┃┃    ┃┃  ┃
ない不思議コピペ ┃  ┃┃    ┃┃  ┃
           └━┘┘   └└━┘

81 名前:132人目の素数さん :2005/06/28(火) 20:55:42
>>78
> 異なる 2 個からなる組を適当に 2 個選んで、
ここの意味がとりにくいけど、
S⊆{1,2,…,19} かつ |S|=7 である集合 S が与えられたとき、
S から相異なる4個の要素 a,b,c,d を取り出して、a+b=c+d とできる
って意味?
でも、そうだとしたら、
S = {1,2,3,5,9,14,19}
が反例になってると思う。

82 名前:78 :2005/06/29(水) 21:37:33
おっと、反例があったか。
では R の部分集合 X で、任意の 0 でない実数 a が a = b - c, b, c ∈ X として一意にかけるような X はあるか?

83 名前:132人目の素数さん :2005/07/01(金) 02:04:47
>>82
いきなりレベルアップしてきたけど、それ解けるの?

84 名前:132人目の素数さん :2005/07/01(金) 02:33:30
>>82
超限帰納法を使えば作ることができることがわかったが、
もっと構成的な方法があるのならもう少し考えてみる。

85 名前:132人目の素数さん :2005/07/01(金) 04:57:18
正の実数全体の集合をVとする。次の(i)(ii)(iii)を全て満たす集合A⊂V,B⊂Vは存在するか。
(i)A∩B=φ,A≠φ,B≠φ,A∪B=V
(ii)x,y∈Aならばx+y∈A,xy∈A
(iii)x,y∈Bならばx+y∈B,xy∈B
…俺は答えは知らない。

86 名前:132人目の素数さん :2005/07/03(日) 18:31:20
>>84
とりあえず超限帰納法を使った存在証明というのをおながいしまつ。

87 名前:132人目の素数さん :2005/07/03(日) 18:55:10
底面が楕円で頂点と底面の中心を結ぶ線が底面と垂直な楕円錘の展開図を描け

僕には分かりません

88 名前:132人目の素数さん :2005/07/03(日) 19:41:14
>>87
模型を作って展開してみよ

89 名前:132人目の素数さん :2005/07/03(日) 20:31:40
>>86
Ωを連続濃度とし、正の実数全体を整列させて {r_α} (α<Ω) とする。
実数の集合 V_α (α<Ω) を以下のように定める。

1) V_0 = 空集合
2) αが極限順序数のとき、V_α = ∪V_β (β<α)
3) α=β+1 のとき。
V_βの濃度はΩより小さいことが帰納法により証明できる。

イ) V_β中の2点で距離が r_β となるものがあれば、V_α=V_β とする。

ロ) V_β中の2点で距離が r_β となるものがないときは、
V_βに距離が r_β となるような2点 {x,y} を新たにつけ加える。

距離が r_β となるような2点 x,y で、
x,y のどちらかが、 V_βの元であるか、V_β のある点との距離が
V_β 中のある2点間の距離として実現されているような x,y の組の濃度もΩより小さい。
距離が r_β となるような2点の組全体の濃度はΩだから、上の条件を満たさないような
x,y の組が存在するので、V_α=V_β∪{x,y} と定める。

V=∪V_α (α<Ω) と定めれば、V が求める集合となる。

90 名前:132人目の素数さん :2005/07/03(日) 20:38:48
ちょっと工夫が必要だが、同じようにして次もできる。

ユークリッド平面の点集合 V で次を満たすものがある。
1) V のどの異なる3点も一直線上にはない。
2) どんな三角形に対しても、V の異なる3点を頂点とする三角形で、
与えられた三角形と合同になるものがただひとつ存在する。

91 名前:132人目の素数さん :2005/07/03(日) 20:41:16
>>89
>V_α=V_β∪{x,y}
 
とさだめたときV_αの任意の異なる4元a,b,c,dでa-b=c-dを満たすものがないことの
保証がよくわからないんだけど。

92 名前:132人目の素数さん :2005/07/03(日) 20:56:59
ごめんなさい。{x,y} の条件に次を追加する必要があった。
(*) x,y の中点は、V_β の2点の中点と一致しない。


V_β∪{x,y}から4点 a,b,c,d を取ったとき、a-b≠c-d を示す。

a,b,c∈V_β で d=x のとき。
c,x の距離が V_β の2点 a,b の距離と一致するので、x,y の定め方に反する。

a,b の距離が r_β (=x,y の距離) となることはないので、
a,b∈V_β で c=x,d=y は起き得ない。

a,c∈V_β で b=x,d=y のとき、
a-x=c-d より a-c=x-y となるので上と同様。

a,d∈V_β で b=x,c=y のとき。(この場合が抜けていた)
a,d の中点と b,c の中点が一致してしまう。

93 名前:132人目の素数さん :2005/07/03(日) 21:15:03
>>92
今度は
V_β中の2点で距離が r_β となるものがないとき
距離が r_β となるような2点 {x,y} で
(*)x,y の中点は、V_β の2点の中点と一致しない
を満たす {x,y} が存在することの保証がいると思うんだけど。

94 名前:132人目の素数さん :2005/07/03(日) 21:54:04
V_β の2点の中点全体の濃度もΩより小さいので、それは容易。

95 名前:132人目の素数さん :2005/07/03(日) 22:02:25
>>94
濃度に関する議論なんかでホントにいえんの?ちょっと信じられないんだけど。
詳しく解説おながいしまつ。

96 名前:132人目の素数さん :2005/07/03(日) 22:07:33
x,y の距離は r_β と決っているから、x,y の組と x,y の中点は 1対1 に対応している。

97 名前:132人目の素数さん :2005/07/03(日) 22:11:50
>>96
それはわかってるけど。それで?

98 名前:97 :2005/07/03(日) 22:18:11
あ、いやわかったかも。なるほど。いえてるね。

99 名前:132人目の素数さん :2005/07/04(月) 01:18:58
>>85解けた人いまつか?

100 名前:132人目の素数さん :2005/07/04(月) 07:23:54
>>89
>正の実数全体を整列させて {r_α} (α<Ω) とする。
選択公理でこんなことができるってのが、
この証明がすごく直感に反する理由か…

101 名前:100 :2005/07/04(月) 07:33:58
>>100は要するに、
α<Ω とできるところがポイントなんだなって言いたかったわけで

スレ汚しスマン

102 名前:132人目の素数さん :2005/07/04(月) 11:47:40
連続体の濃度を持つ最小の順序数(始数)が存在するということだな。
連続体仮説とは無関係。

103 名前:132人目の素数さん :2005/07/09(土) 16:32:58
age

104 名前:132人目の素数さん :2005/07/10(日) 23:15:27
最近はレベルが高杉でついてゆけませぬ。
もうちっと低レベルなのを頼む。

俺は初代スレから居座って、もうほとんど
持ちネタを出し尽くしてしまった。

105 名前:132人目の素数さん :2005/07/10(日) 23:36:28
んじゃ、かなり低レベルな問題。

一辺の長さが1の正四面体をある平面に正射影したときの、射影の面積の範囲を求めよ。
同様に、一辺の長さが1の正八面体についても検討せよ。



前半は大学入試問題。頑張れ!

106 名前:132人目の素数さん :2005/07/10(日) 23:40:05
もう一つ。

ある立体はどのような平面で切っても断面が円(または点or空集合)になるという。
この立体は球である事を示せ。

ある立体はどのような平面に射影しても、円(または点or空集合)になるという。
この立体は球であるといえるか?

107 名前:132人目の素数さん :2005/07/10(日) 23:42:09
>>106 間違った。。。


後半修正。

ある凸な立体はどのような平面に射影しても、面積が一定であるという。
この立体は球であると言えるか?

108 名前:132人目の素数さん :2005/07/10(日) 23:44:27
さらに訂正・・・

誤) 射影
正) 正射影

109 名前:132人目の素数さん :2005/07/11(月) 00:44:05
>>105
前半は簡単だな。
Aをひとつの頂点としてAが光源にもっとも近い場合に限定して一般性を
うしなわない。Aに隣接する3面の法線と光線のなす角をα、β、γとする。
(ただし全部0〜π/2の間とする。)このとき影の面積はcosα+cosβ+cosγ。
結局この値のとりうる範囲をもとめればよい。これは動点Pがx^2+y^2+z^2=1、x,y,z≧0を
動くときのx+y+zのとりうる値の範囲。それは1≦x+y+z≦√3。

110 名前:132人目の素数さん :2005/07/11(月) 00:57:24
>>106
こっちも前半は簡単。
断面にあらわれる円の半径の最大値をRとし、断面が半径Rになる平面αをとる。
このときの円の中心をOとする。すると半径の最大性からOをとおる任意の平面で
切ったときの円の中心は常にO。よってとくにこの立体にPが属する⇔OP≦Rが成立
するので球。

111 名前:132人目の素数さん :2005/07/12(火) 00:14:19
>>109
もちつけ!

正四面体をABCDとして、AB、CDの中点をそれぞれM,Nとする。
線分MNに垂直な平面に射影したら面積いくつよ?
どう見ても、1/2にならんか?

112 名前:132人目の素数さん :2005/07/12(火) 00:36:23
>>106
矢野健太郎の問題

113 名前:132人目の素数さん :2005/07/12(火) 02:14:53
>>111
ああ、正四面体か、正6面体でといてた。

114 名前:132人目の素数さん :2005/07/15(金) 22:41:18
age

115 名前:132人目の素数さん :2005/07/16(土) 09:01:54
オリンピックの柔道でn人がトーナメントに出場した。
決勝戦が終わるまでに何試合行われるか答えよ。
ただし、敗者復活はや、再試合はないものとする。
(一応、数学者の本に載ってた問題のうろ覚えだが)

116 名前:132人目の素数さん :2005/07/16(土) 10:38:14
>>115
算数初心者ですか? 帰ってください!
算チャレでもやってなさいってこった!

117 名前:132人目の素数さん :2005/07/16(土) 11:03:47
トンチ問題だろう・・・答えはn試合。

118 名前:132人目の素数さん :2005/07/16(土) 12:16:04
3位決定戦か。

119 名前:132人目の素数さん :2005/07/16(土) 16:21:06
3位決定戦ってもっと複雑だったような。

120 名前:132人目の素数さん :2005/07/16(土) 16:41:40
>>115
(n-1)試合

121 名前:132人目の素数さん :2005/07/16(土) 16:42:31
>>115
理由、優勝者以外が負ければいいので
優勝者以外の(n-1)が負ける試合数は(n-1)よって(n-1)試合

122 名前:132人目の素数さん :2005/07/16(土) 16:59:58
>>106
後半は昔漏れがfjに質問した事がある。
塚ちんが答えてくれた。
高校の範囲で理解できる内容ではなかった。
そのときの話では>>107は未解決問題だったと思う。

123 名前:132人目の素数さん :2005/07/16(土) 17:23:22
>>122
詳細をおながいしまつ。

124 名前:132人目の素数さん :2005/07/16(土) 18:08:40
理由もぱっとでるし、n-1と簡単に答えられる。
が、それだけでは不十分。
わざわざ「オリンピックの・・・」とか言ってるのがポイント。
決勝の前に準決勝で敗れたもの同士が3位決定戦をするので、
n<=3のとき、n-1試合。
n>3のとき、n試合。

算数レベルなんだけれど、数学を知ってるとひっかかる・・・
という意味で面白かった。

125 名前:132人目の素数さん :2005/07/16(土) 18:21:03
>>106
の後半は矢野健太郎が留学中に遭遇した問題。
立体図形 K がコンパクトなら K の二点 x, y で、 d(x, y) = [K の直径] になるように取る。
この二点を結ぶ直線を含む全ての平面への射影を考えれば容易。

126 名前:132人目の素数さん :2005/07/16(土) 20:18:47
>>117
釣れた。ついでに吊って来い!

127 名前:132人目の素数さん :2005/07/16(土) 20:51:03
>>119
その通り。
準決勝進出者 (4人いる) に直接負けた者 (当然何人もいる) で敗者復活戦 A,B を行ない、
その勝者が準決勝の敗者 (2人いる) と三位決定戦を行なう。

そんなことも確かめていない >124 は ry)

128 名前:132人目の素数さん :2005/07/16(土) 21:26:15
でも敗者復活はないって書いてあるわけだが。

129 名前:132人目の素数さん :2005/07/16(土) 21:30:00
敗者復活はないので一位以外は一回ずつ負けるからn−1。


130 名前:132人目の素数さん :2005/07/16(土) 21:30:20
銅メダルは 2 人いるのだから三位決定戦もないんじゃない?

131 名前:132人目の素数さん :2005/07/16(土) 21:32:44
>>128
脳内オリンピックの話をしても意味がないわけだが。

132 名前:132人目の素数さん :2005/07/16(土) 21:33:25
>>107

平面ならルーローの三角形はどの直線に射影しても線分の長さは一定。
ルーローの四面体もどの直線に射影しても線分の長さは一定。
と言う事は反例があるかもな。

133 名前:132人目の素数さん :2005/07/17(日) 10:23:09
ちなみにルーローの四面体は等幅図形じゃないヨ。

134 名前:132人目の素数さん :2005/07/17(日) 11:09:20
↓ルーローの四面体改良版の当幅図形
http://www.geocities.jp/technopolismars4319/quasi_Rouleau/quasirurotetragon.html


135 名前:132人目の素数さん :2005/07/17(日) 12:36:20
等幅かどうかが問題じゃないだろ。

136 名前:132人目の素数さん :2005/07/17(日) 17:56:42
>>134
あ、それ俺も見たことある。アプレットでぐりぐりまわしてみると、たしかに等幅っぽいんだけど、このページを作った人も証明はしていないみたい。
2次元の等幅図形を対称軸で回転させた回転体が、3次元で等幅になることは知られてるんだけど、それ以外にも3次元の等幅ってありえるんだろうか。
直感的には無理っぽいんだけど…

137 名前:132人目の素数さん :2005/07/18(月) 00:08:58
(1)円に内接する六角形の向かい合う辺を延長すると、3つの交点ができる。
その交点は、常に一直線上にあることを証明せよ。
(2)円→楕円でも成り立つ。証明せよ。

円の中心をOとして、円状の点をA, B, ,,,Fとして、交点をI,G,Hとして、
IGベクトルとIHベクトルが平行として証明しようとしたけど、無理でした。
(2)は、まったく無理。
おながいします。

138 名前:132人目の素数さん :2005/07/18(月) 02:42:20
>>137
(2)は、(1)の結果を用いてよいなら、
楕円を適当に伸縮して円にする一次変換を考えれば終了。

139 名前:132人目の素数さん :2005/07/18(月) 07:20:12
パスカルの定理orブリアンションの定理でぐぐれ
一寸眠いので、、、

140 名前:132人目の素数さん :2005/07/18(月) 18:41:06
age

141 名前:137 :2005/07/18(月) 19:58:39
なんだ。もう定理になってるのか。面白いと思ったのに。。。

142 名前:132人目の素数さん :2005/07/18(月) 21:03:52
>>141
宿題はよそで聞け、馬鹿!

143 名前:132人目の素数さん :2005/07/19(火) 09:06:31
京大の過去問はおもしろいの多いよな。

144 名前:132人目の素数さん :2005/07/19(火) 09:15:23
Eisensteinがそのまま出た時は適当すぎだと思ったが

145 名前:132人目の素数さん :2005/07/19(火) 22:30:38
1辺の長さが6である正方形ABCDにおいて
対角線AC,BDの交点をOとし,Oを中心とする半径2の円Oを描く。
辺BC上に BM = [   ] となる点Mをとったところ
線分AMは円Oの円周から全体の3分の1の長さの弧を切り取った。

146 名前:132人目の素数さん :2005/07/20(水) 02:44:09
>>145
直線AMと点Oとの距離が1であることを考えると(中略)
(27-3√17)/4 かな。

147 名前:132人目の素数さん :2005/07/20(水) 03:33:43
チェスの8クイーン問題のように、
将棋盤(9x9)に龍馬を互いの利きから外して置くことを考える。
1つの盤に最大いくつの龍馬が置けるか。また、その置き方は何通りあるか。



148 名前:145 :2005/07/20(水) 13:10:31
おいらが3日掛かって考えた問題をあっさり解かれちゃったお。
さすが数学板、クオリティたかす。

149 名前:132人目の素数さん :2005/07/21(木) 07:44:28
>>147
竜王?竜馬=馬??
八方桂?



150 名前:132人目の素数さん :2005/07/21(木) 13:33:59
>>147
竜馬=成り角とすると
(右上から左下方向の)斜めのラインは17本だから
高々17個しかおけない。
このとき左上隅付近を考えると左上隅に置くと
次のライン(左から2番目の一番上、一番左の上から2番目)
には置けない。
右下隅も同様なので、高々15個しか置けない。
で、15個は実現可能。
そのパターン数は...難しそう。

151 名前:ずれてない? :2005/07/21(木) 17:30:41

馬□馬□馬□馬□馬  馬□馬□馬□馬□馬  馬□馬□馬□馬□□
□□□□□□□□□  □□□□□□□□□  □□□□□□□□馬
□馬□□□□□□□  □□□□□□□□□  □馬□□□□□□□
□□□□□□□□馬  馬□□□□□□□馬  □□□□□□□□馬
□□□□□□□□□  □□□□□□□□□  □馬□□□□□□□
馬□□□□□□□馬  馬□□□馬□□□馬  □□□□□□□□馬
□□□馬□□□□□  □□□□□□□□□  □馬□□□□□□□
馬□□□□□□□馬  馬□□□□□□□馬  □□□□□□□□馬
□□馬□馬□馬□□  □□馬□馬□馬□□  馬□馬□馬□馬□□


馬□馬□馬□馬□□  馬□馬□馬□馬□□  □馬□馬□馬□馬□
□□□□□□□□馬  □□□□□□□□馬  □□□□□□□□□
□馬□□□□□□□  □□□□□□□□□  馬□馬□□□□□馬
□□□□□□□□馬  馬□□□□□□□馬  □□□□□□□□□
□□□□□□□□□  □□□馬□□□□□  □□□□□□□□馬
馬□馬□□□□□馬  馬□□□□□□□馬  □□□□□□□□□
□□□□□□□□□  □□□□□□□□□  馬□馬□□□□□馬
□□□□□□□□馬  □□□□□□□□馬  □□□□□□□□□
馬□馬□馬□馬□□  馬□馬□馬□馬□□  □馬□馬□馬□馬□


□馬□馬□馬□馬□
□□□□□□□□□
馬□□□□□□□馬
□□□□□□□□□
馬□□□馬□□□馬
□□□□□□□□□
馬□□□□□□□馬
□□□□□□□□□
□馬□馬□馬□馬□

152 名前:132人目の素数さん :2005/07/22(金) 20:58:23
正解>151
オイラが10分かけて思いついて、2時間かけて解いた問題をこうもやすやすと解かれるとは・・・。

153 名前:132人目の素数さん :2005/07/22(金) 21:42:56
>>152
手計算で求まる?

154 名前:132人目の素数さん :2005/07/22(金) 23:04:22
プログラムで解きました。
数学的にはどう解くのかなぁ・・・ってことでこちらに書きました。
>150の高々15個という持っていき方は参考になったのですが、
どうして「15個置ける」となるのかは、頭が弱いのでわかりませんでした(−−;;

155 名前:132人目の素数さん :2005/07/23(土) 00:22:45
8クイーンも全数探索するわけだから、
頭の中だけで言えることは >>150 くらいと思う

156 名前:132人目の素数さん :2005/07/31(日) 11:51:30
長方形の折り紙を何回か折って正五角形にする事を考える。
ただしこの時に角と角を合わせるように折る必要は無いとする。
直線に沿って折るのならそれで良いとする。

折る回数が最小になるようにするには長方形をどのような形にすればよいか。

157 名前:132人目の素数さん :2005/08/02(火) 03:00:32
>ただしこの時に角と角を合わせるように折る必要は無いとする。
>直線に沿って折るのならそれで良いとする。

これの意味がよくわからん
角はかならず辺の位置に重なるように折らないといけないということなのかな?

158 名前:132人目の素数さん :2005/08/02(火) 22:06:15
>角と角を合わせるように折る必要は無い
対角線に沿って折らなきゃいけない、って訳では無いという事です

159 名前:156 :2005/08/03(水) 00:33:18
うーん説明しにくい。
とにかく、普通に折るんならそれで良しって事で。

160 名前:132人目の素数さん :2005/08/03(水) 00:38:47
箸袋を結んで、
2ヵ所折れば正五角形になることは分かった。


161 名前:132人目の素数さん :2005/08/03(水) 03:12:31
???
4辺を作るのに4回折ればよい。(のこる一辺は元の長方形の一辺の一部)
ってのじゃおもしろい問題ってわけじゃなさそうだしなぁ‥

折り線を決定するのに許される手順を定義してくれ

162 名前:132人目の素数さん :2005/08/03(水) 03:24:59
正方形ABCDの ADの中点をM、BCの中点をNとする。
ABをDCに重なるように折る(折り目はMNになる)
次にMNがAB(DCにも)重なるように折る。

これは折る回数は2回なのか?それとも折り目は3本になるから3回なのか?

163 名前:132人目の素数さん :2005/08/03(水) 09:19:47
161で4回、162で2回って感じに考えて下さい。
ってあんまり上手く言えない以上この問題は取り下げた方がいいですね。

自分が用意した答えを言います。
AB=1、BC=tan54゜の長方形ABCDを用意して、
1:BCがACに重なるように折り曲げる。
2:ADがACに重なるように折り曲げる。
3:ACの中点をMとし、Mを通りACに垂直な直線をlとした時に
 直線lに沿って折り曲げる。
これで3回折るだけで正五角形が出来ます。

後は2回折るだけでは直角がどうしても残る事を言えばok。

これからは分かり難い問題は絶対書かないように気をつけますから
今回の事は許してください。

164 名前:132人目の素数さん :2005/08/03(水) 20:03:50
横 7.2cm×縦 1.9cm の紙を用意する。
四隅をABCDとすると、
(1) DがBCに重なるように
(2) AもBCに重なるように
(3) 頂点Bと頂点Cが重なるように折る。
一辺 2cmの正五角形ができる。
折り目は2本。
縦横の比率は、sin 72゚ : 3+2(cos72゜)

165 名前:163 :2005/08/04(木) 09:18:33
もうごめんなさいね。出直してきます…

166 名前:132人目の素数さん :2005/08/09(火) 10:26:00
【難しい問題】
1+1=2をネタにした面白い問題を作れ

167 名前:132人目の素数さん :2005/08/09(火) 10:37:30
>>166
他スレへ行ってくれ

168 名前:132人目の素数さん :2005/08/14(日) 04:59:55
r(10+2r(5))=3.804226065181
19/5
175/46


169 名前:132人目の素数さん :2005/08/14(日) 11:34:42
将棋のルール上存在する局面で、着手可能な手の数が最大になる局面はどういったもので、着手可能な手は何通りか。

170 名前:132人目の素数さん :2005/08/17(水) 20:32:59
盤面王2枚以外残り全部手駒(王2枚は桂馬の打ち場所確保のため2二と2八ぐらいに置いておく)
と思ったんだけど5一龍置いとくだけでも銀金角飛打ちの4手引いても14手儲けか…

171 名前:132人目の素数さん :2005/08/17(水) 21:45:58
5二飛なら飛角金銀桂香歩打ちを差し引いても29手儲けだな

172 名前:132人目の素数さん :2005/08/17(水) 23:45:34
593手という局面を作った。
歩、桂、金はどこに配置しても、損をする。
香車は最下段に置くと最大の得をする。銀は2段目に置くと最大の得をする。
飛は1段目に置くと最大の得をする。自玉は2段目に置くと最大の得をする。
角はよく分かってないです。敵玉は1段目に置くのが最小の損で済むのですが・・・うーむ。
角と敵玉のところが上手く説明できないと、ダメですね。

飛□□□□□□□□
□□玉□銀□銀銀王
□□□□角□□□□
□□□□□□□□□
□□□□□□□□□
□□□□□□□□□
□□□□□□□□□
□□□□□□□□□
□香□香□香□□□
先手の持ち駒:残り駒全部
王は敵玉ね。

173 名前:132人目の素数さん :2005/08/17(水) 23:54:08
>>170
何で儲かるの?
他の開いてるマスへの飛車の打ち込みが消えるから明らかに損のような気がするけど。

174 名前:132人目の素数さん :2005/08/18(木) 00:25:00
>>173
どっちを打っても飛車打は飛車打。


175 名前:132人目の素数さん :2005/08/18(木) 00:28:52
>>174
そっか。納得。

176 名前:132人目の素数さん :2005/08/18(木) 00:30:00
飛車二枚で他の駒を使わないと
二枚持ち駒のとき八十一通りで
一枚1一で一枚持ち駒のとき百十二通り。


177 名前:132人目の素数さん :2005/08/18(木) 00:32:11
後、成りと成らずとかも考えないとダメなのかな。

178 名前:132人目の素数さん :2005/08/18(木) 12:29:44
>>7の問題が未だに分かりませぬ

179 名前:132人目の素数さん :2005/08/18(木) 21:24:35
>>178
大丈夫だ。俺もわかめ。

180 名前:132人目の素数さん :2005/08/19(金) 02:15:50
>>7はおかしいだろ。
点列(1/n,0,0)、n自然数でいくらでもとれるだろ。

181 名前:132人目の素数さん :2005/08/19(金) 02:16:57
しまった

182 名前:132人目の素数さん :2005/08/19(金) 13:29:03
立方体の各面の中心に点を置いて6個かな?

183 名前:132人目の素数さん :2005/08/20(土) 23:11:02
>3次元空間にn(>=3)個の点がありこの中から任意の3点を
>>選ぶと「いずれも」二等辺三角形になる。nの最大値を求めよ。
いずれもって入れとけばわかる問題だったかも。

184 名前:132人目の素数さん :2005/08/21(日) 03:21:38
>>7
一直線上に並んだ等間隔の3点も二等辺三角形と
みなしていいなら、8点いけるかな‥。

円 x^2+y^2=1, z=0 上に正五角形状に5点を取り、
あとは原点と(0,0,1)と(0,0,-1)の3点。

一直線不可なら「あとは」以降の1つがダメになるから7点。

俺はこれが限界。もちろん証明なんて無理。

185 名前:GiantLeaves ◆6fN.Sojv5w :2005/08/21(日) 15:06:51
s,tはともに正の実数で、s<tとする。
川には地面に対して常に一定の流れがある。川にA地点とB地点を定める。
船は流れに対して常に同じ速さで動くものとし、
ボールは流れに対して速度が0とし、A地点にあったボールはしばらくするとB地点に到達するものとする。
船がA地点からB地点までまっすぐ進むのにsだけの時間がかかり、
船がB地点からA地点までまっすぐ進むのにtだけの時間がかかる。
A地点においたボールがB地点に到達するまでの時間はいくらか?

186 名前:132人目の素数さん :2005/08/21(日) 19:55:06
船の速さをx,流れの速さをy,AB間の距離をlとおく。

(x+y)s=l
(x-y)t=l
が成り立つ。
これをlとyについて解くと
y=x(t-s)/(s+t)
l=2xst/(s+t)
これより求める時間はl/yなので
l/y=2st/(t-s)
あってるかな。

187 名前:132人目の素数さん :2005/08/21(日) 20:04:04
ひょっとして185にまじれすした俺はカコワルイですか?


188 名前:132人目の素数さん :2005/08/21(日) 20:05:07
king気取りの基地外だってわかってるんなら問題なし

189 名前:132人目の素数さん :2005/08/21(日) 20:33:19
ある星の赤道上から東へ3000km進み、
北へ3000km進んで到達した場所は
出発地点から4000kmの距離があった。
この星が球状の場合、半径はいくらか?
高校卒業してたら解けるはづ

190 名前:132人目の素数さん :2005/08/21(日) 21:42:35
2n枚のトランプがある。
これを前半と後半のn枚づつの二つの山に分け一枚ずつ互い違いに
なるようにシャッフルする。このシャッフルを
何回繰り返すと元の状態に戻るか。nの式で表せ。


191 名前:GiantLeaves ◆6fN.Sojv5w :2005/08/21(日) 21:47:02
talk:>>186-187 問題をよく読んでみよう。地面の勾配が激しく変化するときなどを考えて、答えは不定。

192 名前:GiantLeaves ◆6fN.Sojv5w :2005/08/21(日) 21:47:41
「地面に対して一定」なのだから。

193 名前:132人目の素数さん :2005/08/21(日) 21:51:19
>>190
A1B1A2B2 → B1A2B2A1 → A2B2A1B1
何回でも繰り返せる



194 名前:GiantLeaves ◆6fN.Sojv5w :2005/08/21(日) 21:52:03
talk:>>189 距離はどのように測るのか?

195 名前:132人目の素数さん :2005/08/21(日) 21:53:14
片側が黒い羊の話思い出した。

196 名前:132人目の素数さん :2005/08/21(日) 22:03:13
オレも

197 名前:132人目の素数さん :2005/08/21(日) 22:15:14
>>195
もっと詳しく!

198 名前:GiantLeaves ◆6fN.Sojv5w :2005/08/21(日) 22:23:56
talk:>>197
一匹の黒い羊を見て三人が一言。
この国には黒い羊がいます。
この国には少なくとも一匹の黒い羊がいます。
この国には少なくとも一匹の、少なくとも片側が黒い羊がいます。

199 名前:132人目の素数さん :2005/08/21(日) 22:28:39
>>189解けねえ・・・

200 名前:132人目の素数さん :2005/08/21(日) 23:27:16
>>189
12000/π?

201 名前:132人目の素数さん :2005/08/22(月) 00:05:22
>>191
地面に対して一定だから勾配の影響はない

202 名前:132人目の素数さん :2005/08/22(月) 00:28:44
>>189
>>200であってるのか?

203 名前:132人目の素数さん :2005/08/22(月) 01:04:17
>>202
>>189じゃないけどあってんじゃね?
ちなみに距離の測り方は大円コースだろ、普通は。
要は大きさがひとしいベクトルOA,OB,OCがあって、
・OA,OBの張る平面とOB,OCの張る平面が垂直
・OA,OBのなす角とOB,OCのなす角が等しい
・OA,OCのなす角はOA,OBのなす角の4/3倍
ってことでしょ。

204 名前:199 :2005/08/22(月) 02:26:27
式は立つんだが全然解けない。本当に12000/πか?
どこで間違えたのやら・・・

205 名前:GiantLeaves ◆6fN.Sojv5w :2005/08/22(月) 08:44:46
talk:>>201 場所によらずに一定ならそれでいいが。

206 名前:132人目の素数さん :2005/08/22(月) 13:06:02
球面三角法で>>189は解く

207 名前:132人目の素数さん :2005/08/22(月) 13:59:11
次の中で実際にあるポストは
1月・2火山・3海中

208 名前:132人目の素数さん :2005/08/22(月) 14:16:22
>>207
3.海中

今は昔、TVで見たよ。

209 名前:132人目の素数さん :2005/08/22(月) 14:41:48
あったり〜♪
今は昔ってなに?

210 名前:132人目の素数さん :2005/08/22(月) 15:52:00
c(3)(c(3),s(3),0)+s(3)(0,0,1)=(c(3)^2,c(3)s(3),s(3)).
c(3)^2=c(4).
(c(2)c(1)-s(2)s(1))^2=2c(2)^2-1.
c(2)^2(1+c(2))/2-c(2)(1-c(2)^2)+(1-c(2)^2)(1-c(2))/2=2c(2)^2-1.
2c(2)^3-2c(2)^2-3c(2)/2+3/2=0.
(c(2)-1)(2c(2)^2-3/2)=0.
c(2)=1,c(4)=1/2.


211 名前:132人目の素数さん :2005/08/22(月) 18:21:49
>>206
球面三角法とわ??age
>>207-209
な――ぜ――じゃ――??age
>>210
な――に――も――の――じゃ――??age

212 名前:132人目の素数さん :2005/08/22(月) 18:53:55
http://image.blog.livedoor.jp/thinkdeep/imgs/3/4/341da381.jpg

こいつの謎を解いてくれ
俺にはどうしても分からん・・

213 名前:132人目の素数さん :2005/08/22(月) 19:13:22
0と1の数字がランダムに並べられた十分大きな乱数表を考える。
乱数表では「・・・010101・・・」と0,1が交互に並ぶよりも
「・・・001111000・・・」のように連続して団子になっているところが多い。
このような0や1の団子の大きさが平均してどれほどかを考える。

(1)乱数表から無造作にひとつ数字を選び、その数字を含む団子の大きさを量る。
(例)…011110・・・の左から3番目の1を指定した場合、団子の大きさは4
   大きさ1の団子も考える。このように団子を選ぶとき、その大きさの期待値を求めよ。
(2)乱数表から無造作にひとつ数字を選び、その数字を含む団子の"右隣の"団子の大きさを量る。
   その期待値を求めよ。

214 名前:132人目の素数さん :2005/08/22(月) 19:14:09
>>212
直角3角形に見えて、実は斜辺が折れ曲がっている。

215 名前:132人目の素数さん :2005/08/22(月) 19:31:24
>>213
えらんだ数字とおなじ数字が右側につづく数をあたえる確率変数をR、
えらんだ数字とおなじ数字が左側につづく数をあたえる確率変数をLとおく。
P(R≧n)=(1/2)^n、P(L≧n)=(1/2)^nであるから
E(R)=P(R≧1)+P(R≧2)+P(R≧3)+・・・=1/2+1/4+1/8+・・・=1
E(L)=P(L≧1)+P(R≧2)+P(R≧3)+・・・=1/2+1/4+1/8+・・・=1
E(団子の大きさ)=E(L+1+R)=3
E(その数字を含む右隣の団子の大きさ)=E(1+R)=2
 
あってる?

216 名前:132人目の素数さん :2005/08/22(月) 19:58:28
>>212
http://www.uploda.org/file/uporg175420.png

太い線で描いて、見た目を誤魔化しているところがミソ。

217 名前:215 :2005/08/22(月) 20:00:11
あれ?
>その数字を含む団子の"右隣の"団子の大きさを
って団子のその数字から右側の部分じゃないか・・・でなおしてきまつ。

218 名前:215 :2005/08/22(月) 20:03:32
“右隣”の団子の大きさをあたえる確率変数をXとおく。
P(X≧n)=(1/2)^(n-1)であるから
E(X)=P(X≧1)+P(X≧2)+P(X≧3)+・・・=1+1/2+1/4+・・・=2
E("右隣の"団子の大きさ)=E(X)=2
かな?

219 名前:213 :2005/08/22(月) 20:18:56
自分の考えた答えも
(1)3
(2)2
ですのであってます。
選び方で期待値が異なるので面白いなとおもって出題しました。

220 名前:132人目の素数さん :2005/08/22(月) 23:53:35
>>212
傾き3/8と2/5の斜辺をつなげて直線になるわけないじゃん!!!って問題。

221 名前:189 :2005/08/23(火) 00:48:56
>>200
あたりっす。やっぱ簡単でしたね。

222 名前:132人目の素数さん :2005/08/23(火) 01:11:56
>>221
いや結構苦労したんだが
途中
cos^2(3000/r)=cos(4000/r)となって結局cos(1000/r)=xとおいて6次方程式解いたんだが
簡単なやりかたがあるのかな?

223 名前:132人目の素数さん :2005/08/23(火) 01:17:52
>>221
自分が問題考えたときは、三角関数の式を色々いじってたら
3倍角の公式がでてきてちょっと面白いかもと思って書いてみた。
「球面三角法」なるものの存在はさっきこのスレ見て知ったorz

224 名前:132人目の素数さん :2005/08/23(火) 01:18:36
× >>221
>>222

225 名前:132人目の素数さん :2005/08/23(火) 01:20:50
>>223
あれ自作問題だったのか、それはすごいわ。
球面三角法は俺も知らんかった


226 名前:132人目の素数さん :2005/08/23(火) 01:26:56
すみません。大学生ですが質問してもいいですか?

227 名前:132人目の素数さん :2005/08/23(火) 01:40:38
>>226
質問は質問スレに行け!
言われないと分からんのか、馬鹿かおまえは!

228 名前:132人目の素数さん :2005/08/23(火) 01:48:32
>>227
…スレ違い失礼しました。

229 名前:1 :2005/08/23(火) 10:53:21
てすと

230 名前:132人目の素数さん :2005/08/23(火) 11:42:10
6組のバカップルを3つのカラオケボックスに男女2人ずつ放り込む。
ムカつくので、どのバカップルも同室にならないようにしたい。
何通りの分け方があるか?ただし、部屋の区別はしない。

231 名前:132人目の素数さん :2005/08/23(火) 12:11:01
>>230
265通り。
一般にn組なら n!(Σ[k=0 to n](-1)^k/k!)通り。

232 名前:132人目の素数さん :2005/08/23(火) 12:31:50
>>231
完全順列の確率の公式ですか?
公式は正しいですが、この問題の答えはそれでいいのですか?

233 名前:132人目の素数さん :2005/08/23(火) 12:40:08
>>231
チンコ洗って出直して来い!

234 名前:132人目の素数さん :2005/08/23(火) 12:43:04
>>233
あぁ勘違い。逝ってくる。

235 名前:132人目の素数さん :2005/08/23(火) 12:49:30
900通りか。

236 名前:132人目の素数さん :2005/08/23(火) 12:54:09
>>235
違うと思います。
なんか、こっちまで自信がなくなってきました…

237 名前:132人目の素数さん :2005/08/23(火) 13:01:18
>>234
答えを書こうか?

   r;;;;;ノヾ        
   ヒ‐=r=;'     ∬   3分間待ってやるッ!
   'ヽニ/  っ━~~   
 _と~,,  ~,,,ノ_  ∀   
    ミ,,,,/~). │ ┷┳━ 
  ̄ ̄ ̄.じ'J ̄ ̄| ┃
  ̄ ̄ ̄ ̄ ̄ ̄ ̄ ┻

238 名前:132人目の素数さん :2005/08/23(火) 13:01:30
>>235
あぁ、ごめんごめんorz
150通り?

239 名前:132人目の素数さん :2005/08/23(火) 13:09:01
>>238
正解です。

おまけ。
6組のバカップルを3つのカラオケボックスに男女2人ずつランダムに放り込むとき、
どのバカップルも同室にならない確率はいくらか? ただし、部屋の区別はしない。

240 名前:GiantLeaves ◆6fN.Sojv5w :2005/08/23(火) 18:41:54
EをR^3の中の体積確定集合かつ有界閉集合(ジョルダン可測かつ有界閉)で内点を持つとする。
fをE上リーマン積分可能で、常に正または0となる(この条件は別にいらないが)関数とし、内点をもつ適当な閉集合F⊂Eを選ぶとF上でのfの積分が0にならないとする。
密度分布がfとなっているEの重心をgと置く。
AをR^3上の線形変換で、退化せず、特異でないものとする(つまり一対一上への写像)。
A(E)={x∈R^3|∃y∈E,x=Ay}の重心はA(g)であることを証明せよ。

241 名前:GiantLeaves ◆6fN.Sojv5w :2005/08/23(火) 18:44:31
追加:[>>240]ではA(E)の密度分布がfAであるときの重心を求める問題。

242 名前:132人目の素数さん :2005/08/24(水) 02:40:46
>>239
1/3?

243 名前:132人目の素数さん :2005/08/24(水) 02:41:35
あ、間違えた。 3組の‥だと思ってた

244 名前:132人目の素数さん :2005/08/24(水) 03:09:52
>>243
もちろん 1/3 じゃないぞ。

   r;;;;;ノヾ        
   ヒ‐=r=;'     ∬   3分間待ってやるッ!
   'ヽニ/  っ━~~   
 _と~,,  ~,,,ノ_  ∀   
    ミ,,,,/~). │ ┷┳━ 
  ̄ ̄ ̄.じ'J ̄ ̄| ┃
  ̄ ̄ ̄ ̄ ̄ ̄ ̄ ┻

245 名前:132人目の素数さん :2005/08/24(水) 03:15:00
1/9。


246 名前:132人目の素数さん :2005/08/24(水) 03:26:27
             (⌒).(⌒)
      (⌒) /.:/___! :| (⌒)    >>245
       }: レ;{.:〈= =j.::ト/.:/_) (⌒)    
‐ 、    ,ノ = ≡ィ=ミ ミ :/.:/:、! /.:/    よーしよし、いい子だ
`ヽヽ⌒〈: ミ ミ ミ: >べミ.ミ ミ ミ:Y.:/ /¨ヽ
   \`ヽi〜/,イフノ ミ ミ ミ ミ i/,//¨  厂 ̄ ̄ ̄`ヽ
-‐=ニ、 ヾj} 〈_ノ' ` ヾ-'⌒>'Y´/    〈 3個か !? |
-―- 、\jリ,/    / `! y<,.} j'´   =@_,厶_  _____,ノ
┤イ ト丶∨    ノ / 、},。)j/  ┌'´ ̄ ̄´  ` ̄`'┐
┬ 卜 イ ! }  // _ "¨ ン′  | 甘いの3個    〉
┬ ┤ト 」ハ{/´  fエl ,. '"|    <  ほしいのか? |
L.卜 ┬ /\.〉   iノ┴'‐--'     └、______,r‐'
\_. イ   `ー'´      
イへ./    , ---、_____ヽ丶   、__,/ ̄ ̄ ̄ ̄`'ー┐
}-‐'´    _ノ_,入_`ブ┐    \  3個…       ヽ
 ̄`7ー'7´/  ノ  jレーく__) ))   | イヤしんぼめ !! |
  {l __{ ..:\__┌<_>‐'´        \_______,/
_____ヽ __,>-‐'´ `´___ ____ ┐7
---‐'´     /___/| ̄又又>|
二ヽ ̄ヽ〜'ニ三ヾ⌒ ̄ヽ、又>'´|
  }}  └ =ニ~~}}  `ー--く_|_/
==' _ -‐ ___,ノ、二._ーァー'
___,. -一'¨¨ ̄  `'ー‐'´ ̄

247 名前:132人目の素数さん :2005/08/24(水) 03:45:00
247

248 名前:132人目の素数さん :2005/08/24(水) 04:53:08
1/15

249 名前:132人目の素数さん :2005/08/24(水) 05:11:53
>>248
馬鹿

250 名前:132人目の素数さん :2005/08/24(水) 14:42:01
次の式を満たす整数(a,b)の組をすべて求めよ
   a^2 - 2b^2 = 1

251 名前:132人目の素数さん :2005/08/24(水) 16:47:32
>>250
マルチすな、蛆虫!

252 名前:132人目の素数さん :2005/08/27(土) 02:26:47
age

253 名前:132人目の素数さん :2005/08/31(水) 02:22:32
数列{f(n)}はf(0)=0,f(1)=1,f(n+1)=f(n)+f(n−1) (n=1,2,3…)を満たす。
A={f(i)|i=0,1,2,…}とおく。以下の問いに答えよ。
(1)B={Σ[i=1〜∞]f(xi)|自然数列{xn}は有限項を除いて0}=Σ[i=1〜∞]Aとおく。B=N∪{0}を示せ。
(2)C={Σ[i=1〜∞]f(xi)|自然数列{xn}は有限項を除いて0であり、xi=xjならばxi=xj=0}
=(Aの異なる元を有限個足した数全体の集合)⊂Bとおく。C=N∪{0}を示せ。
(3)自然数mに対して、Bm={Σ[i=1〜∞]f(xi)|自然数列{xn}は、第m+1項目から全て0}
=Σ[i=1〜m]A ⊂B とおく。集合系{Bm}には極限集合が存在し、lim[m→∞]Bm=B と
なることを示せ。また、Bm=N∪{0}を満たす自然数mは存在するか?

254 名前:132人目の素数さん :2005/08/31(水) 04:10:26
age

255 名前:253 :2005/08/31(水) 12:19:00
ちょっと表現に不都合が…
>>253に出てくる「自然数列{xn}」は、全て「非負整数列{xn}」に置き換えて下さい。

256 名前:132人目の素数さん :2005/09/01(木) 19:12:17
>>253
(1) 1をいっぱいつかっていいので簡単
(2) 任意の自然数(正の整数とする)が異なるフィボナッチ数列内の数の和で表示できるという
意味と解してa≦xnなる自然数aは可能であることをnに関する帰納法でしめす。
(I)n=0のときa≦x0=0なる自然数aは存在しないゆえ成立。
(II)n=1のときa≦x1=1ならa=1ゆえ成立。
(III)n≧2に対しn未満まで正しいとしてa≦xnをとる。a≦x(n-1)なら帰納法の
仮定からよい。x(n-1)<a≦xnのときは0<a-x(n-1)≦xn-x(n-1)=x(n-2)ゆえ
a-x(n-1)=輩(i)、y(i)は相異なるフィボナッチ数列内の数がとれる。a-x(n-1)≦x(n-2)
ゆえy(i)の中にx(n-1)は入らない。よってaも相異なるフィボナッチ数列内の数の和で表示可能。
(I)〜(III)で示された。
(3)F={x(i)|i≧1}とおく。n≧3にたいして単調増大有限列y1<y2<・・・<ykが
輩i=-1+xn、yi∈Fをみたすときk≧[(n-1)/2]をみたすことを示す。
(I)n=3,4なら[(n-1)/2]=1ゆえ明らか。(∵yiの項数は1以上。)
(II)n≧5にたいしてn未満についての主張を仮定する。単調増大有限列y1<y2<・・・<ykが
輩i=xn-1、yi∈Fをみたすと仮定する。yk<x(n-1)のとき納1≦i≦n-2]x(i)=-1+xnにより
k=n-2、y(i)=x(i)でなければならない。このときk=n-2>(n-1)/2≧[(n-1)/2]ゆえ成立。
yk=x(n-1)のときは納i<k]y(i)=-1+xn-x(n-1)=-1+x(n-2)。よって帰納法の仮定よりk-1≧[(n-3)/2]。
よってk≧[(n-1)/2]。
(I),(II)で示された。

257 名前:132人目の素数さん :2005/09/01(木) 20:38:11
>>256
(3)の解答がよく分からんのだが、
>Bm=N∪{0}を満たす自然数mは存在するか?
これの答えは どうなってるの?

258 名前:132人目の素数さん :2005/09/01(木) 21:16:02
>>257
あ、よみかえしたら同じフィボナッチ数列内の数を何回つかってもいいのか・・・
すまん。吊って来る。

259 名前:132人目の素数さん :2005/09/09(金) 18:49:22
最近ネタ切れか?

260 名前:132人目の素数さん :2005/09/10(土) 01:09:47
ってか数学板が全体的に…

261 名前:132人目の素数さん :2005/09/10(土) 02:05:41
電車の某広告にあった問題。

A君、B君、C君が○×クイズ10問をやった。
彼らの解答と正解数は以下の通り。

A:○××○○××○×○ 8問正解
B:××○○××××○× 2問正解
C:○×××○○○○×○ 7問正解

この時、Cが必ず不正解になっていると言える問題は何問目か?

262 名前:132人目の素数さん :2005/09/10(土) 02:27:47
0123456789
A:○××○○××○×○ 8問正解
B:××○○××××○× 2問正解
C:○×××○○○○×○ 7問正解

3156047928
A:○×××○○○○×× 8問正解
B:○×××××××○○ 2問正解
C:××○○○○○○×× 7問正解

AとBが同じ答えの物のうち2問が正解ならAとBに関しては矛盾はない。他の場合はAとBに関して矛盾になる。
だから、047928のラベルのついた6問題に関してはCも正解。
だから、3156のラベルの問題のうちAとBは正解2問、Cは1問。
ラベル1に関してCが正解なら356に関してABが正解になってしまう。
だから、ラベル1に関してCは必ず不正解。
つまり、2問目。

263 名前:132人目の素数さん :2005/09/10(土) 02:29:54
ラベルが見にくかったな。
---0123456789
A:○××○○××○×○ 8問正解
B:××○○××××○× 2問正解
C:○×××○○○○×○ 7問正解

---3156047928
A:○×××○○○○×× 8問正解
B:○×××××××○○ 2問正解
C:××○○○○○○×× 7問正解


264 名前:132人目の素数さん :2005/09/10(土) 02:30:30
ラベル的にもうだめぽ。

265 名前:132人目の素数さん :2005/09/10(土) 05:01:43
正解です。こうやると分かりやすいかも。

B:××○○××××○× 2問正解
↓全解答反転
D:○○××○○○○×○ 8問正解
C:○×××○○○○×○ 7問正解

266 名前:132人目の素数さん :2005/09/10(土) 05:30:41
>>261
電車の通っていない田舎者ですが、
いったい何の広告だったのでしょうか?

267 名前:132人目の素数さん :2005/09/10(土) 06:59:51
AC解答同じ問題=7問。AC解答違う問題=3問。AC正解数計15問。
AC解答違う問題の正解数は、問題数と同じで3問。
したがって、
AC解答同じ問題…AC両者正解=(15-3)/2=6問、両者不正解1問。
AC解答違う問題…A正解2問、C正解1問。

AC解答違う問題のBの解答はすべてAと同じ。
よって、Bの正解の問題はAC解答違う問題3問中の2問で、AC解答同じ問題7問のBの答えはすべて間違い。

よって、AC解答同じ問題中、Bと解答の等しい問題が不正解。

268 名前:132人目の素数さん :2005/09/10(土) 11:29:17
>>266
日能研って受験塾は電車広告に中学入試の問題を載せてるの。

269 名前:132人目の素数さん :2005/09/10(土) 11:43:53
京葉線でしょ>>261

270 名前:132人目の素数さん :2005/09/10(土) 12:44:01
>>269
山の手でも中央線でもあるんじゃないかな?

271 名前:132人目の素数さん :2005/09/10(土) 12:54:33
東横線でも地下鉄でもあるがな

272 名前:132人目の素数さん :2005/09/10(土) 15:31:55
答えられるか?

3人で一泊3万円の部屋に泊まることになった。
前払いで3万円を払ったが、後で主人が2万円の部屋に
案内してしまったことに気づいた。そこでバイトに1万円を
持たせて返してくるように言いつけた。ところがこのバイト、
7千円を自分のポケットに入れて、3千円をお釣りとして
返してしまった。
3千円のお釣りが帰ってきたので、払った宿代は27000円
バイトが盗んだお金は7000円
合計すると27000円+7000円=34000円

最初に払ったお金は3万円なのだが、余った4千円は
どこから生まれてきたのでしょう?



273 名前:132人目の素数さん :2005/09/10(土) 15:59:14
>>272
どうやらバナッハタルスキー分割をしてしまったようだな。

274 名前:132人目の素数さん :2005/09/10(土) 16:07:25
>>272
ハウスドルフ空間

275 名前:名無しさん@そうだ選挙に行こう :2005/09/10(土) 18:27:33
>>272
畳み込み積分すれば分かる。

276 名前:261 :2005/09/10(土) 18:56:02
>京葉線でしょ>>261
>山の手でも中央線でもあるんじゃないかな?
>東横線でも地下鉄でもあるがな
自分は小田急江ノ島線で見ますた

277 名前:名無しさん@そうだ選挙に行こう :2005/09/10(土) 19:52:08
誰か>>253の(3)の解答きぼんぬ。

278 名前:名無しさん@そうだ選挙に行こう :2005/09/10(土) 21:00:00
f(n)≦n^m。


279 名前:名無しさん@そうだ選挙に行こう :2005/09/11(日) 00:45:58
>>278
??

280 名前:名無しさん@そうだ選挙に行こう :2005/09/11(日) 10:08:25
ここの人たちにとってはすぐにわかるかもしれないけど・・・

f(x)を x^k (0≦k≦n) の係数がa_kのn次多項式と定める。
xに関する方程式 f(x)=0 の解をα_1,α_2,・・・,α_nとする。(ただし、解に0は含まれない)

このとき、解の逆数の和、Σ[k=0,n](1/α_k)=-a_1/a_0 であることを証明せよ。

281 名前:名無しさん@そうだ選挙に行こう :2005/09/11(日) 10:09:22
>>280
宿題はスレ違い

282 名前:名無しさん@そうだ選挙に行こう :2005/09/11(日) 10:11:07
>>280
宿題じゃないです。
面白い問題だということで書き込んだだけです。

283 名前:名無しさん@そうだ選挙に行こう :2005/09/11(日) 10:12:15
× >>280
○ >>281 

284 名前:名無しさん@そうだ選挙に行こう :2005/09/11(日) 10:18:20
z = 1/x

285 名前:名無しさん@そうだ選挙に行こう :2005/09/11(日) 11:04:29
>>280
クズが!

286 名前:名無しさん@そうだ選挙に行こう :2005/09/11(日) 15:52:59
問:面が全て正三角形の凸多面体の種類は有限か否か。有限だったら何種類か。

google検索して調べちゃやーよ

287 名前:名無しさん@そうだ選挙に行こう :2005/09/11(日) 16:02:00
>>277
>>253
こんな解答ができた。
(3)問題を次によみかえる。
 
@@@@ 問題 @@@@
x(i)をフィボナッチ数列、つまりx(1)=1,x(2)=1,x(n+2)=x(n+1)+x(n) (n≧0)とする。
有限個をのぞいて全部0である非負整数列の集合をCとする。J(c)を把(i)でさだめ
n∈Nに対してE(n)をmin{J(c) | 把(i)x(i)=n}でさだめる。{E(n)}は有界でないことをしめせ。
 
@@@@ 解答 @@@@
c∈Cに対してJ’(c)をJ(c)-(1/2)#{i | c(i)=1}でさだめ、n∈Nに対しE’(n)を
min{J’(c) | 把(i)x(i)=n}で定める。次は綺麗な証明が見つからないのでみとめる。
 
補題 任意のnとn=把(i)x(i)であるc∈Cにたいしてc’∈Cで
n=把(i)x(i)、J’(c’)≦J’(c)、c’(i)≦1 (∀i)なるc’がとれる。
 
これをみとめる。F(n)をmin{#I | 納i∈I]x(i)=n}でさだめる。すると
(1/2)F(n)≦E’(n)≦E(n)が成立する。実際補題よりE’(n)=J’(c)、c(i)≦1である
c(i)がみつかる。このとき把(i)x(i)=納c(i)=1]x(i)より把(i)≧F(n)。さらにJ’(c)の定義と
c(i)≦1より把(i)=2J’(c)。∴(1/2)F(n)≦E’(n)。一方でJ’(c)≦J(c)からE’(n)≦E(n)は明らか。
さて>>256によりF(x(n)-1)≧[(n-1)/2]である。(ちなみに“=”になる。)
よってE(x(n)-1)≧(1/2)[(n-1)/2]であるからE(n)は有界でない。
 
補題の証明なんだけど結構ながい。書く気にならん。どうしたもんだろ。

288 名前:名無しさん@そうだ選挙に行こう :2005/09/11(日) 22:36:48
楕円曲線Eのエル関数L(E,s)を、s=1の周りでテイラー展開すると次のように書けたとする。

L(E,s)=(係数)×(s-1)のr乗+倍(s-1)の(r+1)乗以上の項}

このとき、rはこの楕円曲線上の点で、x,y両成分ともに有理数である点と無限遠点O全体のなす有限生成アーベル群のランクとなることを証明できるか?


289 名前:253 :2005/09/11(日) 23:04:43
>>256
(1),(2)大正解です。
>>287
うおお…合ってそうだが俺には判断できぬ。

一応、考えていた(3)の解答を書いておきます(概要のみ)。
m∈Nを、Bm=N∪{0}が成り立つ自然数とする。このとき、V=Σ[a∈Bm−{0}]1/aは発散する
ことになる(Σ[a∈N]1/aは発散するので)。ところが、
V≦Σ[ki=0〜∞ (i=1,…,m),(k1,k2,…,km)≠0]1/{f(k1)+f(k2)+…+f(km)}
及びf(n)〜α^n (α=(1+√5)/2)及び相加相乗平均の不等式などにより、Vは収束することが
分かり、矛盾。

余談ですが、Vが収束することから、lim[n→∞]#(Bm∩[1,n])/n=0となることも分かります。
N全体に占めるBmの割合はゼロであって、実は かなりスカスカの集合になっている感じ。f(n)の
オーダーが指数関数だから、高々m個足したところで、Nのほんの一部しか表せない、ということかな。

290 名前:名無しさん@そうだ選挙に行こう :2005/09/11(日) 23:18:23
>>287
長い証明上等。読んでみたいから気合いいれて書いてちょ。

291 名前:287 :2005/09/11(日) 23:22:15
どもす。明日かいてみまつ。

292 名前:132人目の素数さん :2005/09/12(月) 19:01:29
2次多項式f(x)と整数b(≠0)の組み合わせで
f(x)(f(x)-b)=0の解が互いに異なる整数になる物を考える。
|b|が最小となる時の|b|はいくつ?

293 名前:132人目の素数さん :2005/09/12(月) 19:03:35
>>292
f(x)(f(x)-b)=0の解が異なる4つの整数ってこと?

294 名前:132人目の素数さん :2005/09/12(月) 19:05:44
>>293 いぇす。同じ解があっちゃだめ

295 名前:132人目の素数さん :2005/09/12(月) 19:40:00
f(x)=x(x−1)/2。
b=1。


296 名前:287 :2005/09/12(月) 23:06:58
清書してみたらもっと綺麗な解答がみつかった。
再度用語の整理。x[i] (i≧1)はフィボナッチ数列。Cは有限個をのぞいて0の非負整数列の集合。
このとき次が成立。
 
補題 c∈Cにたいしてc’∈Cを把[i]x[i]=把’[i]x[i]、把’[i]≦把[i]、c’[i]≦1を
みたすものがとれる。
 
証明 Z×Zの元とみなしてここにに辞書式順序をいれる。
(u,v)=(把[i]^2,min{i|c[i]≧2)をZ×Zの元とみなしてこの順序に関する帰納法。
u=0ならあきらか。(u,v)<(U,V)まで成立するとして
(u,v)=(U,V)と仮定する。∀i c[i]≦1ならc’=cとすればよい。∃i c[i]≧2と仮定する。
CaseI)∃i c[i],c[i+1]>0のき。このときはiをc[i],c[i+1]>0、c[i+2]=0となるiがとれる。
d∈Cをd[i]=c[i]-1、d[i+1]=c[i+1]-1、d[i+2]=c[i+2]+1、d[j]=c[j] (j≠i,i+1,i+2)
で定める。このとき播[i]x[i]=把[i]x[i]、播[i]<把[i]、播[i]^2<把[i]^2
であるから帰納法の仮定より題意をみたすc’が存在する。
CaseII) CaseIでなくc[1]≧2 or c[2]≧2のとき。
このときc[1]=0 or c[2]=0である。
d∈Cをd[1]=c[1]+c[2]-1、c[1]=1、d[j]=c[j] (j≠1,2)
で定める。このとき播[i]x[i]=把[i]x[i]、播[i]=把[i]、播[i]^2<把[i]^2
であるから帰納法の仮定より題意をみたすc’が存在する。
CaseIII) CaseI,IIでないとき。
i=min{i|c[i]≧2}とおく。このときi≧3でありc[i-1]=c[i+1]=0。
d∈Cをd[i-2]=c[i-2]+1、d[i-1]=0、d[i]=c[i]-2、d[i+1]=1
で定める。このとき播[i]x[i]=把[i]x[i]、播[i]=把[i]、播[i]^2≦把[i]^2
であり、最後の不等号の等号成立は(c[i-2]、c[i-1]、c[i]、c[i+1])=(1,0,2,0)
のときのみである。このときは(d[i-2]、d[i-1]、d[i]、d[i+1])=(2,0,0,1)。
よっていづれにせよ(播[i]^2,min{i|d[i]≧2)<(把[i]^2,min{i|c[i]≧2)であるから
帰納法の仮定より題意をみたすc’が存在する。□
(続く)

297 名前:132人目の素数さん :2005/09/12(月) 23:08:19
(続き)
主張 E(n)=min{把[i]|把[i]x[i]=n}、F(n)=min{把[i]|把[i]x[i]=n、c[i]≦1}で
さだめるときE(n)=F(n)。
 
証明 E(n)≦F(n)はあきらか。F(n)=把[i]、把[i]x[i]=nをみたすものをとるとき
c’を把[i]’≦把[i]、把’[i]x[i]=nをみたすものがとれるゆえF(n)≦E(n)。□
 
定理 {E(n)}は非有界。
 
証明 >>256よりF(x[n]-1)≧[(n-1)/2]なのであきらか。□

298 名前:132人目の素数さん :2005/09/16(金) 11:48:23
>>289でええんちゃう?

299 名前:132人目の素数さん :2005/09/16(金) 21:39:14
自然数aが自然数bで割り切れるとき2^a-1は2^b-1で割り切れることを証明せよ


300 名前:GiantLeaves ◆6fN.Sojv5w :2005/09/16(金) 22:34:13
2,3,5のいずれの倍数でもない整数nに対して、
ある正整数mが存在して(10^m-1)/9がnの倍数になることを証明せよ。

301 名前:132人目の素数さん :2005/09/16(金) 22:41:38
>>299
b|aとする。2^a=(2^b)^(a/b)≡1^(a/b)≡1 (mod 2^b-1)。
 
>>300
(10,9n)=1より10^φ(9n)≡1 (mod 9n)。(3,n)=1よりn|(10^φ(9n)-1)/9。φはオイラーの関数。

302 名前:301 :2005/09/16(金) 22:45:09
>>300
・・・(3,n)=1よりn|(10^φ(9n)-1)/9。・・・
“(3,n)=1より”はいらんかったorz。10^φ(9n)-1=9nkから(10^φ(9n)-1)/9=nkでよかった。

303 名前:132人目の素数さん :2005/09/16(金) 23:41:12
なんかで読んだんだが。
平面上にn(>=3)個の点がある。このなかから任意の2点を選んだとき
この2点を通る直線上に第3の点が必ず存在するとき、n個の点は
すべて同一直線上にあることを示せ。


304 名前:132人目の素数さん :2005/09/16(金) 23:52:59
>>303
帰納法ですぐ示せそうだな。

305 名前:132人目の素数さん :2005/09/17(土) 00:01:53
>>303
それ知ってる。予想として提示されてから6年くらいして、やっと証明されたんじゃなかったかな。
証明そのものは結構簡単だけど、そのアイデアが凄かった。

306 名前:132人目の素数さん :2005/09/17(土) 03:27:14
非ユークリッド平面、例えば球面とかだと
正八面体の頂点上に置けば仮定を満たすよね。
って事はユークリッド平面にしか無い性質を使うのか。

307 名前:132人目の素数さん :2005/09/17(土) 06:06:38
>>306
それ仮定を満たすかな?
正八面体の頂点のうち、2点は北極点・南極点の位置に置いてよい。この2点を通る直線で、
この2点しか通らないものが存在する(経度を変えていけばすぐ見つかる)。

308 名前:132人目の素数さん :2005/09/17(土) 06:26:50
>>307
そういや球面は2点を通る直線が一意的に決まらない場合あるのか。
「一意的に決まる」って性質を満たさないんじゃ仕方ねーな。除外だ。

309 名前:132人目の素数さん :2005/09/17(土) 11:52:55
1からnまでの数字が1つずつ書かれたn枚のカードを箱に入れる。
「箱からカードを1枚取り出して、数字を確認して元に戻す」
という操作を、n種類のカードが全て出現するまで繰り返す。
終了するまでに取り出されたカードの枚数をXとして、E(X)をもとめよ。
…というのを高校の頃考えた。

過去ログ読んでません。既出だったらすまそ。

310 名前:132人目の素数さん :2005/09/17(土) 13:43:23
>>309
http://www.geocities.co.jp/CollegeLife-Club/7442/math/#omake

311 名前:132人目の素数さん :2005/09/17(土) 19:30:07
一人で1時間に小包10個配達出来る地域があるとする。
二人・三人だと1時間にそれぞれ何個配達出来る?
尚、二人・三人の時は配達区域が二分の一・三分の一になるものとする。

がいしゅつスマソ

312 名前:311 :2005/09/17(土) 19:44:23
あげてみる(▼∀▼)フォーーーーーー!

313 名前:132人目の素数さん :2005/09/17(土) 20:11:38
単純に2倍3倍になるんじゃね?

314 名前:132人目の素数さん :2005/09/18(日) 15:01:12
配達区域が重複していれば2倍、3倍かも知れぬが、
担当する区域が狭くなることで、配達効率が上がる
かも知れない。

315 名前:132人目の素数さん :2005/09/19(月) 05:41:45
仕事量的に考えると一人で一時間で10個→二人で30分で10個になる?
二人で一時間だと20個なのかな?

316 名前:132人目の素数さん :2005/09/19(月) 07:29:40
配達し終わった後の帰る時間を考慮しなくてはとかいろいろ考えてしまうんだがな。
1点から出るループの数とその長さみたいな。

317 名前:132人目の素数さん :2005/09/19(月) 12:16:48
巡回セールスマン問題?


318 名前:132人目の素数さん :2005/09/19(月) 18:52:22
はぁ?

319 名前:132人目の素数さん :2005/09/20(火) 23:07:35
ひまつぶしにどぞ
―――――――― 問題 ――――――――
Mを正の整数、e1,e2,・・・,ekをMの約数からなる数列する。
Xを{1,2,3,・・・,M}での一様分布にしたがう確率変数とする。
このときXが全てのeiで割りきれない確率は(1-1/e1)(1-1/e2)・・・(1-1/ek)以上で
あることを示せ

320 名前:132人目の素数さん :2005/09/22(木) 20:59:37
次のようなゲームを考える。
(1)ジョーカーを除いた52枚のトランプをよく切り山にして置く。
(2)プレーヤーは赤か黒を宣言する。
(3)山の一番上のカードを一枚めくり色を確認する。
(4)宣言とめくったカードの色が一致すれば1ポイント獲得。めくったカードは山にもどさない。
(5)これをカードが無くなるまで繰り返す。

このゲームを次の戦略で行った。
山に残っているカードが赤が多いときは赤を宣言する。
そうでないときは黒を宣言する。
この戦略を戦略Aと呼ぶ。

問1 戦略Aは必ず26ポイント以上獲得できることを示せ。
問2 必ず27ポイント以上獲得できる戦略は存在しないことを示せ。
問3 戦略Aの平均点を求めよ
問4 戦略Aはもっとも優れた戦略といえるか?

正直問3と問4は俺もわかりません。


321 名前:132人目の素数さん :2005/09/22(木) 21:01:47
>>320
それ過去にでたよ。どうすんだったかな?だれかメモのこしてんじゃね?

322 名前:132人目の素数さん :2005/09/22(木) 21:09:16
とりあえず問い3の答えは
(26-1/2)C[52,26]+2^51
のようだ。証明ものこしてあったけど長い。

323 名前:132人目の素数さん :2005/09/22(木) 21:44:03
>>352
最後の+2^51とかどう考えてもおかしいだろ

324 名前:132人目の素数さん :2005/09/22(木) 21:47:01
>>323
C[2n,n]〜4^nだからこれでちゃんと52以下におさまるようになるみたい。

325 名前:132人目の素数さん :2005/09/22(木) 21:50:44
>>324
(26-1/2)C[52,26]>0なのに2^51とか足したら何点になるんだよ

326 名前:324 :2005/09/22(木) 21:51:38
すまぬ。よみまちがえてた。答えは
((26-1/2)C[52,26]+2^5)/C[52,26]
だって。


327 名前:132人目の素数さん :2005/09/22(木) 22:09:08
>>320
まず問題をn×nの碁盤目の路の各交差、三叉路に対しその右側または上側の路を
指定していくことを「戦略」とよぶこととし、この碁盤目の左下から右上に動点が最短経路
をうごく。指定した路を通過したとき1点がもらえるゲームと解釈する。
碁盤目の左下を原点右上を(n,n)とする直交座標をxy座標とする。
(問1)
戦略Aで確実にn点とれることをnに関する帰納法でしめす。
n=1では容易。n<kで成立するとしてn=kと仮定する。動点の経路が(0,0),(n,n)以外で
y=x上にのらないときは動点はつねにy≧x、またはy≦xにいる。前者なら動点が右に
動くとき、後者なら上に動くときにかならず1点もらえる。ちょうどn回づつ動点は右、上に
うごくのでn点以上もらえる。0<k<nなる整数kで(k,k)にくるときは帰納法の仮定より
(0,0)〜(k,k)でk点、(k,k)〜(n,n)でn-k点とれるので計n点はとれる。□
(問2)
n+1点以上とれる戦略がないことをnに関する帰納法で示す。
n=1では容易。n<kで成立するとしてn=kと仮定する。帰納法の仮定から(0,0)〜(n-1,n-1)に
至る経路で高々n-1点以下しかとれない経路が存在する。(n-1,n-1)での路の指定が上なら右、
右なら上を通る経路を通るとき得点は1点しかとれないのでこの経路では高々n点しかとれない。□

328 名前:132人目の素数さん :2005/09/22(木) 22:10:00
┏┳┳┳┳┓
┣╋╋╋╋┫
┣╋╋╋╋┫
┣╋╋╋╋┫
┣╋╋╋╋┫
┗┻┻┻┻┛


329 名前:320 :2005/09/22(木) 22:11:20
外出だったか。orz
いい問題思いついたと思ったんだが。


330 名前:132人目の素数さん :2005/09/22(木) 22:14:49
いい問題だと思うけど。

331 名前:132人目の素数さん :2005/09/22(木) 23:06:12
>>326
それって少数であらわすと難点?


332 名前:132人目の素数さん :2005/09/22(木) 23:27:00
30.040665になった。

333 名前:132人目の素数さん :2005/09/23(金) 20:00:53
>>320
山が上から順に赤黒赤黒…赤黒となっていて、しかも初めに黒と宣言した場合は、
最終的な得点ってゼロにならない?

334 名前:132人目の素数さん :2005/09/23(金) 21:46:40
>>333
黒の残数が多いから、次に黒と宣言して当たるが?

335 名前:132人目の素数さん :2005/09/24(土) 03:05:31
ちょっとこの人面白すぎ。

別れようってほどじゃないけど恋人に引いた瞬間 13
http://love3.2ch.net/test/read.cgi/ex/1127399171/l50

94 名前: 恋人は名無しさん 投稿日: 2005/09/24(土) 02:38:23
>>91
呆れてモノも言えないんだが・・。
最後にもう一度だけ教えてやる。
確率の比較としてあげただけ、とお前はぬかすが、
確立論として話す事が間違っている。
もう少し説明してやると、交通事故って確率は、根底に(ゼロを目指したいのだが)ある程度は仕方がない、という前提がある。
何故なら車での交通手段というのは必要不可欠だからだ。
翻って、歩きタバコの危険性というのは何だ? ただの喫煙者のエゴだ。この歩きタバコという行為に何の必然性も無い。
つまり比較する、という検証は、同じ土台でなければ意味が無いんだよ。
片方は、ある意味、仕方がない悲劇、に対して、片方はゼロに出来る訳だ。つまりそこは比較対象にならない。
どうだ? 猿にでも分かる様に丁寧に説明してやった訳だが、どうせお前は馬鹿だから訳分からん事言い返すんだろうな。
ほんと、惨めだよお前ほどの馬鹿って。

336 名前:132人目の素数さん :2005/09/24(土) 03:35:59
全然面白く無いぞ
単に必要悪だ、と言ってるだけじゃん

というか貼るならせめて雑談スレに貼るべき内容

337 名前:132人目の素数さん :2005/09/24(土) 03:53:58
ん?必要悪といってるのか?
俺は何が言いたいか理解不能なんだが・・・

338 名前:132人目の素数さん :2005/09/24(土) 03:58:33
いやいや。
確率の話を持ち出して、確立論とやらを語ると言って、確率の話に戻ったという
精神分裂気味なところに突っ込んでほしいのでは?

339 名前:132人目の素数さん :2005/09/24(土) 05:30:43
>>338
自演乙。

340 名前:132人目の素数さん :2005/09/24(土) 20:18:33
確率の定義や計算に面白い部分も無いのに数学板に貼るなヴォケ。

341 名前:132人目の素数さん :2005/09/24(土) 20:32:59
>確率の定義や計算に面白い部分も無いのに

確率の公理的定義とかモンテカルロ法とかの計算が面白くないっていうこと?
確率論を数学板から追い出せなんて言ってるの?

342 名前:132人目の素数さん :2005/09/24(土) 21:03:10
nは3で割るとa、5で割るとb、7で割るとcのあまりが出るとき
nの一般式は?

343 名前:132人目の素数さん :2005/09/24(土) 21:10:03
それのどこが面白いの?
中国剰余定理そのままじゃん

344 名前:132人目の素数さん :2005/09/24(土) 22:08:21
>>341
笑える突っ込み所が無いって事だ。さっさと巣に帰れ。

345 名前:132人目の素数さん :2005/09/24(土) 22:19:00
>>335に言ってるのか、確率論の人全般に言ってるのかハッキリさせてくれ

346 名前:132人目の素数さん :2005/09/24(土) 22:21:28
まあまあ落ち着け。
良スレが荒れる。


347 名前:132人目の素数さん :2005/09/25(日) 21:34:42
平面上に直線が何本かあり、直線同士が交わって出来る交点が2つ以上あるとする。
この時、いずれかの交点は直線が2本だけしか通ってない事を証明せよ。

348 名前:132人目の素数さん :2005/09/26(月) 01:46:35
>>347
有限本?

349 名前:132人目の素数さん :2005/09/26(月) 06:12:50
>>348
有限本

350 名前:132人目の素数さん :2005/09/26(月) 22:57:48
>>347
ヒントおながいしまつ

351 名前:132人目の素数さん :2005/09/26(月) 23:05:13
>>347
鳩の巣原理とか使う?


352 名前:132人目の素数さん :2005/09/26(月) 23:42:56
>>347
泥臭いかもしれないけどゴリゴリやったらできた・・・ハズ。
もしそのような直線の有限集合Sが存在したとする。l∈Sをひとつ固定する。
Pを交点の集合としてPの元でl上でなくかつlとの距離が最小であるものの一つXをとる。
まずaを通る直線でlと平行でないものが3つ以上あるときを考える。
そのうちの3つをm,n,kとする.。m,n,kとlの交点をy,z,wとするときY,Z,Wがl上この順に
ならんでいるとしてよい。すると仮定よりzを通る直線がlとn以外にもう一つ必要だが
それはかならず線分XYか線分XZの端点以外の部分と共有点をもつことになり
その点はXよりもlとの距離が小さくなる。これはXの取り方に反する。
よってXを通るSの元は3個であり内一つはlと平行。それをmとする。
直交座標xyを設定してl:y=0、m:y=1と仮定して一般性をうしなわない。
l上の点でx座標が最大である点をA(a,0)、m上の点でx座標が最大である点をB(b,1)とする。
さらにBをとおる直線でmとの交点のx座標が最大になる点をC(c,0)、
Aをとおる直線でlとの交点のx座標が最大になる点をD(d,1)とおく。
c<aと仮定する。Aを通るl以外の直線は2つ以上あるのでいづれかはBをとおらない。
それをkとおく。するとkはlのx>bの部分かまたは線分BCの両端点以外の部分と交叉する。
これはbの最大性やXがもっともlに近いこと等に反する。よってc=a。同様にしてd=b。
Aを通るlと直線AB以外のSの元uと、Bを通るmと直線AB以外のSの元vをとる。
仮定よりuとmの交点をE(f,1)とすればbの最大性よりf<b。
vとlの交点をE(e,0)とすればaの最大性よりe<a。よってABFEはこの順で台形ABFEの頂点と
なるが対角線AFとBEの交点のy座標は0より大きく1より小さい。矛盾。

353 名前:132人目の素数さん :2005/09/26(月) 23:55:23
ウチの本によると、>>347>>303は同値であるらしい。同値であることの
証明は載ってなかったけど。
で、本に載ってた>>347の証明を読んだが、方針は>>352と同じで「Pの元で
l上でなくかつlとの距離が最小であるものの一つXをとる。」として始めて
いた。その後は 面倒な計算なしに かなりすっきり終わってた。

354 名前:132人目の素数さん :2005/09/26(月) 23:56:39
>>353
その本を紹介してください。

355 名前:132人目の素数さん :2005/09/27(火) 00:29:20
初等的に解いた高等数学の問題(III)
数学新書
ア・エム・ヤグロム他
東京図書

でつ。

356 名前:132人目の素数さん :2005/09/27(火) 00:51:45
>>355
残念ながら、詮索では見つからないですね。
東京図書だから、いやな予感がしてた、やっぱり見つからない…
_| ̄|○

357 名前:132人目の素数さん :2005/09/27(火) 01:01:09
>>355
俺が高校生のとき高校の図書室で見かけてだな…過去に教員が1人借りた
だけの、カバーもボロボロの本だった。読んでみたら面白かったから、俺も
買おうと思ったのだが、それでどうしたかと言うとくぁwせdrftgyふじこlp

358 名前:132人目の素数さん :2005/09/27(火) 01:29:25
>>357
師父、この若輩者めに是非ご教授ください

359 名前:132人目の素数さん :2005/09/27(火) 05:55:43
Challenging Mathematical Problems With Elementary Solutions (Challenging Mathematical Problems with Elementary Solutions)
A. M. Yaglom (著), I. M. Yaglom (著)
http://www.amazon.co.jp/exec/obidos/ASIN/0486655377/qid=1127767883/sr=8-6/ref=sr_8_xs_ap_i6_xgl14/249-7205889-6621901
価格: ¥1,358 (税込)

英訳はこれだと思うよ

>>347>>303の同値性を知るためには
射影幾何の双対原理とかいったことを勉強すれば良い

360 名前:132人目の素数さん :2005/09/27(火) 07:16:52
フラクタルならありそう?

361 名前:132人目の素数さん :2005/09/27(火) 07:37:43
6角形にやれば平面を3交点で埋め尽くせるじゃないか

362 名前:132人目の素数さん :2005/09/27(火) 07:45:47
>>361
>>349

363 名前:132人目の素数さん :2005/09/27(火) 12:04:53
347は数学オリンピック本選の問題。
>>305によると303は6年くらいしてやっと証明されたらしいが、
そんな問題を本選に出す数オリは外道だな。

364 名前:132人目の素数さん :2005/09/27(火) 16:26:51
あ、いや、記憶違いかもしれない。でも たぶんそう。もしかしたら、単に
誰も本気で解こうとしなかっただけかも。

数オリって、たまに おかしな出題するよな。2004年の予選問題12番とか。
ハッピーエンド問題まんまだった。エスターが予想して、その2週間後に
セケレシュが巧妙な証明(しかもラムゼー理論の再発見になってるらしい)で
証明して、その後すぐにエルデシュがラムゼー理論を使わない証明を発見したとか。

365 名前:132人目の素数さん :2005/09/28(水) 06:22:11
n次の実正方行列A=(aij)とn次対称群Snを考える。任意のf∈Snに対して
Π[iー1〜n]aifi=0が成り立つための、Aの必要十分条件を求めよ。

366 名前:132人目の素数さん :2005/10/03(月) 00:48:31
裏面が白紙で、表面に数字 1, 2, 3, 4 の書かれたカード (1), (2), (3), (4) がある。
はじめ、机の上に4枚とも表にしておく。1回の操作において、無作為に2枚を裏返す。
n回の操作の後、すべてのカードが表を向いている確率を求めよ

367 名前:132人目の素数さん :2005/10/03(月) 01:20:00
>>366
エロ難しいのですが…
ヒントくれ!

368 名前:132人目の素数さん :2005/10/03(月) 01:33:32
>>366
カードに番号がかいてあることに意味あんの?
n回目の操作後にカードが表0枚の状態をXn、表2枚の状態をYn、表4枚の状態をZnとおく。
あきらかにP(X1)=P(Z1)=0、P(Y1)=1。対称性からすべてのnでP(Xn)=P(Zn)。
P(A∩B)/P(B)をP(A|B)とかく。遷移行列は
P(X(n+1)|Xn)=0、P(Y(n+1)|Xn)=1、P(Z(n+1)|Xn)=0、
P(X(n+1)|Yn)=1/6、P(Y(n+1)|Yn)=2/3、P(Z(n+1)|Yn)=1/6、
P(X(n+1)|Zn)=0、P(Y(n+1)|Zn)=1、P(Z(n+1)|Zn)=0、
P(X(n))=an、P(Yn)=bn、P(Zn)=cnとおくと次の漸化式をえる。
a(n+1)=(1/6)bn、b(n+1)=an+(2/3)bn+cn、c(n+1)=(1/6)bn
といて
an=(1/4)-(3/4)(1/3)^n、bn=(1/2)+(3/2)(1/3)^n、cn=(1/4)-(3/4)(1/3)^n

369 名前:132人目の素数さん :2005/10/03(月) 01:39:40
>>368
元の問題は、n回の操作で、(1)のカードが表を向いている確率を求めよ。
というのがありました

370 名前:132人目の素数さん :2005/10/03(月) 01:49:36
>>369
それは、n回の操作で、(1)のカードが表を向いている確率をPnと置くと
P0=1、P1=1/2
一回の操作で、(1)のカードがひっくり返る確率は1/2なので
Pn=(1/2)P(n-1)+(1/2)(1-P(n-1))=1/2
n≧1でPn=1/2は簡単に得られますな

371 名前:132人目の素数さん :2005/10/03(月) 02:04:57
>>370
そうですね。
そこで、遷移行列とか難しいことを考えずに出来ないかなと考えているのですが…

372 名前:132人目の素数さん :2005/10/03(月) 02:41:15
素数を小さい方から順にp1,p2,…,pn,…とおき、xn=p1p2…pnとおく。
自然数1,2,…,xnから無作為に数を1つ取り出し、それをMとする。N∋i≦nに対して、
Qi:pi|Mを満たす  とおく。
(1)P(Qi)P(Qj)=P(Qi∩Qj) (i≠j)を示せ。
(2)P(¬Qi)P(¬Qj)=P(¬Qi∩¬Qj)を示せ。
(3)Mがどのpi (i=1〜n)でも割り切れない確率を求めよ。
(4)自然数1,2,…,xnのうち、どのpi(1≦i≦n)でも割り切れない数はいくつあるか。

373 名前:372 :2005/10/03(月) 03:19:33
ゴメン。(2)と(3)が全然繋がってなかったわ('A`)

374 名前:132人目の素数さん :2005/10/03(月) 04:04:10
>>365
少なくとも一つの列または行が全部0

375 名前:132人目の素数さん :2005/10/03(月) 04:10:00
001
001
111


376 名前:132人目の素数さん :2005/10/03(月) 04:20:01
>>372
それぞれの素数で割った余りの組を考える。


377 名前:132人目の素数さん :2005/10/03(月) 05:21:21
>>368
bn=(3/4)+(1/4)(-1/3)^(n-1) では?

378 名前:132人目の素数さん :2005/10/03(月) 05:24:20
>>366
> 裏面が白紙で、表面に数字 1, 2, 3, 4 の書かれたカード (1), (2), (3), (4) がある。
> はじめ、机の上に4枚とも表にしておく。1回の操作において、無作為に2枚を裏返す。
> n回の操作の後、すべてのカードが表を向いている確率を求めよ

n回の操作の後、(1)、(2) のカードが表を向いている確率を求めよ。
としたら面白いのではなかろうか?

379 名前:132人目の素数さん :2005/10/03(月) 10:38:42
>>375
おっと間違った。
また考える。

380 名前:132人目の素数さん :2005/10/04(火) 00:03:26
>>377
ちゃんと計算してないが、bn=1-an-cnが成り立つだからぱっと見、>>368でよさげな気がするぞ。

381 名前:132人目の素数さん :2005/10/04(火) 01:29:35
全部間違ってるのにそんなのが成り立ってたって意味がない

382 名前:132人目の素数さん :2005/10/04(火) 01:42:56
>>381
そだな、失礼。 ちゃんと計算したらan、cnが間違ってるな。

383 名前:132人目の素数さん :2005/10/08(土) 00:11:00
競馬ゲームを作ろうとして、ふと思いついたんですが。
たとえば馬が6頭走るとして、すべての馬のオッズが100倍とかだったら
全部の馬に賭ければ必ず儲かってしまうわけで。
そのような事態を避けたいわけですが、
どのような範囲でオッズを決めたらよいですか。
単勝だけ考えてください。



384 名前:132人目の素数さん :2005/10/08(土) 00:22:04
>>383
勘で1/ai≧1。

385 名前:132人目の素数さん :2005/10/08(土) 01:20:23
それぞれの馬が勝つ確率 pi のとき
オッズを 1/pi にすればいい。


386 名前:132人目の素数さん :2005/10/08(土) 09:16:23
それに寺銭も考慮して8掛け

387 名前:132人目の素数さん :2005/10/10(月) 17:16:51
テラ銭2割ばぼりすぎ

388 名前:132人目の素数さん :2005/10/10(月) 22:47:16
JRAは2割5分だが。

389 名前:132人目の素数さん :2005/10/11(火) 08:42:28
1 から n までの数字の書かれた玉が2個ずつ合計 n 個袋に入っている。
無作為に2個ずつ取り出し、数字が同じなら捨て、異なるなら袋に戻す操作を
なくなるまで繰り返すとき、なくなるまでに行う操作の平均回数を求めよ。

390 名前:132人目の素数さん :2005/10/11(火) 09:08:00
袋に玉が2k個ずつ入っている状態を Sk とする。
Sk から S{k-1} に状態遷移するまでに
必要な試行回数をあらわすランダム変数を Xk,
N=嚢k (k=1からn)とすると、求めるものはE[N]=忍[Xk].

Sk から S{k-1} に状態遷移する確率は k/2kC2 = 1/(2k-1) で
平均回数はこの逆数になるので
E[Xk] = 2k-1
よって
E[N]=(2k-1)= n(n+1) - n = n^2

391 名前:132人目の素数さん :2005/10/11(火) 09:19:22
>>389
× : 1 から n までの数字の書かれた玉が2個ずつ合計 n 個袋に入っている。
○ : 1 から n までの数字の書かれた玉が2個ずつ合計 2n 個袋に入っている。

書き間違いでした。

392 名前:132人目の素数さん :2005/10/11(火) 11:18:45
>>390
正解です。さすが!

393 名前:132人目の素数さん :2005/10/11(火) 12:13:09
50枚のカードで神経衰弱をランダムにやると2500回以上かかるということか。
過去に引いたカードを全部記憶できるとすると、35回くらいまでは落とせそうだな。

394 名前:132人目の素数さん :2005/10/11(火) 12:43:48
>>393
> 過去に引いたカードを全部記憶できるとすると、35回くらいまでは落とせそうだな。

師父、その計算を、もっと詳しく!

395 名前:132人目の素数さん :2005/10/12(水) 01:47:31
13^(13^13) を 11 で割った余りを求めよ。

396 名前:GiantLeaves ◆6fN.Sojv5w :2005/10/12(水) 08:15:59
talk:>>395
13^(13^13)=13^((10+3)^13)=13^((10+3)^13-3^13+3^13)=13^((10+3)^13-3^13+3*(10-1)^6)
=(11+2)^(10(∑_{k=0}^{12}(13!/k!/(13-k)!*10^(12-k)*3^k)+3∑_{k=0}^{5}(6!/k!/(6-k)!*10^(5-k)*(-1)^k))+3)
フェルマーの定理より、13^(13^13)を11で割ったあまりは8.

397 名前:132人目の素数さん :2005/10/12(水) 22:23:00
アルゴリズムの問題なんですが。
ツリー状のグラフTが与えられたとして、
Tの部分グラフのなかで連結なものの数を求める
多項式時間アルゴリズムを示せ。

398 名前:132人目の素数さん :2005/10/12(水) 22:35:00
一つの点を決めて遠い方から順に数えていく。


399 名前:132人目の素数さん :2005/10/12(水) 22:48:32
Σ_[p+q+r=n] C[p+q,p]C[q+r,q]C[r+p,r] = Σ_[s=0,n] C[2s,s]
を示せ。

400 名前:132人目の素数さん :2005/10/12(水) 23:00:04
>397
同型なグラフは二重に数えてよいんだよね?

頂点 v を含むのと含まないのとを別々に考えていけばいいんじゃない?
含むほうは、各部分木をどこでちょん切るかを数えればいいだけだし、
含まないほうはG-vに再帰的にアルゴリズムを適用するだけ。

401 名前:132人目の素数さん :2005/10/12(水) 23:01:03
>G-vに
G-v の各連結成分に、の間違い。

402 名前:132人目の素数さん :2005/10/12(水) 23:05:52
>>400
ちゃんと多項式時間で計算できるかが問題。
下手すると指数時間になる。


403 名前:未解決(個人的に) :2005/10/12(水) 23:51:13
p、q、r、s、t、uは全て素数とする。

(1) p*q+r*s=t*uなる組(p、q、r、s、t、u)は無数に存在するか。
(2) p*q+r*s=t なる組(p、q、r、s、t)は無数に存在するか。


具体例はいくらでもありますが、無数にあるかが何とも言えません。

404 名前:132人目の素数さん :2005/10/13(木) 00:16:40
>402
v に対する再帰の実行回数は、高々 n(各頂点につき一度しか実行されない)。
部分木をどこでちょんぎるか、のほうは確かに説明不足でしたね。

これも再帰的にやります。
v の部分木の一つを T' (vも含むようにとる)とします。
v の次数は 1。v の子を u として、u の各部分木(u を根とする)に対して
アルゴリズムを再帰的に実行して数え、それらを掛け合わせたもの + 1 が
T' のちょん切り方の個数。(+1 は v-u 間でちょん切る場合の数)

計算時間は全部でO(n^2)。
v の選び方を工夫すれば、もっと減らせそうな気はするけど、よくわからない。
ランダムな選び方は、例えばk分木の時にはあまり速くならない(ほとんど葉だから)。

405 名前:132人目の素数さん :2005/10/13(木) 03:20:09
と、思ったけど、O(n) でできるのか。
最初に根となる頂点を一つ選んで、
枝を根から葉の方向に向きをつけておく。
一つ目の再帰では、常に現在見ている部分木の根を v としてとるようにすれば、
ちょん切り方を数える再帰では、各頂点ごとに一回だけ数えて覚えておけば再利用できる。

どちらの再帰も各頂点ごとに一度づつ実行されるので、トータルで O(n)。

406 名前:132人目の素数さん :2005/10/13(木) 19:07:26
>>399
これムズイ。ヒントだしてやってチョンマゲ。

407 名前:132人目の素数さん :2005/10/13(木) 22:17:21
>>406
俺漏れも!

408 名前:397 :2005/10/13(木) 22:45:16
>>405
素晴らしいです。ところでこれが一般のグラフになると
とたんに難しくなるわけですが、チャレンジしてみませんか。


409 名前:132人目の素数さん :2005/10/13(木) 22:56:30
>>408
一般のグラフでも結論として多項式時間で計算できることはいえんの?
それとその場合のグラフは2重辺とかループとかあり?

410 名前:397 :2005/10/13(木) 23:06:06
>>409
じつは多項式時間で計算できるかどうかも私にはわかっていません。
正直私は挫折しました。
2重辺やループは無しということで。


411 名前:132人目の素数さん :2005/10/13(木) 23:15:03
>>410
じゃあ問題はこう?
−問題−
与えられた有限集合VとVの2元集合の族Eの組G=(V,E)に対し
Gの部分グラフの数をあたえる関数f(G)はGの点の数、つまり#Vに関して多項式時間で
計算できるか?つまりあるfを計算するアルゴリズムTと多項式Pが存在して任意のGに対して
(Tがf(G)を計算する時間)≦f(Gの点の数)
が成り立つか?
−−
で桶?まあ計算論ホントにやってるひとがみたらつっこみどこ満載なんだろうけど。
まあ門外漢の感覚的お遊びってことで。

412 名前:397 :2005/10/13(木) 23:22:57
そうですね。
それでOKだと思います。
他力本願だが誰か解いてくれないかなー。

413 名前:132人目の素数さん :2005/10/14(金) 00:42:33
>>399
ヒントくらはいアゲ。

414 名前:132人目の素数さん :2005/10/14(金) 00:57:29
>411
#P-complete なんじゃない?
多項式で解ける気がしないなぁ。

415 名前:132人目の素数さん :2005/10/14(金) 01:19:22
#P-completeってNP完全って香具師?そんな略しかたするん?NPのNってnumber?

416 名前:132人目の素数さん :2005/10/14(金) 01:20:39


417 名前:132人目の素数さん :2005/10/14(金) 01:31:18
>415
NP は non-deterministically polynomial。

#P は大雑把に言うとものを数える、という問題のクラスで、
#P-complete は #P の中で完全
(その問題が多項式で解ければどの#P問題も多項式で解ける)
な問題の集合。
例えば SAT の式がいくつ解を持つか?という問題は #P-complete。

#P は NP よりも難しい。
NP は、答えが一つ以上あるかどうかを問う問題のクラスであり、
#P は答えがいくつあるかを問う問題のクラスだから。

418 名前:132人目の素数さん :2005/10/14(金) 01:48:31
>>417
( ・∀・)つ〃∩ ヘェーヘェーヘェーヘェーヘェーヘェーヘェーヘェーヘェー。ひとつ賢なった。thx。

419 名前:132人目の素数さん :2005/10/14(金) 02:23:08
xf(f(x))=1をみたす関数f(x)を求めよ

420 名前:132人目の素数さん :2005/10/14(金) 05:14:36
>>419
f(x) = x^i

421 名前:sage :2005/10/14(金) 08:51:42
x(x^i)^i=1。なるほど。で一般形は?

422 名前:132人目の素数さん :2005/10/14(金) 13:10:34
>>421
それが教えを請う者の態度ですか?
チンコ洗って出直して来なさい。

423 名前:132人目の素数さん :2005/10/14(金) 19:04:06
>>421
一般形なんかもとまらんような気がする。f(f(x))=1/x。すくなくとも
A^2=[[0,1],[1,0]]である行列用意しといてAで表示されるメビウス変換とかは全部これの解だし。
もちろんこれで全部じゃないし。
>>399
ヒントくらはい。

424 名前:132人目の素数さん :2005/10/14(金) 23:53:28
>>365
行列のサイズをnxnとしてa+b>nととり、
axbの零行列をOとする。A,B,Cをax(n-b),(n-a)xb,(n-a)x(n-b)行列として


|OA|
|BC|

を考え、これに行の入れ替え、列の入れ替えをおこなったものです。証明は
zeroでない要素をあればとり、仮に1,1成分として、(n-1)x(n-1)小行列
について帰納法の仮定を使う。さらにこれを利用してブロックわけして
仮にn=p+qとして、pxp, qxqについて帰納法の仮定をもちいる。

425 名前:365 :2005/10/15(土) 03:19:56
正解です。これを使って、次の問題が簡単に解けます。

k種類の石がn個ずつある。これらkn個の石を、k個の箱にn個ずつ入れる。
どんな石の入れ方に対しても、これらk個の箱から1つずつ石を選んで(合計で
k個の石を選ぶ)、k種類の石を全て取り出すことが出来ることを示せ。
(初等的に解いた高等数学の問題3より)

本には別の解答が載っていました。この定理と、これを少し一般化した定理は、
現代数学の多くの問題に対して重要らしいです。でもそんなの見たことないよorz

426 名前:132人目の素数さん :2005/10/15(土) 06:12:19
>425
このスレの趣旨には合わないかもしれないけど、
Hall の結婚定理を使っても簡単に解けるね。

>これを少し一般化した定理
ってのは本に載ってるの?
だとしたら、うぷしてもらえませんか?

427 名前:426 :2005/10/15(土) 06:22:36
これだけでは何なので、問題を出してみます。
以下のような半無限(上と右方向に無限)な平面があります。
一番左下に●が一つだけ置かれています。

++++++
++++++
++++++
++++++
●+++++

これに対して、次のような操作を考えます。

*盤上の●を一つ取り除き、代わりに●が有ったところの一つ上と一つ右に新しい●を配置
+++  +++
+++  ●++
●++  +●+

このとき、●の上や右に既に別の●が有った場合はこの操作を行うことができません。

以上の操作を繰り返して、左下の3x3のマスから●を除くことができるでしょうか?
できる場合はその手順を、できない場合はその証明をしてください。

428 名前:132人目の素数さん :2005/10/15(土) 07:25:23
>427
>できる場合はその手順を、できない場合はその証明をしてください。
つまり、できないということだな

429 名前:132人目の素数さん :2005/10/15(土) 08:01:11
>>428
どうして、そういうことを言うのかなぁ…

430 名前:132人目の素数さん :2005/10/15(土) 14:37:38
2x2 からは丸をなくすことはできたけど
それ以上は一マスもススメそうにないなぁ.

431 名前:132人目の素数さん :2005/10/15(土) 16:27:19
このように考えるのはどうだろう?

>このとき、●の上や右に既に別の●が有った場合はこの操作を行うことができません。
まず↑このルールを無視して●の重なりを許すことにする
(元のルールでもこの無視したルールの手数以上かかるのは明らか。)

このとき3x3のマスから●をすべて追い出すのに最低かかる手数を考えると19手かかる。(詳細は略)
一手に付き一個●が増えるのだから、つまりその時点で3×3のマスの外に20個の●があることになる。

さて、この20個の●は、3×3のマスの外に出すことしか考えられていないから19手では4×4より外にはない。
ところが3×3のマスの外でかつ4×4のマスの中には7個の●しか存在できないので
20個の●のうち少なくとも13個は4×4の外に追い出すためにさらに移動をしなくてはならない。
もちろんその移動で13個の●は倍の26個に増えているはずだ。

同じように
26個 の●のうち5×5のマスの中にいられるのは9個だけ。少なくとも17個は5×5の外にあるのでその倍の
34個 の●のうち6×6のマスの中にいられるのは11個だけ。少なくとも23個は6×6の外にあるのでその倍の
46個 の‥
そのようにして、このゲームを終えるためにはこの数列がどこかで0以下にならないといけないはずなのだが‥




432 名前:431 :2005/10/15(土) 16:34:33
訂正

これを
さて、この20個の●は、3×3のマスの外に出すことしか考えられていないから

これに
さてここで元の●を重ねてはならないルールに立ち戻る。
この20個の●は、3×3のマスの外に出すことしか考えられていないから



433 名前:132人目の素数さん :2005/10/15(土) 18:49:29
>>427
面白い!
正攻法でやったら解けました。

434 名前:132人目の素数さん :2005/10/15(土) 19:56:02
>>399
降参でつ。答えおしえてください。

435 名前:132人目の素数さん :2005/10/15(土) 20:04:50
>>433 いや、だから解けないんだって

436 名前:132人目の素数さん :2005/10/15(土) 20:41:00
昔解いた。


437 名前:132人目の素数さん :2005/10/15(土) 20:58:16
碁石と碁盤使えば良いんじゃね?

438 名前:132人目の素数さん :2005/10/15(土) 21:14:40
>>435
ん?
否定的に解いたよ。

439 名前:132人目の素数さん :2005/10/15(土) 21:34:36
   〃∩ ∧_∧
   ⊂⌒(  ・ω・)  はいはいワロシュワロシュ
     `ヽ_っ⌒/⌒c     
        ⌒ ⌒

440 名前:132人目の素数さん :2005/10/15(土) 22:30:13
誤爆?

441 名前:132人目の素数さん :2005/10/16(日) 00:59:34
〉〉438 ではここに答えをぜひ

442 名前:132人目の素数さん :2005/10/16(日) 01:10:00
それぞれの位置に数を与えて●のある位置の
数の合計が一定になるようにする。
最初の位置に1を与えそこから一つ進むごとに1/2倍の数を与えれば
全部で4で3×3の部分は3+1/16だから全て出すことはできない。


443 名前:132人目の素数さん :2005/10/16(日) 01:30:28
さっぱりわからないのでおしえてください
全部で4、の全部とはどれのことですか?
3+1/16だと出すことができないというのはどういう理屈ですか?

444 名前:132人目の素数さん :2005/10/16(日) 01:34:29
ああ、すまん今考え直したらわかったよ
全部で4ってのは全ての升目の合計ってことで
そこから3×3の部分を除くと1未満になるから
全部を出すことはできないってことだね。

445 名前:132人目の素数さん :2005/10/16(日) 01:37:51
全部で4にはならんと思うのだが‥

446 名前:445 :2005/10/16(日) 01:40:14
いやすまん。なる。激しく計算ミスをしていた。


447 名前:132人目の素数さん :2005/10/16(日) 01:56:01
>442
なるほどねぇ。

じゃぁ、途中の段階では●同士が重なっても良い、としても解けないのか。

448 名前:132人目の素数さん :2005/10/16(日) 02:07:15
そういえば似た問題がピーターフランクルの問題集にあったね

449 名前:132人目の素数さん :2005/10/16(日) 02:22:01
ソリティア(トランプじゃないヨ)の1列バージョンだったっけ。α=(√5−1)/2を使ってたはず。
こういうのって どうやって見つけるのかな。

450 名前:132人目の素数さん :2005/10/16(日) 02:25:22
>>447
重ねるのを許す場合は 解けるのでは?同じマスにいくつもの石を乗せられるから、
3×3の部分を除いても1未満になるとは限らない。例えば、1/32が書いてあるマスに
33個の石が重なっていたら、和は1より大きい。

451 名前:132人目の素数さん :2005/10/16(日) 02:25:31
不変量って考え方に思い当たれば
普通に気付くんじゃないかな

452 名前:132人目の素数さん :2005/10/16(日) 02:36:01
問題文を別の表現で言ってみる。

一番左下に1グラムの粘土が一つだけ置かれている。これに対して、次のような操作を考える。
「盤上の粘土を1つ選び、それを2つに等分割し、元の粘土があった場所の一つ上と一つ右に移動させる。
ただし、粘土の上や右に既に別の粘土が あった場合は、この操作を行えない。」

…答え言ってるような ものだな。

453 名前:132人目の素数さん :2005/10/16(日) 02:40:02
生まれた?●をずっと上に追いやって右側に出来たやつは
それもまた上に追いやってって順繰りにやっても3×3の枡から出ることがないのか?

454 名前:452 :2005/10/16(日) 02:52:42
いやいや、そんなことは無いか。ヒントには なるだろうけど。

>>453
俺もそれ考えたけど、オセロの盤で試してみたら無理っぽかった。

455 名前:132人目の素数さん :2005/10/16(日) 03:17:52
>453
無理だよ。

上の議論からもうちょっとだけがんばって見ると

++
+++
からなくすこともできないことがわかる。

456 名前:132人目の素数さん :2005/10/16(日) 05:03:13
「もうちょっと」の詳細きぼん

457 名前:132人目の素数さん :2005/10/16(日) 07:36:54
>456
あんまり面白くはないんだけど、
マスを全部●で埋めても、ってあったけど、
実際には全部うめることができないでしょ。
例えば一列目には、どんな操作をしても、●は一つしか置けない。
だから一列目に置かれている●の合計値は高々1/8。

こんな風に、より正確に数えてやると
さらに小さな領域でも無理だということがわかる。

458 名前:132人目の素数さん :2005/10/16(日) 10:06:49
だから、●のある場所の重みは常に1なの。

459 名前:132人目の素数さん :2005/10/16(日) 10:10:18
問題豆乳
半径1の円周上にn個の点を任意にとる。この円周上に別
の点zをとり、点zからn個の点までの距離の積を作る。
点zを円周上のどこにとっても、この積が2以下であれば、
最初にとったn個の点は正n角形の頂点になることを示せ。

460 名前:132人目の素数さん :2005/10/17(月) 14:07:34
>>399
texでもないと証明が読めないのでtexで。(1)
$$\sum_{p\geq 0}\frac{(p+q)!}{p!q!}x^p=\frac{1}{(1-x)^{1+q}},
\sum_{p\geq 0}\frac{(p+r)!}{p!r!}(\frac{t}{x})^p=
\frac{1}{(1-\frac{t}{x})^{1+r}}$$
から$0<t< k <1$ を固定して, $C:\mid x \mid =k$に関する周回積分
に関する等式
$$\sum_{p\geq 0}\frac{(p+q)!}{p!q!}
\frac{(p+r)!}{p!r!}t^p
=\frac{1}{2\pi i}
\int_C\frac{1}{(1-x)^{1+q}(1-\frac{t}{x})^{1+r}}\frac{dx}{x}$$
を得る。これに$\displaystyle\frac{(q+r)!}{q!r!}t^{q+r}$をかけて
$q+r=m, q\geq 0, r\geq 0$に関して加えれば、
\begin{align*}
& \sum_{q+r=m, q\geq 0, r\geq 0}\sum_{p\geq 0}\frac{(p+q)!}{p!q!}
\frac{(p+r)!}{p!r!}\frac{(q+r)!}{q!r!}t^{p+q+r} \\
= &\frac{1}{2\pi i}
\int_C\frac{1}{(1-x)(1-\frac{t}{x})}
(\frac{1}{1-x}+\frac{1}{1-\frac{t}{x}})^m
\frac{dx}{x} t^{m}
\end{align*}

461 名前:132人目の素数さん :2005/10/17(月) 14:08:14
>>399
texでもないと証明が読めないのでtexで。(2)
上の式を(1)として
$m$に関する和を取るのだが、その前に$C$を$\bold C-\{1,t\}$でのhomotopy
equivalentな$D$で取り換えて、あとの都合上、$D$上で
$$\mid x-t \mid \geq\frac{1}{8}, \mid 1-x \mid \geq \frac{1}{8},
\mid x \mid <1$$
としたいのだが、これは$t$が十分$0$に近いときに可能なのでこの様に
$t$を取り換えておく。さらに必要なら$t$を小くとり、$D$上で
$$\mid t \frac{x^2-2x+t}{(1-x)(x-t)} \mid <1$$
となるようにとる。こうしておいて、$m\geq 0$に関して和をとれば、
\begin{align*}
& \sum_{q+r=m, q\geq 0, r\geq 0}\sum_{p\geq 0}\frac{(p+q)!}{p!q!}
\frac{(p+r)!}{p!r!}\frac{(q+r)!}{q!r!}t^{p+q+r} \\
= &\frac{1}{2\pi i}
\int_D\frac{dx}{(t-1)(x^2-x+t)} \\
=&\frac{1}{1-t}(1-4t^2)^{-\frac{1}{2}}
\end{align*}
ここで
$$(1-4t^2)^{-\frac{1}{2}}=\sum_{i \geq 0}\frac{(2i)!}{i!i!}t^{2i}$$
をもちいて$t^n$の係数を比較して定理を得る。


462 名前:132人目の素数さん :2005/10/17(月) 17:25:30
>>461
訂正 1-4t^2を1-4tに


463 名前:132人目の素数さん :2005/10/18(火) 10:36:09
>>399
やっとできたと思ったら答えでてたのか。せっかくなので書いてみる。TeXじゃないし。
納p+q+r=n]C[p+q,p]C[q+r,q]C[r+p,p]をanとおく。
非負整数nに対して多項式Pnを
 Pn=納r=0,n](xy(x+y+2))^(n-r)・((x+1)(y+1))^r
とおく。まずこの(xy)^nの項の係数がanに等しいことをしめす。
(x+y+2)^(n-r)=(x+1+y+1)^(n-r)に2項定理を使って
 Pn=(xy)^(n-r)納r=0,n]納p=0,n-r]C[n-r,p](x+1)^(p+r)・(y+1)^(n-p)
であるが(x+1)^(p+r)・(y+1)^(n-p)の(xy)^rの係数は
C[p+r,r]C[n-p,r]に等しい。よって主張が得られた。
容易に
 Pn=xy(x+y+2)P[n-1]+((x+1)(y+1))^n――(1)
  =(x+1)(y+1)P[n-1]+(xy(x+y+2))^n――(2)
であるから(xy)×(2)-(1)により
(xy-1)Pn=(xy)^2-xy)P[n-1]+(xy(x+y+2))^n-((x+1)(y+1))^n
 ∴Pn=xyP[n-1]+((xy)^(n+1)・(x+y+2)^n-((x+1)(y+1))^n)/(xy-1)
そこで右辺第2項の(xy)^nの項を計算する。
 ((xy)^(n+1)・(x+y+2)^n-((x+1)(y+1))^n)/(xy-1)
 =((xy)^n+(xy)^(n+1)+・・・+1)(x+y+2)^n+((x+y+2)^n-((x+1)(y+1))^n)/(xy-1)
であるが第2項はx,yに関する次数が2n未満の多項式であるので(xy)^nの項は現れない。
第1項の(xy)^nの係数は(x+y+2)^nの(xy)^iの形の項の係数の総和にひとしい。
それはx=t^2,y=1/t^2を代入したときの定数項に等しいがそれは
(x+y+2)^n=(t+1/t)^2nであるゆえC[2n,n]にひとしい。
以上よりPn-xyP[n-1]の(xy)^nの項の係数はC[2n,n]に等しい。
P0=1の定数項は1であるゆえC[0,0]に等しくよって以上により
an=納s=0,n]C[2s,s]となる。□

464 名前:132人目の素数さん :2005/10/18(火) 11:24:22
>>463
君こそ英雄だ! 最大の功績だ!

465 名前:132人目の素数さん :2005/10/18(火) 15:53:54
Σ[k=0,n] (C[n,k])^3 = Σ[k=0,n] (C[n,k])^2 C[2k,n] を示せ。

466 名前:132人目の素数さん :2005/10/18(火) 22:43:02
>>465
C[2k,n]?これだと2k<nのときこまるとおもうんだけど。

467 名前:132人目の素数さん :2005/10/18(火) 23:13:07
0って定義するんじゃなかったか?
それで等式が成立するかどうかは確かめてないけど

468 名前:132人目の素数さん :2005/10/22(土) 09:43:49
ABCDに正の整数を入れて等式を成立させて下さい

(A÷B)の3乗+(C÷D)の3乗=17

469 名前:132人目の素数さん :2005/10/22(土) 09:51:13
6頭立ての競馬で,1位になる確率が

馬1:1/3
馬2:1/4
馬3:1/5
馬4:1/6
馬5:1/30
馬6:1/60
とする
還元率を90%にしたい時,単勝と馬連の倍率をいくらにすればよいか?

470 名前:132人目の素数さん :2005/10/22(土) 10:14:52
>>465
マダァ-? (・∀・ )っ/凵⌒☆チンチン

471 名前:132人目の素数さん :2005/10/22(土) 11:10:47
単勝
単純に逆数に0.9掛ければ良くない?2.7倍、3.6倍・・・54倍
馬連は競馬知らんのでわからん。

472 名前:132人目の素数さん :2005/10/22(土) 11:14:00
>468
17は素数であって3で割って2あまるからQ(ω)における
因数分解を考えて、むりだよ。

473 名前:132人目の素数さん :2005/10/22(土) 11:40:41
無理じゃない
ちゃんと答えがある

474 名前:132人目の素数さん :2005/10/22(土) 11:45:30
間違えた。17の因子(a/b)+(c/d)からくることがある。


475 名前:132人目の素数さん :2005/10/22(土) 17:15:20
>468
正でなくてもよければ
(-1/7)^3+(18/7)^3=17なので楕円曲線x^3+y^3=17*z^3(原点を(-1,1,0))
の加法をつかって座標を正にできるのでは?epsilon因子は3と17のみ
計算すればきっとL関数のs=1での位数が奇数になるのでは?

476 名前:132人目の素数さん :2005/10/22(土) 17:34:27
>>471
馬連は1着と2着の組合せ(順不同)を当てる
1-3や5-6等
6頭立てなら15通り

477 名前:132人目の素数さん :2005/10/22(土) 19:00:00
(104940/40831)^3+(11663/40831)^3=17.


478 名前:132人目の素数さん :2005/10/23(日) 22:46:32
>468
x^3+y^3=17*z^3から(1,-1,0)へのprojectionつかい変換すると
w^2=12*17*z^3-3. zを有理数で動かしてwが有理数となるものを
計算機でもとめる。もとの変数にもどすと、(x,y)=(-1/7,18/7)
と比較的小さい解がもとまる。(-1/7,18/7)でのtangentともとの
曲線の交点をもとめ、(104940/40831, 11663/40831)を得る。


479 名前:132人目の素数さん :2005/10/24(月) 00:17:11
>419
f(0)は定義されない。
p -> q -> 1/p -> 1/q -> p ...(*)
とくるので、R-{0}をz〜1/zという同値類別してそれをAとおく。pの類を[p]
と書くと[1],[-1]は別に考えて、A-{[1],[-2]}を二つづつ組にし[p],[q]
とおいて上の(*)でfを定める。これは全て方程式をみたす。



480 名前:132人目の素数さん :2005/10/27(木) 14:29:39
x-y平面上の部分領域0≦x,y≦1上に四点A,B,C,Dをとる。
A,B,C,Dの座標を一様分布に従い考えるとき、A,B,C,Dの凸包の面積の期待値を求めよ。

481 名前:132人目の素数さん :2005/10/27(木) 23:07:31
n≧2の時 M_n=4^n-1 とする。
この時k個の素数の積となる M_n=(p_1)^(l_1)*(p_2)^(l_2)…(p_k)^(l_k)
l_1、l_2、…l_kは適当な自然数である。

4^(n-1)≧k^2 を証明せよ

482 名前:132人目の素数さん :2005/10/28(金) 06:13:59
>>481 M_kの相異なる素因数の個数の評価だとしたら、それは粗いんじゃないか?

M_n は必ず3で割り切れて、それ以外の素因数(あるとすれば)は5以上だから、M_n≧3*5^(k-1)。
一方、M_n≦3*5^(n-1) が成立することは帰納法で容易に示せる。よって k≦n。
ここから k≦2^(n-1) であることはすぐに出る。

483 名前:132人目の素数さん :2005/10/28(金) 10:46:18
>>480
ムズ杉。3点でも充分、問題として成立する気がする。

484 名前:132人目の素数さん :2005/10/28(金) 21:10:15
ランダムな時間にTVをつけて朝ズバがやってる確率を求めよ

485 名前:132人目の素数さん :2005/10/28(金) 21:19:50
まず朝ズバの定義をキボン。


ぶっちゃけ、その番組を知らない。

486 名前:132人目の素数さん :2005/10/28(金) 21:41:54
限りなく0に近い

487 名前:132人目の素数さん :2005/10/29(土) 03:43:36
問1 http://www.uploda.org/file/uporg225338.jpg
問2 http://www.uploda.org/file/uporg225340.jpg
問3 http://www.uploda.org/file/uporg225367.jpg
問4 http://www.uploda.org/file/uporg225370.jpg
問5 http://www.uploda.org/file/uporg225389.jpg
問6 http://www.uploda.org/file/uporg225390.jpg
問7 http://www.uploda.org/file/uporg225393.jpg
問8 http://www.uploda.org/file/uporg225394.jpg

488 名前:132人目の素数さん :2005/10/29(土) 04:58:29
>>487
算数の問題
http://science4.2ch.net/test/read.cgi/math/1055763431/l50


489 名前:132人目の素数さん :2005/10/31(月) 01:10:45
>>483
3点で、たぶん11/72。

490 名前:132人目の素数さん :2005/11/01(火) 17:58:20
前前スレ814の問題を考えてたんだけど555個でいいんでしょうか?
http://www3.tokai.or.jp/meta/gokudo-/omoshi-log/1074751156.html
バナナを消費するタイミングが書いてないので誤差があるかもしれないけど。
よろしくお願いします。

491 名前:132人目の素数さん :2005/11/01(火) 20:47:29
>>490
確かにバナナの消費タイミングを厳密に決める必要があるな。
たとえばこんな感じか?

・道中には1キロごとに石碑が立っていて、バナナを保存できるのはその石碑のところだけとする。
・また、進むか引き返すかを選択できるのも石碑地点だけとする。
・さらに、バナナを食うのは必ず石碑と石碑の中間点とする。

つまりn本のバナナを持って石碑を出発すれば、次の石碑までは引き返すことはできず、
次の石碑で手持ちバナナはn-1本になっている。
もし空手で出発すれば、たとえ次の石碑に山のようなバナナが保存してあっても、
途中でのたれ死ぬ。

‥とか設定した上で、行程10キロ、バナナ30本、最大手持ち10本
くらいに単純化して考えてみてはどうだろう。5本いける?

492 名前:132人目の素数さん :2005/11/01(火) 21:01:35
転載しましょ。答えでてないしね。
 
814 名前:132人目の素数さん :04/08/23 23:11
これぞまさに「面白い問題」かな
 
:問題
あなたは1000kmの砂漠を横断して隣町まで行きます
今3000個のバナナを持っています
1km毎に1個のバナナを食べないと死にます
一度に運べるバナナは最大で1000個です
この状況で、あなたは最大何個のバナナを隣町まで運べるでしょうか?
 
 
これで意味わかる?
つまり最初に1000個のバナナを運んで、途中に置いて、一旦戻ってまた行かないと隣町に着いても全部無くなる
 
まず、最初に1000個のバナナを運ぶ
途中の地点にバナナを何個か置いていく
その後引き返す(引き返している間にもバナナを食う必要がある)
そしてまた1000個のバナナを持って旅立つ
途中で置いておいたバナナを補給する
そんな感じ
考えてくだせえ

493 名前:490 :2005/11/01(火) 21:14:37
つまり1キロずつに区切ってその間にバナナを食い始め、食い終わるわけですね。
ということは1本持ってスタートして1km地点で折り返した場合、
スタート地点についてからそこにあるバナナを食べるのではなく帰り途中に死んでしまう、と。

>‥とか設定した上で、行程10キロ、バナナ30本、最大手持ち10本
>くらいに単純化して考えてみてはどうだろう。5本いける?

同じ考え方でやってみたところ3本しかもっていけないという結論になったけど・・・。
どうしても余りが出るんだけどしかたないのかなぁ?

494 名前:132人目の素数さん :2005/11/01(火) 21:42:48
「行程10キロ、バナナ30本、最大手持ち10本」の設定で、
「常に全てのバナナを次の地点に運ぶ」という作戦(*)でやると、4本はいけるよ。

(*) バナナが3点以上に分散しないようにする作戦。たとえば最初は、
スタート地点と1キロ地点を往復して、全てのバナナを1キロ地点に集める。
それには2.5往復必要なので、結果1キロ地点に集まったバナナは25本になる。
以後繰り返し。

5キロ地点で1本捨てることになるのがもったいない。何とかならんかねえ。

495 名前:490 :2005/11/01(火) 22:04:04
ああ、4本のまちがいだ。
でも>>493のやり方で5m地点まで1mずつもってくと5m地点に11本あつまるから
そこから10本もってゴールに直行すれば5本行ける?

496 名前:132人目の素数さん :2005/11/01(火) 22:11:33
結局>>492の問題で>>491の解釈のもとでの現在わかっている可能な最大数っていくつ?
現時点で「n本までは可能」が証明できた最大の数って結局いくら?
みんなどれぐらいの答えもってるの?

497 名前:132人目の素数さん :2005/11/01(火) 22:51:30
[1] 1,000本のバナナを 持って 1本食って 1km 先に行き、
  帰りに食う1本を抜いて998本置き、出発点に戻る。
[2] また 1,000本持って 998本置いて、出発点に戻る。
[3] また 1,000本持って1本食って 1km地点に行く。
  到着したとき、バナナは 998+998+999=2,995本。
[4] 1kmを2往復半して運ぶのである。
  1km進むごとにバナナは 5 本ずつ減っていく。
[5] 1,000÷5=200
  200km 進んだとき、バナナは 2,000本になっている。
[6] ここからは1往復半して運べばよい。
[7] 201km地点には 1,997本運べる。
  つまり、1km 進むごとに 3 本ずつ減る。
[8] 1,000÷3=333 余り 1
  200+333=533 km地点には、1,001本のバナナを運べる。
[9] もったいないが、1本は沙漠に残し、1,000本積んで
  ゴールをめざす。1 km進むごとに1本消費する。
[10]残りは 1,000−533=467 km である。
  1,000本のうち 467本食ってしまうから、
  到着時には 533本が残る。


498 名前:132人目の素数さん :2005/11/01(火) 23:30:20
>>497
なるほど。なんとなくこれ最大っぽい。後は534本ははこべないことの証明か。

499 名前:132人目の素数さん :2005/11/02(水) 15:24:39
考え方がわかってきたような。

1. 途中のある地点まで1000本もってスタートし、
戻るのに必要な本数を除いたあまりをすべて置いてくる、というプロセスを3回繰り返すとすると、
スタート地点からxメートル地点に全てのバナナを運ぶと
(1000-2x)*2+(1000-x) = 3000-5x 本のバナナを運ぶことができる。
ちなみに1mずつ2.5往復して運ぶのを繰り返したとしても結局
それぞれの1km区間について2.5往復しているので結果は同じ。

2. このとき、x>=200だと運べるバナナの本数が2000を切るわけだが、
2000を切るということは、ある一部分の区間について2.5往復という手間を省くことができる。
200を超えるxメートル地点に全てのバナナを運ぼうとした場合、
単純にそこまで2.5往復するよりも、一度200m地点に2000本のバナナを集め、
そこから1.5往復すれば、200mを越える区間について1往復分のバナナ消費量を省くことができる。
従って、200m地点から200+yメートル地点に運ぶことができるバナナの本数は
(1000-2y)+(1000-y) = 2000-3y

500 名前:499 :2005/11/02(水) 15:25:01
3. 同様にして、ある地点に運べるバナナの本数が1000本を切る場合も
一部の区間について消費量を省くことができる。
y>334だと1000本を切るので、(200+333)mを超える区間にバナナを運ぶ場合は
533m地点にバナナ(1001本)を集めてそこから片道で運ぶのが最も効率がよい。
また、これ以上は引き返してバナナを補充する必要がないので(1本だけ残る)
あとはゴール地点に行くだけである。
従ってゴールに運べるバナナの本数は
1000-(1000-533) = 533本

501 名前:132人目の素数さん :2005/11/02(水) 18:05:21
>>499-500
すまん。証明わからん。orz。これで証明になってんの?もうすこし詳しく。

502 名前:132人目の素数さん :2005/11/02(水) 18:18:59
問題もうすこし数学的に書きなおすと
 
非負整数からなる有限数列(ai) (0≦i≦1000)と0≦i≦1000の組
(i,ai)に次の操作がゆるされている。
操作I
i<1000のときaiをai-k、a(i+1)をa(i+1)+k-1におきかえiをi+1におきかえる。
ただしkは1≦k≦ai,1000をみたす整数
操作II
i>0のときaiをai-k、a(i-1)をa(i-1)+k-1におきかえiをi-1におきかえる。
ただしkは1≦k≦ai,1000をみたす整数
でaiをa0=1000、ai=0 (i>0)とおいて組(0,ai)に操作I、操作IIを何回かくわえてえられる組
(1000,bi)におきかえることができるもののなかでb1000の最大値をもとめよ。
 
とかになると思うんだけど。
ほとんどあきらかに操作Iではk=min{ai,1000} (つまり進むときは常に運べるだけ運ぶ)
かつ操作IIではk=1 (つまり戻るときには1本だけもってもどる)
に限定してもよさそうにはみえる。しかしどうやって証明していいかもわからんし。
操作IIを2回連続しない解のなかに最大値が存在するのもあきらかっぽいんだけど
どうやって証明すりゃいいかわからんしorz。

503 名前:499 :2005/11/02(水) 19:15:06
証明ではないけど自分なりの解法ということで。

ただ、バナナが2001〜3000本残っていたら全てを運搬するのに2.5往復せざるをえないし、
1001〜2000本残っていたら1.5往復する必要がある。
(放置するなら別だけど。)
533本以上バナナをゴールに運ぶということは道中のバナナの消費量を抑えなければいけない、
すなわち移動量を減らす必要があるけど、
これ以上に効率のよい歩き方というのはないといっていいんじゃないかなぁ?

504 名前:132人目の素数さん :2005/11/02(水) 19:32:51
>>503
それはオレもおもった。>>497の解は全部で2466工程で1本を砂漠に残してる。
運べる本数は3000-工程数-砂漠に残した数。結局534本はこべるとすれば
工程数が2465工程未満か2466工程で砂漠に残さないか。
どっちでも不能といえばいいんだけど。

505 名前:132人目の素数さん :2005/11/03(木) 01:14:06
533km地点に1001本のバナナを運んだ時点で
533+1/3km地点まで一往復半して
533+1/3km地点に1000本のバナナを置くことが出来るので
そこから残り全部持っていけば466km進んだ時点で
残りの距離は2/3km、所持バナナ数534、あとの工程では
進む距離は1kmに満たないので534本運べる。

506 名前:132人目の素数さん :2005/11/03(木) 01:18:56
1からnまでの、n個の自然数をそれぞれ1回ずつ足したり引いたりして
0にすることを考える。
例えばn=4だったら 1−2−3+4=0 とすればよい。

問題:どうやっても0にできないのはどのようなnか?

507 名前:132人目の素数さん :2005/11/03(木) 01:34:26
>>506
Σ[k=1,n]kが奇数になるときだから4でわって1or2あまる数

508 名前:132人目の素数さん :2005/11/03(木) 02:04:06
>>505
なるほど。行動を1kmごとって縛りをぬけば1本のこらずはこべるのか。むしろこの問題は
>>492の問題文には1kmごとに行ったり着たりしないといけないとは一言もいってないでしょ?」
ってとこがミソなんだな。まあちょっと数学的な面白さといえるかどうか微妙だけど。
で後はこれがホントに最大ですかってとこだな。

509 名前:132人目の素数さん :2005/11/03(木) 10:12:30
まずはじめに、この問題の中で引き返す動きが得になりうるのは
引き返した先においてきたバナナがあって、
それを回収しに行く場合以外には考えられない。
よって、先に進むときは常にもてるだけもって行き、
引き返すときは引き返したい地点まで行って帰ってくるのに
餓死しない最低限だけ持っていくというのが最善である。
また、置いてきたバナナはその地点までの往復の距離よりはたくさんないと
取りに行く意味がないので、実際には引き返す際に携えるバナナは
片道の距離移動するのに餓死しないだけの本数となり、
引き返し始めるときの、「次にバナナを食べるまでに歩く距離」をx,
片道の距離をyとすると、[(1-x)+y]本あればいいことになる。ただし[ ]はガウス記号。

次にこの問題では、残バナナ数が2001本以上3000本以下の間と
1001本〜2000本の間ではそれぞれ、大まかに動いても小刻みに動いても損得は変わらない
大まかに動くというのは、例えばいきなり200km地点まで2.5往復するような場合。
1000本持っていく→600本置いて引き返す→
1000本持っていく→600本置いて引き返す→
1000本持っていく→200km地点に2000本置ける
小刻みに動くというのは、例えば200m先まで2.5往復を1000回繰り返すような場合。
1000本持っていく→1000本置いて引き返す→
1000本持っていく→1000本置いて引き返す→
1000本持っていく→200m地点に2999本置ける

よって、残り工夫する余地があるのはワンセットの運搬でバナナの残り本数が
2001以上から2000以下の間をまたぐかどうかだが、これは少し考えれば
1.5往復で済ませられる所を無駄に2.5往復するだけになるので損。
したがってこれ以上効率よく運ぶことはできない。

510 名前:132人目の素数さん :2005/11/03(木) 11:28:11
>2001以上から2000以下の間をまたぐかどうかだが
残った問題は、こういう数字を誰が想像出来るかってことだけだな。

511 名前:132人目の素数さん :2005/11/03(木) 20:36:25
ん? バナナの問題って、解決不能じゃなかったっけ?(「ビューティフルマインド」に書かれてた)

512 名前:132人目の素数さん :2005/11/03(木) 22:36:48
解決不能ってことはないでしょう。あたえられた問題に対しては
考えうる組合せは有限なんだから、最悪全通り調べれば答えはわかるはず。

513 名前:132人目の素数さん :2005/11/04(金) 06:37:06
まとめwikiを作ってみました。
(といってもまだほとんど内容はありませんが。)
http://www6.atwiki.jp/omoshiro2ch/

とりあえず、現行スレの内容を登録していっているのですが、
なかなか面倒なので、ぼちぼちとやっていこうと思っています。

wiki は誰でも編集できるので、
レイアウト、カテゴリなどの改善は、
ご自由に行ってもらってかいまいませんが、
その際は本スレにご一報ください。

過去ログ登録作業のお手伝いも、もちろん大歓迎です。

514 名前:132人目の素数さん :2005/11/04(金) 07:14:44
>>513
他にWikiでまとめているのは、数学の本スレ http://www3.atwiki.jp/math/ だけだっけ?

515 名前:132人目の素数さん :2005/11/07(月) 07:55:29
あげ
>バナナの問題
結局、解決不能なの?
動きを離散に制限すれば解けると思うけど、
任意の点で折り返すことができるなら、解けないってことなのかな?

516 名前:132人目の素数さん :2005/11/07(月) 17:18:35
バナナの消費も連続ってことなら解けるんじゃないの?

517 名前:御前等死ねよ :2005/11/07(月) 17:19:33
御前等死ねよこんな所でこそこそ話し合ってるんじゃえぞ
御前等みたいなどうしようもない無い┃┃¨ ╋┓には
一般的な人も対応に困ってるんだよ。
だから社会に迷惑をかけない人生遅れよ┃┃¨┿┓!
御前等も少しは社会に適応できる努力デモしたらどうだ?

518 名前:132人目の素数さん :2005/11/07(月) 20:01:44
やっぱオレって天才「┃┃¨ ╋┓」ってのが「バカ」のことだと見抜いちゃった♥。サッスガ―。

519 名前:132人目の素数さん :2005/11/08(火) 12:10:07
70p四方の正方形内に50個の小さい点が打たれているとき、2点間の距離が15p以内の2点が必ず存在することを示せ。
かつて秋山仁さんがテレビで紹介していた数学オリンピックの問題です。

520 名前:132人目の素数さん :2005/11/08(火) 12:21:22
数オリ?予選問題かな。

7x7のグリッドに区切って鳩の巣原理でしょ。

521 名前:132人目の素数さん :2005/11/08(火) 12:32:47
二次方程式ax^2+bx+c=0のa→0に於ける解について述べよ

522 名前:132人目の素数さん :2005/11/08(火) 12:41:50
n を自然数とする。
以下の性質を満たす多項式 f(n) の個数はいくつか。
* f(2) = n
* 各項の係数は {0,1,2,3} のいずれか
* 各項の次数は 0 以上の整数

523 名前:132人目の素数さん :2005/11/08(火) 12:46:16
* 各項の次数は 0 以上の整数

あ?

524 名前:132人目の素数さん :2005/11/08(火) 12:57:55
>あ?
じゃ何が聞きたいのかわからん。田舎のヤンキーかお前は。
x^{-2} のような項は無い、という意味だけど、何か問題あった?

525 名前:132人目の素数さん :2005/11/08(火) 12:58:12
>>522
1/((1-t^2)(1-t))のn次の係数。
(f(n)はf(x)のまちがい?)


526 名前:132人目の素数さん :2005/11/08(火) 13:00:41
n次の係数-> coeffidient for t
It is interesting problem !


527 名前:132人目の素数さん :2005/11/08(火) 13:01:42
>>524
それ普通多項式って言わない。

528 名前:132人目の素数さん :2005/11/08(火) 13:03:47
多項式に負の次数が含む流儀があんのかね。まず見ないね。

529 名前:132人目の素数さん :2005/11/08(火) 13:15:56
要するに、

蚤_i 2^i   a_i∈{0,1,2,3}
i=0
が n になるような a_i の取り方は何通り?ってこと。
わかりにくい書き方してごめん。

530 名前:132人目の素数さん :2005/11/08(火) 13:19:04
>>529
ごめん、また揚げ足とっていい?

多項式っていうと、無限級数じゃないんだからΣ[i=0,∞]は変かと。。。







ごめん、たいした問題じゃなかった。

531 名前:132人目の素数さん :2005/11/08(火) 13:24:51
いや、もう多項式のことは忘れて、529を正としてください。
狽フ上限はlognでもいいですが、ただの組合せの問題なので細かいことは。。。

すれ違いの質問なんだけど、
x^{1/2} も多項式ではない?
x^a と書いたとき、多項式、といえば暗黙のうちに a は自然数?

532 名前:132人目の素数さん :2005/11/08(火) 13:35:53
>>531
言葉の問題だから、大して重要ではないと思うけど
俺は違うと思う。
x^a って書いたとき、aは非負整数になると思う。



追伸、問題はいい問題だよね。考えてみるわ

533 名前:531 :2005/11/08(火) 14:07:38
なるほど。サンクス。
俺はアルゴリズム屋で、計算量を語るときには n^{2.5} であろうが
全部「多項式」でひっくるめてしまうから、感覚がずれてたんだな。
オブジェクトとしての多項式を指すときとは区別しないといけないのか。
勉強になった。

534 名前:132人目の素数さん :2005/11/08(火) 14:31:19
526です。522は正しくは

1/((1-t^2)(1-t))をテーラー展開したときのtのn乗の係数。

でした。

535 名前:GiantLeaves ◆6fN.Sojv5w :2005/11/08(火) 19:03:18
x,yの整式f(x,y)に、x=a,y=bを代入して0になるときもf(x,y)は(x-a)(y-b)で割り切れるとは限らないことを示せ。

536 名前:132人目の素数さん :2005/11/08(火) 19:10:04
f(x,y) = x+y-a-b

537 名前:132人目の素数さん :2005/11/08(火) 20:10:11
x,yの整式f(x,y)に、x=aを代入して0になるときもf(x,y)は(x-a)で割り切れるとは限らないことを示せ。

538 名前:GiantLeaves ◆6fN.Sojv5w :2005/11/08(火) 20:16:16
talk:>>537 x→(x-a)+aの変換で、(x-a)で割ったあまりを考える。するとそれは0しかない。

539 名前:132人目の素数さん :2005/11/08(火) 20:19:56
一年前の今くらいの数学板にKingOfMathematicianてのがいたけど,
キミの芸風はそっくりだよ.彼は崩れたらしいが,キミは精進してくれ.

540 名前:132人目の素数さん :2005/11/08(火) 20:46:30
2^.5がnormal numberであることの証明

541 名前:132人目の素数さん :2005/11/08(火) 22:18:42
バナナのかわりにガソリンみたいな連続量でも答えは同じ?
ガソリン3000リットルで

542 名前:132人目の素数さん :2005/11/08(火) 22:22:22
>>533
計算量を語るときでも多項式ではない

543 名前:132人目の素数さん :2005/11/08(火) 23:05:23
2=蚤n10^-nの無限級数に展開してみて?

544 名前:VIPPER :2005/11/09(水) 03:01:04
VIPからきますた、数学の天才、ちょっときてくれ(`・ω・´)

開成中の入試過去問題にお手上げ状態┐(´ー`)┌

【秀才】 この問題の解き方教えてくれ 【集まれ】
http://news19.2ch.net/test/read.cgi/news/1131301609/

http://146.progoo.com/rental/img_bbs2/img_data/146_2965_46f7b0e076.jpg


545 名前:132人目の素数さん :2005/11/09(水) 07:15:16
>>522
[n/2]+1 ([]はガウス記号)


546 名前:132人目の素数さん :2005/11/09(水) 07:59:13
正解です

>534
その係数を具体的に求めれば >545 になると思います。

547 名前:132人目の素数さん :2005/11/09(水) 08:23:50
>>543
たとえば0<= an <=9 とかいう条件がないと解はたくさん
あるけど、おもしろくない。右の条件で
考えると10進表示と対応するから、

2=1.9999...=2.00000...

問題の意図はみたいな表示を考えているのか?
これでもあまりおもしろくないが。

548 名前:132人目の素数さん :2005/11/09(水) 17:39:21
現在開成中学合格のために勉強している小学6年生ですが、下の問題が解けません。誰か教えて下さい!
↓↓↓↓

87を2進法で表せ。

549 名前:132人目の素数さん :2005/11/09(水) 17:41:48
>>548
申し訳ありませんが当スレでは宿題は取り扱っておりませんので、
質問スレへお回りください。

◆ わからない問題はここに書いてね 178 ◆
http://science4.2ch.net/test/read.cgi/math/1130634000/

550 名前:132人目の素数さん :2005/11/13(日) 23:52:43
f(x)=x^x^x^…
について、

(1)f(√2)を求めよ。
(2)f(1/2)を求めよ。





y=f(x)のグラフとか考えてみると、チョット面白いと思います。

551 名前:132人目の素数さん :2005/11/14(月) 00:01:58
>550
(1)発散
(2)0

だろ普通に

552 名前:132人目の素数さん :2005/11/14(月) 00:03:23
違うだろ

553 名前:132人目の素数さん :2005/11/14(月) 00:06:17
ある商品を買うと,6種類の内ランダムに1種類がおまけとしてついてきます

@6種類全て揃えるには平均いくつの商品を買えば良いか
A6種類全てを2個ずつ揃えるには平均いくつの商品を買えば良いか
B6種類全てをn個ずつ揃えるには平均いくつの商品を買えば良いか

554 名前:132人目の素数さん :2005/11/14(月) 00:13:45
>550
(1)2、(2)1/4だな

555 名前:132人目の素数さん :2005/11/14(月) 12:03:51
>>550
f(√2)=2でOK.
f(1/4)=1/2だが、f(1/2)=?
ちなみにmapleをもっているひとのためにグラフの描画。
plot([t^(1/t),t,t=1/4..2]);
mapleもっていないひとで希望があればupしますが。


556 名前:132人目の素数さん :2005/11/14(月) 13:55:07
>>555
是非みたいです。

557 名前:132人目の素数さん :2005/11/14(月) 15:17:56
>>555
>>550
のグラフをupしました。

http://www.aa-iiwa.net/gazou/mori/joyful04/img/736.jpg


558 名前:132人目の素数さん :2005/11/14(月) 15:30:56
>>557
激ジョブ!

559 名前:132人目の素数さん :2005/11/14(月) 15:33:06
>>555
>>550
なんかdeleteされたみたい。再アップ

http://wibo.m78.com/clip/img/86764.jpg

560 名前:132人目の素数さん :2005/11/14(月) 15:35:24
>>555
>>550

スレよごしげめん。直リンできないみたいだ。

http://wibo.m78.com/clip/clip.cgi?

561 名前:132人目の素数さん :2005/11/14(月) 15:50:14
>>560
専用ブラウザで見てるから、直リンできてる。問題ない。

562 名前:132人目の素数さん :2005/11/14(月) 23:46:25
なんか凄いところに貼ってるねw

563 名前:132人目の素数さん :2005/11/14(月) 23:54:39
釣られてるんだろよ

564 名前:とおりすがり :2005/11/15(火) 10:36:50
>>550
質問です.面白い問題だと思うけど,例えば,
3^3^3 はどう解釈するのかな?(3^3)^3 それとも 3^(3^3)
後者だと無限のとこからはじめないといけないことになるような・・・
う〜んこれは計算の順序は決まってる?僕にはわからないのだが・・・

565 名前:132人目の素数さん :2005/11/15(火) 16:17:07
>>550
これはa_1=x, a_n=x^a_(n-1)と定義したときの
lim_{n →∞}a_n
と解釈したが。


566 名前:132人目の素数さん :2005/11/15(火) 17:14:54
…は普通は極限として解釈するだろ
3^3^3は普通は3^(3^3)

567 名前:132人目の素数さん :2005/11/15(火) 19:25:29
無向グラフGが与えられたときにGの最小閉路をもとめる
多項式時間アルゴリズムをしめせ。(TSPじゃないよ)


568 名前:132人目の素数さん :2005/11/15(火) 21:23:39
xlogf(x)=f(x)>>550?

569 名前:132人目の素数さん :2005/11/15(火) 22:07:27
>568
そうですね。
これよりf(x)の最大値が実数の範囲でeとなることも分かって、なんか不思議。
ちなみに>550を微分したらど-なるんだ!?

570 名前:132人目の素数さん :2005/11/15(火) 22:15:47
>550の出題者なんですが、f(x)は微分可能なはずです。逆関数(y=x^(1/x))が微分可能なので…

571 名前:132人目の素数さん :2005/11/15(火) 22:30:11
>>550
有名問題だよな。
俺は吸うせみで見た。

572 名前:132人目の素数さん :2005/11/16(水) 00:25:59
演算Эは実数に対して次で定義される。

実数a、bに対して
aЭbはa+b、aーb、ab、a/b、a^bのいずれかを表す。

このとき、aЭbЭc(a、b、c≠0)の値の期待値を求めよ。

573 名前:132人目の素数さん :2005/11/16(水) 00:35:45
aЭbЭcは
(aЭb)Эc の意味?

574 名前:132人目の素数さん :2005/11/16(水) 00:46:22
>>573
同じ演算子が続けは、普通は左から計算するのが普通だが、
3^3^3のように、3^(3^3) の意味で使っている輩がいるんだよね。
だから紛らわしい…

575 名前:132人目の素数さん :2005/11/16(水) 05:03:58
でたらめな問題だな

576 名前:132人目の素数さん :2005/11/16(水) 05:14:15
>>574
> 同じ演算子が続けは、普通は左から計算するのが普通だが

そんなことはない。プログラム言語では2項演算子は左から右
に結合させるか、右から左か、すべて決まっている。
べき乗に関していえば、
BASIC: a^b^c = (a^b)^c
FORTRAN: a**b**c = a**(b**c)
だ。ほかに右から左に結合させる演算子の代表的なものとして、
C言語における代入演算子 a = b = cなどがある。

577 名前:576 :2005/11/16(水) 05:40:40
で、 >>572 の答えだが、pЭq の期待値をε(p,q)と書けば、
ε(p,q) = (1/5)((p+q)+(p-q)+pq+p/q+p^q) = (1/5)(p(2+q+1/q)+p^q).
演算子の結合を >>573のように解釈すれば、aЭbЭc の期待値は、
ε(ε(a,b),c) でいいのかな?

578 名前:132人目の素数さん :2005/11/16(水) 12:30:12
13131=(131)31=(1x1)31=131=1+1=2
131=1x1=1
131=1+1=2
1=2


579 名前:132人目の素数さん :2005/11/16(水) 13:00:13
a+b/c=(a+b)/c


580 名前:132人目の素数さん :2005/11/16(水) 13:17:23
>>579

           ,>: : :i: : ̄`: : : : : : : : \
        /: : : /\: :ト 、 、ヽ:.:.:.. : :、ヽ
.       ヽニィ: : |′   jノ  ヽ!:|:.:.:.:.:.:.l:.ヾ_‐ァ
         /: :Ν       ,. - リハ:.:.N:.:.:ニ=-
         |; : l_   ,_..イtラ.フ   |//⌒!:\  
      -彳 : l、モト:  ``     l/ l.)/:.:.リ  
         l: :ハ:l  '}          jl ' ノ:.:.l/  だれだ、そいつは?
       V `l  丶-     _, ´ 厂:/:/    引き取ってもらえ!
              、  、=ァ''_´/  /イ:.:.|〈
            \ `ニ-   / ルW≧
             ヽj|lj;, ィ       l
              |   

581 名前:132人目の素数さん :2005/11/16(水) 14:28:57
a#b$c

(a#b)$c
a+b+c,a+b-c,a-b+c,a-b-c,a*b+c,a*b-c,a*b*c,a*b/c
,a/b+c,a/b-c,a/b*c,a/b/c,a^b+c,a^b-c,a^b*c,a^b/c

a#(b$c)
a+b*c,a+b/c,a+b^c,a-b*c,a-b/c,a-b^c,a*b^c,a/b^c,a^b^c


582 名前:132人目の素数さん :2005/11/16(水) 15:34:43
地道に場合わけして解くべきだ!w

583 名前:132人目の素数さん :2005/11/16(水) 15:52:04
三角形ABCに対して、『三角形ABCの情報』を次で定義する:△ABCの辺の長さ(a、b、c)、内角(A、B、C)、面積Sのいずれかの値

また、機械Mは一回の起動につき『三角形ABCの情報』を無作為に一つ選び、それを示す。

三角形が与えられていて、その形状を機械Mによって決定したい。形状が決定できるための機械Mの起動回数の期待値を求めよ。

584 名前:132人目の素数さん :2005/11/17(木) 01:07:42
n個のしぼんだ風船が一列に並んでいます。
各風船の容量は 1/i (i=1,2,,,,n)で互いに異なり、外見からは容量がわかりません。
どのような順番で並んでいるかもわかりません。
容量を超えて空気を入れると風船は割れてしまいます。

このn個の風船に対し、できるだけたくさんの空気をいれることを考えます。
(風船が割れてしまった場合はカウントされません)
どのような戦略をとれば沢山の空気をいれることができるでしょうか。

期待値ができるだけ大きくなるような、
確率的な戦略を考えてみてください。

585 名前:584 :2005/11/17(木) 01:39:08
補足です。

風船に一定量の空気を入れてみて、割れなかったらさらに後から足す、
ということは可能とします。

586 名前:132人目の素数さん :2005/11/17(木) 12:06:40
商品を買うと,a種類の内b種類がランダムおまけとしてついてきます(1回でb種類が同じおまけになる事はない)
おまけ全てをc個ずつ集めるには,平均いくつの商品を買えば良いか?

587 名前:132人目の素数さん :2005/11/17(木) 12:20:09
>586
いまいち問題の内容がつかめない。

おまけは全部で a 種類あって、その中から b 個がランダムに選ばれる。
選ばれた b 個の中には同じ種類のおまけが二つ以上含まれることはない。

ってことかいな。

588 名前:132人目の素数さん :2005/11/17(木) 13:05:16
>>584
期待値の下限が、
1/(1^2) + 1/(2^2) + 1/(3^2) 1/(4^2) + …… + 1/(n^2)
になるところまでは分かった。

589 名前:132人目の素数さん :2005/11/17(木) 13:07:05
↑あ、ごめん。+記号一個抜けてたw
まあ、分かるよね。

590 名前:587 :2005/11/17(木) 13:24:32
うーん。 b = c = 1 のときは有名問題で aloga だけど、
b = 2, c = 1 のときですでに式がめちゃくちゃ汚くなってきた。

これ一般の a,b,c できれいに解けんのか?

591 名前:587 :2005/11/17(木) 13:31:24
q_k を
* q_0=q_1=q_2=1,
* q_k = (k-1)q_{k-1} + n(n-1)q_{k-2} (k>=3)
を満たす数列として、

数列 p_k を
* p_0=1,p_1=0,p_2=1
* p_k = k(k-1)(n-2)(n-3)・・・(n-k+1)q_k/{n(n-1)}^{k-1} (k>=3)
#最初の二つだけが k になってるのは間違いではない
として定義する

で、期待値が
a
Σp_k
k=1

になる。

592 名前:132人目の素数さん :2005/11/17(木) 16:33:35
>>591
詳しくおながいしまつ

593 名前:132人目の素数さん :2005/11/17(木) 18:43:34
詳しくあげ

594 名前:132人目の素数さん :2005/11/18(金) 04:44:46
Sk を、おまけをk個持っている状態とする。
Pr{i,j} を状態 i から k へ遷移する確率とすると
* Pr{k,k} = k(k-1)/n(n-1)
* Pr{k,k+1} = k(n-k)/n(n-1)
* Pr{k,k+2} = (n-k)(n-k-1)/n(n-1)

商品を買い始めてから全ての
おまけを集めるまでに状態 k に到達する確率を Pr{k} とすると
* Pr{k} = Pr{k-1}Pr{k-1,k} + Pr{k-2}Pr{k-2,k}
* Pr{0} = 1
* Pr{1} = 0

Xk を状態 k での滞留時間とすると、
Sk にたどり着く確率が Pr{k} でその後 Sk->Sk のループを繰り返す
回数の期待値が 1/Pr{k,k} だから
* E[Xk] = Pr{k}/Pr{k,k}
これを計算すれば >587 の p_k になる。

でもとめるものは
* ΣE[Xk]

q_k は面倒なので求めなかった。
つーか初項がたぶん間違ってる。

595 名前:132人目の素数さん :2005/11/18(金) 05:14:04
>>590
> うーん。 b = c = 1 のときは有名問題で aloga だけど、

aloga って何?

596 名前:132人目の素数さん :2005/11/18(金) 05:16:57
a*log(a)


597 名前:587 :2005/11/18(金) 05:25:32
"coupon collector" でググってみると良い。
どうも、c=1の場合でも研究の対象になってるみたいだな。
c=1 の場合にすら一般式を出すのは seems impossible だそうだ。

598 名前:132人目の素数さん :2005/11/18(金) 05:43:38
>>586
1回で
がなければ問題がはっきりするけどそのへん明らかにして再掲されては?

599 名前:132人目の素数さん :2005/11/18(金) 06:06:03
>>598が再掲しろと言ってるぞ!

600 名前:584 :2005/11/18(金) 07:27:56
>588
風船をランダムに並べ替えて、順に1の空気を入れていって
1入れることに成功すれば残りのものに1/2を入れていって
成功すれば1/3....
ただし、容量 1/k の風船が過去に割れていることがわかっていれば
(風船が割れれば容量がわかる)1/kを飛ばして1/(k+1)を試す

というやり方ですよね。
もう少し工夫すると、Σ1/k!まであげることが知られています。
私の思いついたのはΣ1/k!よりほんのちょっぴり大きくなるものですが、
よりよい戦略はまだあると思います。

601 名前:132人目の素数さん :2005/11/18(金) 08:20:34
n個のふうせんがある
まず全部に 1/n の空気を入れる
次に、一つを横にのけておいて、残りの(n-1)個に 1/(n-1) の空気を入れる
成功すれば、その(n-1)個ではじめにもどる
失敗すれば、のけておいたふうせんと割れなかった(n-2)個のふうせんの
あわせて(n-1)個ではじめにもどる

これでも同じ結果になるな

602 名前:132人目の素数さん :2005/11/18(金) 12:21:59
割れたら駄目という意味だと思ってた。


603 名前:132人目の素数さん :2005/11/18(金) 22:58:15
>>586再括
@商品を買うと,6種類の内1種類がランダムおまけとしてついてきます
おまけ全てを2個ずつ集めるには,平均いくつの商品を買えば良いか?

A商品を買うと,6種類の内1種類がランダムおまけとしてついてきます
おまけ全てをa個ずつ集めるには,平均いくつの商品を買えば良いか?


難し過ぎるからこれにしとく
因みに1個ずつ集めるには1+(6/5)+(6/4)+(6/3)+(6/2)+(6/1)=14.7個

604 名前:132人目の素数さん :2005/11/18(金) 23:32:50
>>603
解ける?>>597の人によると一般には seems impossible という意見があるみたいだけど。
持ってる答えまちがいない?

605 名前:132人目の素数さん :2005/11/19(土) 01:24:50
y'=y(logy)'を解け。

別に面白くないか…

606 名前:132人目の素数さん :2005/11/19(土) 02:51:55
>>605
はいはい。
質問は質問スレに書け!

607 名前:とおりすがり :2005/11/19(土) 03:54:13
a^x=1/x
a:定数
って解ける?

608 名前:132人目の素数さん :2005/11/19(土) 04:24:22
>>607
>>606


609 名前:とおりすがり :2005/11/19(土) 05:30:46
>>608
面白い問題じゃなかった?
このスレって答えがわかってないとだめってことだっけ?

610 名前:132人目の素数さん :2005/11/19(土) 16:01:41
解けたら面白いけど
解けなかったら、ああやっぱりね、ってなるだけだよ

611 名前:GiantLeaves ◆6fN.Sojv5w :2005/11/19(土) 16:44:21
talk:>>605 それはもしかしてギャグのつもりなのか?

612 名前:132人目の素数さん :2005/11/19(土) 17:29:13
一辺が1cmの立方体を隙間なく積んで一つの直方体を作りました
この直方体は,一番外側にある立方体と,内側にある立方体の数が等しいです
直方体の縦横高さはそれぞれ何cmでしょうか?
ただし直方体の体積が最も小さいものを答えて下さい

613 名前:132人目の素数さん :2005/11/19(土) 20:51:36
>>612
総当りで解いただけだけど、8×10×12 ですね。

614 名前:132人目の素数さん :2005/11/19(土) 21:00:49
a^3-(a-2)^3=(a-2)^3

615 名前:132人目の素数さん :2005/11/19(土) 21:14:45
立方体?

616 名前:132人目の素数さん :2005/11/19(土) 21:43:36
abc-(a-2)(b-2)(c-2)=(a-2)(b-2)(c-2)
abc=2((abc)-2(ab+bc+ca)+4(a+b+c)-8)
abc-4(ab+bc+ca)+8(a+b+c)=16
a=b=c->a^3-12a^2+24a=16->a=2,4,8,16
a=b<>c->a^2c-4a(2a+c)+8(2a+c)=16

617 名前:132人目の素数さん :2005/11/20(日) 09:01:12
◯◯◯◯◯◯◯◯
◯◯◯◯◯◯◯◯
◯◯◯◯◯◯◯◯
◯◯◯◯◯◯◯◯
◯◯◯◯◯◯◯◯
◯◯◯◯◯◯◯◯
◯◯◯◯◯◯◯◯
◯◯◯◯◯◯◯◯


↑上の64個の◯に1〜16の整数を入れて,どこから4×4の正方形を取り出しても四方陣になっているようにして下さい
四方陣とは,4×4の正方形の各◯に1〜16の整数を1個ずつ入れて,縦横斜めの合計を全て同じにしたもの

618 名前:132人目の素数さん :2005/11/20(日) 10:22:41
4×4の正方形を取り出しても四方陣に足して組み立てるだけ

619 名前:132人目の素数さん :2005/11/20(日) 17:26:30
@
1回交換型のポ-カ-で,他の役は一切無視してひたすらロイヤルフラッシュを狙った場合,成功率はいくらか?
(23456のストレ-トフラッシュも5枚交換してロイヤルフラッシュを狙う)

A
麻雀で,天和と十三不塔の確率はそれぞれいくらか?
(近似値ではなく,正確な値を求めて下さい)

620 名前:132人目の素数さん :2005/11/20(日) 17:51:41
>>619
面白くありません

621 名前:132人目の素数さん :2005/11/20(日) 17:53:29
天和は_。
解けるレベルの問題を出題します。
親の配牌で大三元が完成している(三元牌が3種ともに少なくとも3枚ある)確率を求めよ。

622 名前:132人目の素数さん :2005/11/20(日) 18:08:32
次のような数列{a(n)}は無数に存在するか。

・a(n)は単調増加
・n→∞のときa(n)→∞
・任意のnに対してa(n+2)-a(n+1)<a(n+1)-a(n)
・a(n)の一般項はnの多項式、三角関数(tan(n)など)、指数関数(k^nなど)の組合せのみで表される

623 名前:132人目の素数さん :2005/11/20(日) 18:09:28
3.828156682x10^(-9)

624 名前:132人目の素数さん :2005/11/20(日) 18:11:39
>>622
a(n)=log(n+k)でいいんでね?無数に存在するかって・・・一個みつけたらその列の
第k項目以降からなる部分列は全部条件みたすんじゃね?

625 名前:132人目の素数さん :2005/11/20(日) 18:12:58
logはだめなのかな?

626 名前:132人目の素数さん :2005/11/20(日) 18:13:35
an=n-2^(-n)でどうじゃ?

627 名前:132人目の素数さん :2005/11/20(日) 18:15:46
>624さん
>622ですが、対数関数は用いずにということです。説明不足ですみません。

628 名前:132人目の素数さん :2005/11/20(日) 18:18:28
>626さん
それは条件3番目を満たしますか?多分nが大きくなってくると満たされないはずです。

629 名前:132人目の素数さん :2005/11/20(日) 18:22:07
a(n+2)-a(n+1)<a(n+1)-a(n)
⇔a(n+1)>(1/2)(a(n+2)+a(n))
はf(x)が下に凸の凸関数のときa(n)=-f(n)ならみたされるべ?
f(x)=-x+2^(-x)は凸関数。

630 名前:132人目の素数さん :2005/11/20(日) 18:31:25
>629さん
a1=1/2、a2=3/4、a3=7/8でa3-a2=1/4=a2-a1となり条件を満たしません。予想では存在しない気がします。

631 名前:132人目の素数さん :2005/11/20(日) 18:32:06
ぬるぽ


632 名前:132人目の素数さん :2005/11/20(日) 18:35:08
がっ

633 名前:132人目の素数さん :2005/11/20(日) 18:36:10
a1=1-(1/2)
a2=2-(1/4)
a3=3-(1/8)
a3-a2=1-(1/8)+(1/4)=1+(1/8)=9/8
a2-a1=1-(1/4)+(1/2)=1+(1/4)=5/4
で9/8<5/4だとおもうけど?凸不等式しらんの?

634 名前:132人目の素数さん :2005/11/20(日) 18:42:54
>633さん
あ、すみません。何故かnを1と勘違いしてました。全然面白い問題じゃないですね(´Д`)吊ってきます

635 名前:132人目の素数さん :2005/11/20(日) 22:53:04
なんだ?

636 名前:132人目の素数さん :2005/11/21(月) 01:15:05
まあいいんでないの?
反例を見つける問題は、少なくとも>>619みたいのよりは面白いと思うよ。

637 名前:132人目の素数さん :2005/11/21(月) 02:12:22
優しいね
なんだかんだでみんないい人
普通に思った

638 名前:132人目の素数さん :2005/11/21(月) 02:26:39
便乗して出題!

(1)log#n(x)=loglog…(logがn個)…logxとする。
log#n(x)を微分せよ。

(2)x^#n=x^x^…(xがn個)…^xとする。
x(n)を微分せよ。ただし、(1)の記号を用いてもよい。

639 名前:132人目の素数さん :2005/11/21(月) 07:03:39
>>638

(1)
log#n(x)
=log#n-1(x^1)
=-log#n-1(x)
∴log#n(x)=((-1)^(n-1))*n^-1

かな?(2)は分からん

640 名前:132人目の素数さん :2005/11/21(月) 07:04:15
ここの人なら分かりそうなので教えてください。
今まで付き合った人数を当てると言われて見事に当てられた
んですが、まず自分が何歳まで生きると思うかを相手に言って
その数字に付き合った人数をたして、何かをかけて、その数字の
十の位と一の位を足して、また何かをかけて、、
みたいな感じだったんですけど、自分も使ってみたいんで
正しい聞きだし方を誰か教えてください。


641 名前:132人目の素数さん :2005/11/21(月) 10:19:01
・自分が何歳まで生きると思うかを教えて下さい。
・その数字に付き合った人数をたして心の中で思って下さい。
・その数字に9をかけてください。
・その数字の十の位と一の位を足してください。
 百の位や千の位があればそれも全部足してください。
・それに0をかけてください。
・その数字に付き合った人数をたしてください。
・いくつになりましたか?

642 名前:132人目の素数さん :2005/11/21(月) 12:45:02
0になりますた

643 名前:132人目の素数さん :2005/11/21(月) 12:50:30
マジレスすれば>>641の下から三行目が不要

644 名前:132人目の素数さん :2005/11/21(月) 12:51:27
あー不要というよりは、そこで3くらいをかけさせておいたほうがいいのか

645 名前:132人目の素数さん :2005/11/22(火) 23:03:25
どの2つの頂点間の距離も整数になっている直方体は存在するか?
存在するならどんな直方体か?

646 名前:132人目の素数さん :2005/11/22(火) 23:12:57
>>645
それまだ未解決だと思う。(最近の発展は知らんけど)

647 名前:132人目の素数さん :2005/11/22(火) 23:38:23
>645
うん、未解決。半年前くらいの数セミにもあった。

648 名前:132人目の素数さん :2005/11/23(水) 00:09:09
実数a、b(a、b≠0)に対して、次のようなm、nが存在するコトを示せ。

・a=mb+n
・n∈Z
・0≦m<a

649 名前:132人目の素数さん :2005/11/23(水) 00:21:15
a=0.9,b=1/10000,
とおくと、0≦bm<0.01 よりn≦0,mb<am<0.1
だから存在しないとおもうが。


650 名前:132人目の素数さん :2005/11/23(水) 00:34:09
>648
間違った(´Д`)

実数a、b(a、b≠0)に対して、次のようなm、nが存在するコトを示せ。

・a=mb+n
・m∈Z
・0<n≦a

651 名前:132人目の素数さん :2005/11/23(水) 00:42:51
(;^ω^)

652 名前:132人目の素数さん :2005/11/23(水) 00:48:38
・a=mb+n
・m∈Z
・0≦n<b

のことじゃなくて?
まあ≦の=をどっちにつけるかはどっちでもいいけど

653 名前:132人目の素数さん :2005/11/23(水) 00:55:39
>>650
aが正でなくては3行目の不等式は苦しいが?


654 名前:132人目の素数さん :2005/11/23(水) 02:02:23
>>603 (1)
あと、m 種類を1個ずつ、n 種類を2個ずつ集めればコンプという状態から、
コンプまでにかかる平均回数を a[m,n] とすると

a[0,0] = 0,
a[m,n] = (m*a[m-1,n] + n*a[m+1,n-1] + 6) / (m+n)

が成立して、計算すると
a[0,6] = 390968681/16200000 = 24.133…

平均 24個強買えば良いと

655 名前:132人目の素数さん :2005/11/23(水) 02:18:51
分母は m+n じゃなくて 6 じゃない?

656 名前:132人目の素数さん :2005/11/23(水) 02:21:03
あぁ、ごめん。勘違い。

657 名前:132人目の素数さん :2005/11/23(水) 14:21:49
↓の問題お願いします。
http://www10.plala.or.jp/mathcontest/2005r.htm

658 名前:132人目の素数さん :2005/11/23(水) 14:26:04
0と1からなるword(つまり有限の数列)A=(a1,...,ak),
B=(b1,...,bl)に対してその結合ABをAB=(a1,...,ak,b1,...,bl)
と定義する。さらにwordの無限列B(1)=A1,B(2)=A1A2,B(3)=A1A2A3,...
に対してU_i B(i)を無限列 A1A2A3....として定義する。

wordの列 S(0),S(1),...,T(0),T(1),...
を帰納的に次のように定義する。
(1) S(1)=0, T(0)=1,
(2)S(i+1)=S(i)S(i)T(i), T(i+1)=S(i)T(i)T(i).
このとき S(∞)=U_i S(i)とおく。

S(∞)=001 001 011 001 001 011 001 011 011
001 001 011 001 001 011 001 011 011
001 001 011 001 011 011 001 011 011

このときS(∞)の中にはCをwordとして、...CCC..という
パターンは決して現れないと思われるがどうだろうか?
(いかにも正しそうだが証明できるのだろうか?)

659 名前:132人目の素数さん :2005/11/23(水) 17:33:41
658です。
訂正
(1) S(1)=0, T(0)=1,
でなく、
(1) S(0)=0, T(0)=1,
でした。

660 名前:132人目の素数さん :2005/11/23(水) 18:45:25
それどっかで見たことあるな。
結果は肯定的だったと思うけど証明は思い出せない。

661 名前:132人目の素数さん :2005/11/23(水) 20:13:43
>>658
S(∞) の最初の文字を 0文字目とする。

非負整数 n を3進法で表して、下の桁から見ていったときに、
2 よりも 0 が先に現れるなら、S(∞) の n文字目は 0、
0 よりも 2 が先に現れるなら、S(∞) の n文字目は 1。

よって、k を非負整数として、
n ≡ 0*3^k + (3^k-1)/2 (mod 3^(k+1)) のとき、S(∞) の n文字目は 0、
n ≡ 2*3^k + (3^k-1)/2 (mod 3^(k+1)) のとき、S(∞) の n文字目は 1。

m を非負整数として、m ≠ 0 (mod 3^k), m ≡ 0 (mod 3^(k-1)) とする。
n ≡ (3^k-1)/2 (mod 3^k) なら、
S(∞) 中の n, (n+m), (n+2m) 文字目、の3文字のうちひとつは 0 で、ひとつは 1。
よって、S(∞) に m 文字から成る語の3回以上の繰り返しは存在しない。

662 名前:132人目の素数さん :2005/11/23(水) 21:30:00
それで証明できるけどkが一つ違う。


663 名前:?犬笠銀次郎 :2005/11/23(水) 21:40:17
碇けいいちは女体と布団圧縮袋が位相同型であることを、実験によって示した。

http://ginjiro.blogspot.com

664 名前:132人目の素数さん :2005/11/24(木) 00:38:40
>>661
658です。なんかエレガントそうな証明ですね。もう一度
論理を追ってみます。

665 名前:132人目の素数さん :2005/11/24(木) 00:43:46
問題文:

整数係数多項式f(x)に対し、『綺麗な多項式』を次の様に定義します。

各項の係数が1であるような多項式

また、綺麗な多項式でない多項式f(x)に対して、t=x+a(aは有理数)なる置換により得られる多項式f(t)が綺麗な多項式であるとき、f(x)をtを用いて『綺麗な多項式に出来る』と呼ぶことにします。

例えば、f(x)=x^2+3xは綺麗な多項式でないが、t=x-3/2なる置換により綺麗な多項式にできます。

666 名前:132人目の素数さん :2005/11/24(木) 00:45:25
問1:
綺麗な多項式に出来ない多項式が無数に存在することを示してください。


問2:
与えられた綺麗な多項式でない多項式が、綺麗な多項式に出来るかどうかを判定するアルゴリズムを示してください。

667 名前:132人目の素数さん :2005/11/24(木) 00:50:00
>>665
意味不明。


668 名前:132人目の素数さん :2005/11/24(木) 00:58:46
>666
言いたいコトは分かるが解けないorz

669 名前:132人目の素数さん :2005/11/24(木) 00:59:50
1+x+x^2+x^3+x^4+...=1/(1-x)

670 名前:132人目の素数さん :2005/11/24(木) 00:59:53
>>661
658です。たしかにmの条件でkがひとつずれていますが。
あと111...1タイプ は 0 になりますね。
しかしこれは美しい証明です!ありがとうございます!

671 名前:132人目の素数さん :2005/11/24(木) 01:13:15
.....0000011111

672 名前:132人目の素数さん :2005/11/24(木) 04:10:10
>666
ちょっと簡単すぎるなぁ。
n次の多項式として、
n次の係数が1でないといけないのと、
f(x+a)のn-1時が0か1になるようなaはそれぞれ一通りだから
簡単にチェックできるし、判例もみつかる。

673 名前:132人目の素数さん :2005/11/24(木) 13:30:31
上から2次しか合わせられないから、明らかだね

674 名前:132人目の素数さん :2005/11/24(木) 17:57:31
平面上にいくつか点を置く
どの3つの点も一直線上になく,どの2つの点の距離も整数である
平面上にはいくつまで点を置けるか?
また,その置き方は?
さらに,立体まで拡張したらどうか?

675 名前:132人目の素数さん :2005/11/24(木) 18:04:55
>>674
与えられたnに対して
題意を満たすようにn点を平面状に配置することは可能。

676 名前:132人目の素数さん :2005/11/24(木) 18:09:04
頂角A=20度の二等辺三角形ABCがある。
AB、AC上に∠DBC=50度、∠ECB=60度の点D、Eをそれぞれとる。
この時∠DEC=何度?

中学生レベルで解けるけど、結構難しい…と思う。
多分答え知らずに10分で導き出せたらかなり凄いわ。
俺は答え知ってても10分くらいかかる。

677 名前:132人目の素数さん :2005/11/24(木) 18:13:41
>>677
ラングレーの問題

678 名前:132人目の素数さん :2005/11/24(木) 18:23:58
>>676
この位→>

679 名前:132人目の素数さん :2005/11/24(木) 18:33:01
>>676
もしかして
AC、AB上に それぞれ〜
じゃね?

680 名前:132人目の素数さん :2005/11/24(木) 18:44:31
>>675
証明おながいしまつ

681 名前:676 :2005/11/24(木) 19:37:27
>>679
ごめん。それだわ。

682 名前:675 :2005/11/24(木) 22:06:08
>>674
「どの2点間の距離も有理数である」として略証をしめす。

(補題1)
任意の整数nに対して
0<θ<(π/(2n))かつcosθ,sinθがともに有理数であるθが存在する。
(補題1の証明)
sinθ=2m/(m^2+1) cosθ=(m^2-1)/(m^2+1) (mは整数)とすると
mを十分大きくすれば (x>0のとき sinx<xを用いて)
補題の成立が言える。
(補題2)
cosθ,sinθがともに有理数のとき任意の整数nについて
sin(nθ)は有理数である。
(補題2の証明)
加法定理より自明。

(本題の証明)
補題1で構成したθを用いて
n個の点Pi(cos(2iθ),sin(2iθ))(i=1,2,3,..,n)を単位円上に配置する。
ここでPiとPj(i<j)の距離は 2sin((j-i)θ)で補題2よりこれは有理数。



683 名前:132人目の素数さん :2005/11/25(金) 02:07:59
これが無限個の点にすると成り立たなくなるから不思議だよなぁ。

684 名前:132人目の素数さん :2005/11/25(金) 22:01:48
>>683
無限個は存在しない証明おながいできますか?

685 名前:132人目の素数さん :2005/11/25(金) 22:17:40
>>684
おながいと言うのをやめたら書いてやってもいい!

686 名前:132人目の素数さん :2005/11/25(金) 22:21:15
>>685
おねがいするぴょん

687 名前:132人目の素数さん :2005/11/25(金) 22:22:17
>684
大数の2005年3月号、宿題読め ぜんぶ同じ

688 名前:132人目の素数さん :2005/11/25(金) 22:28:03
>>686
心が篭ってないから、書きかけたけどやめた

689 名前:132人目の素数さん :2005/11/25(金) 22:31:05
>>688
えぇ――――――!!!!!!!!
心をこめておねがいします。

690 名前:132人目の素数さん :2005/11/25(金) 22:36:46
>>687
大数の2005年3月号、宿題の答えも3月号にのってるすか?大数なんか
ヨンだことないからわかんね。大学の図書館にも当然ないし。とりよせんのもなー。
どっか立ち読みできる本屋ないすか?関西圏で。

691 名前:132人目の素数さん :2005/11/25(金) 22:45:07
>690
どこの本屋でも立ち読みできるよ。たまに立ち読みすると楽しい

692 名前:132人目の素数さん :2005/11/25(金) 23:18:02
>>691
まじっすか?でも面独裁なー。探すの。紀伊国屋にはなさそうだしジュンクまでいけば
あるかもしれないけど遠いし。>>683の証明の方針だけでもだれかおしえてもらえません?
ここまでいわれたら標準的な数学科の学部生ならできるやろぐらいの方針。

693 名前:132人目の素数さん :2005/11/25(金) 23:40:10
>標準的な数学科の学部生
「平均的な」なら悲惨なものだけどなw

694 名前:132人目の素数さん :2005/11/26(土) 00:35:27
         │\
         │  \  (∀゚ )
         │   \ // )
         │     」」¬
         |
         |
         |
         |
         |
         |

695 名前:132人目の素数さん :2005/11/26(土) 00:36:55
l、m、nをl+m+n=1を満たす正実数とする。
また、実数a、bに対して演算a?bを次で定義する。:a?bは確率lでa+b、確率mでa*b、確率nでa/bの値を返す

k個0でない実数a_1、a_2、…、a_kについて(a_1)?(a_2)?…?(a_k)の値の期待値を?(k)と書くとき、lim[k→∞]?(k)は収束するか。

696 名前:132人目の素数さん :2005/11/26(土) 00:37:30
         \   ∩─ー、    ====
           \/ ● 、_ `ヽ   ======
           / \( ●  ● |つ
           |   X_入__ノ   ミ   そんな餌で俺様が釣られクマ―!
            、 (_/   ノ /⌒l
            /\___ノ゙_/  /  =====
            〈         __ノ  ====
            \ \_    \
             \___)     \   ======   (´⌒
                \   ___ \__  (´⌒;;(´⌒;;
                  \___)___)(´;;⌒  (´⌒;;  ズザザザ
                               (´⌒; (´⌒;;

697 名前:132人目の素数さん :2005/11/26(土) 18:01:39
N(a,b)={a+nb|n∈Z}とおく。
(1)B={N(a,b)|a∈Z,b∈N}⊂P(Z)とおくと、BはZのある位相の開基となるための条件を満たしていることを示せ。
(2)θを、Bを開基とする位相とする。θ−開集合は空集合or無限集合であることを示せ。
(3)N(a,b)∈Bは明らかにθ−開集合であるが、同時にθ−閉集合でもあることを示せ。
(4)Z−{-1,1}=∪[p:素数]N(0,p) を示せ。
(5)素数が無限個あることを示せ。

698 名前:132人目の素数さん :2005/11/26(土) 18:04:59
書き方がちょっとマズイかな。(2)訂正。
(2)θを、Bを開基とする位相とする。O∈θならば、Oは空集合or無限集合であることを示せ。


699 名前:132人目の素数さん :2005/11/26(土) 18:09:54
一辺が100の正方形の中からランダムに3つ点を取る
その3点を頂点とする三角形の面積の平均はいくらか?

700 名前:132人目の素数さん :2005/11/26(土) 18:36:58
多分100e/3とかじゃね?勘だけど。

701 名前:132人目の素数さん :2005/11/26(土) 18:37:46
次のうち、他のものと異なるものはどれか。

東京、富山、京都、岡山、青森、山口

702 名前:132人目の素数さん :2005/11/26(土) 18:42:52
東京だけが首都

703 名前:701 :2005/11/26(土) 18:52:06
さっきの問題、指定が少なすぎると思う香具師いるかもだけど、我慢して解いて…

後レスですまソ。

704 名前:132人目の素数さん :2005/11/26(土) 19:19:34
□があるとか?
ない岡山

705 名前:132人目の素数さん :2005/11/26(土) 20:35:50
>>701
京都。
縦書きして裏から見ると分かる。


706 名前:GiantLeaves ◆6fN.Sojv5w :2005/11/26(土) 21:20:25
talk:>>701 山口だけ、ほとんどの小学二年生が書ける。

707 名前:132人目の素数さん :2005/11/26(土) 21:24:18
>>701
東京 首都

708 名前:132人目の素数さん :2005/11/26(土) 21:26:17
>>701
青森
色がある

709 名前:132人目の素数さん :2005/11/26(土) 21:28:59
>>701
山口
総画数が10画未満

710 名前:132人目の素数さん :2005/11/26(土) 21:31:24
>>701
青森
や行(や・ゆ・よ)を使わない

711 名前:132人目の素数さん :2005/11/26(土) 21:31:40
θっていうのはスクリプトOじゃないのか?
いやθで悪いっていうわけじゃないんだけど

712 名前:132人目の素数さん :2005/11/26(土) 21:36:06
>>701
山口
右下に点がついていない。

713 名前:132人目の素数さん :2005/11/26(土) 21:38:00
やまくち。


714 名前:132人目の素数さん :2005/11/26(土) 21:44:31
富山
新幹線の通る気配が無い

715 名前:132人目の素数さん :2005/11/26(土) 21:51:56
京都だけ府
また、両方の漢字が同じ訓を持つ

716 名前:132人目の素数さん :2005/11/26(土) 21:56:58
東京
海に接していない。(湾には接しているが、飽くまで湾である)

717 名前:132人目の素数さん :2005/11/26(土) 22:00:33
>>716
離島は東京にあらず!  っていうと怒られますわよ。

718 名前:132人目の素数さん :2005/11/26(土) 22:05:15
そうか・・・orz
失念ォィォィ

719 名前:132人目の素数さん :2005/11/26(土) 22:09:21
>>705が合ってるような気がする

720 名前:132人目の素数さん :2005/11/26(土) 22:13:35
東京。
東京市 というのはない。

721 名前:132人目の素数さん :2005/11/26(土) 22:35:00
>>720
第三…

722 名前:701 :2005/11/26(土) 23:42:15
たくさんのレス、dクス。これって答え言ったほうがいいんかな…?
答えが意外につまらんくて、おまいらに文句言われるのが怖いんだが…orz

723 名前:132人目の素数さん :2005/11/26(土) 23:47:38
それでも、答えをキボン

724 名前:132人目の素数さん :2005/11/27(日) 01:22:10
次のような関数f(x,y)は存在するか。

・任意の点でf(x,y)>1
・任意の点で連続
・x>yのときy+p>xなる任意の正数pに対して、f(x,y+p)=e^f(x,y)
・x<yのときx+q<yなる任意の正数qに対して、f(x+q,y)=logf(x,y)
・x=yのとき任意の正数rに対して、f(x+r,y-r)=f(x,y)

725 名前:707=708=709=710=712=714=716=718 :2005/11/27(日) 01:36:26
>>701-723
>>701が鬱にならないことを祈る。

>>707>>708>>709>>710>>712>>714>>716>>718
上の8件は全部俺のレス

東京島民の方々気を悪くしていたらごめんなさい。

726 名前:132人目の素数さん :2005/11/27(日) 01:41:48
京都以外左右対称ってことだろww

727 名前:132人目の素数さん :2005/11/27(日) 01:42:39
まぁ、それでいいんだけどな、たぶんな。

728 名前:132人目の素数さん :2005/11/27(日) 01:56:24
>>721
いやいや

729 名前:132人目の素数さん :2005/11/27(日) 15:28:28
>>724
>・任意の点で連続
>・x>yのときy+p>xなる任意の正数pに対して、f(x,y+p)=e^f(x,y)
 
この2つですでに矛盾してるやん。3番目からz>x>yのときf(x,z)=e^f(x,y)
xを固定してa=f(x,x+1)、b=f(x,x-1)とおくとa=e^bよりa≠b。
f(x,x)=lim[z→x+0]f(x,z)=a、f(x,x)=lim[y→x-0]f(x,y)=b。

730 名前:132人目の素数さん :2005/11/28(月) 00:26:29
>676

http://science4.2ch.net/test/read.cgi/math/1131511061/20-40


731 名前:132人目の素数さん :2005/11/28(月) 03:17:44
>>699
正方形の一辺を 1 として計算する。

三点の位置ベクトルを P,Q,R、三角形の面積を S とすれば、 
∫SdPdQdR を計算すればいいが、次のような変数 r,θ,h,t,h',t' で計算する。
正方形の中心を O として、r = |PR↑|、
ある一辺から反時計回りに計った PR↑ の角度を θ(0≦θ<2π)、
O から直線 PR に下ろした脚を H として、
OH の長さを h (PR↑から反時計回りに π/2 回転した方向を正とする)、
HP の長さを t (PR↑の方向を正とする)、
Q を通り、直線 PR に平行な直線に O から下ろした脚を H' として、
同様に OH' の長さを h', H'Q の長さを t' とする。

dPdQdR = rdtdt'drdh'dhdθ, S = (1/2)r|h-h'| で、面積の平均値は
E(S) = (1/2) ∫ r^2 |h-h'| dtdt'drdh'dhdθ。

パラメータ (θ,h) の直線から正方形が切り取る線分の長さを L(θ,h) とする。

E(S) の被積分関数は t,t' によらず、∫dt = L(θ,h)-r, ∫dt' = L(θ,h') から、
E(S) = (1/2) ∫ r^2 |h-h'| L(θ,h') {L(θ,h)-r} drdh'dhdθ

問題の対称性から、積分領域を 0≦θ≦π/4, h≧0 に制限する(そのかわり積分値を 8*2 倍する)。
0≦r≦L(θ,h), -(c+s)/2≦h'≦(c+s)/2, 0≦h≦(c+s)/2, 0≦θ≦π/4
(c = cos(θ), s = sin(θ) と置いた。以下もこれに従う)
が積分領域になり、0≦θ≦π/4 の範囲で L(θ,h) は次のようになる、

|h|≦(c-s)/2 のとき: L(θ,h) = 1/c
(c-s)/2<|h|≦(c+s)/2 のとき: L(θ,h) = (c+s-2|h|)/(2cs)

732 名前:132人目の素数さん :2005/11/28(月) 03:18:27
(続き)
r,h',h,θ の順に積分すると、

E(S) = 8 ∫r^2 |h-h'| L(θ,h') {L(θ,h)-r} drdh'dhdθ
= (2/3) ∫ |h-h'| L(θ,h') L(θ,h)^4 dh'dhdθ
= (1/18) ∫[h = 0, (c-s)/2] L(θ,h)^4 (12h^2 + 3c^2 + s^2)/c dhdθ
 + (1/36) ∫[h = (c-s)/2, (c+s)/2] L(θ,h)^4 {(c+s-2h)^3 + 24csh}/(cs) dhdθ
= (1/180) ∫[θ = 0, π/4] (20c^3 - 18c^2s + 12cs^2 - 5s^3)/c^5 dθ
= 11/144 = 0.0763888…

一辺 100 ならこれの 100^2 倍

733 名前:132人目の素数さん :2005/11/28(月) 19:03:03
3×3×3の立方体を1×1×1の立方体27個に分割する時,重ねて切るのを禁止したら26回切る必要がある(1回切ると1つずつ断片増えるから)
では重ねて切ってもよいとすると,何回切れば良い事になるか?

734 名前:132人目の素数さん :2005/11/28(月) 19:20:08
今凄いアイディアが浮かんだ
6回だけで切れる

735 名前:132人目の素数さん :2005/11/28(月) 19:51:30
じゃあ5回で切れる

736 名前:132人目の素数さん :2005/11/28(月) 20:05:41
まじで?

737 名前:132人目の素数さん :2005/11/28(月) 20:09:00
│  │
│  │
│  │
└┰┘
  ┃
  ┃


738 名前:GiantLeaves ◆6fN.Sojv5w :2005/11/28(月) 20:31:15
talk:>>735 曲げるのか?

739 名前:132人目の素数さん :2005/11/28(月) 20:31:40
27-18-9-5-3-2-1
6回か
もっと少なく切れそうな気もするけどね

740 名前:132人目の素数さん :2005/11/28(月) 20:33:56
中央の立方体が6面あるから5回だけで切れるはず

741 名前:132人目の素数さん :2005/11/28(月) 20:35:13
>>740
kwsk

742 名前:132人目の素数さん :2005/11/28(月) 20:37:16
4×4×4も6回だな
5×5×5は9回になるのか?

743 名前:132人目の素数さん :2005/11/28(月) 20:44:57
半径1の円の中に斜辺1の直角二等辺三角形は何個入るか。
(但し、互いに重ならないように入れること)

744 名前:132人目の素数さん :2005/11/28(月) 20:50:02
4個じゃだめ。。。?

745 名前:132人目の素数さん :2005/11/28(月) 20:54:15
すくなくとも6個ははいるんじゃね?一辺1の正3角形ですら6個はいるんだから。

746 名前:132人目の素数さん :2005/11/28(月) 20:56:32
>>744
どう考えても8個はいけそうだが

747 名前:132人目の素数さん :2005/11/28(月) 21:00:36
ほんとだ。9つは無理?

748 名前:132人目の素数さん :2005/11/28(月) 21:09:53
>747
多分8個が最大だね。証明は難しい気がするけど。

749 名前:132人目の素数さん :2005/11/28(月) 21:10:29
π*1*1 ≒ 3.141592……
(1/(2*√2*√2)) * 8 = 2

ってか・・・
まだ、入る余地はあるんだな。

750 名前:132人目の素数さん :2005/11/28(月) 21:10:43
>>746
お前頭良いな

751 名前:132人目の素数さん :2005/11/28(月) 21:12:14
面積的にはまだまだいけそうだけどな。しかしできなくても不思議じゃない気もする。
この手のpacking problemは最大性の証明の部分がむずい。綺麗にスパっときもちよく
とけるのかな?

752 名前:132人目の素数さん :2005/11/28(月) 21:16:32
>749
やっぱり入れる三角形の形状があるからね。単に面積だけで考えられないのが難しいなぁ。

753 名前:132人目の素数さん :2005/11/28(月) 21:20:27
円の対象性から円の中心と三角形の頂点が一致しなきゃだめっしょ、多分。だったら8個が最大。それの証明は必要性がむずいね

754 名前:132人目の素数さん :2005/11/28(月) 21:26:27
円の対象性から六個入る正三角形より少しでも大きい正三角形は一個も入らない

755 名前:132人目の素数さん :2005/11/28(月) 21:27:34
八個つめた段階で、もうこれ以上ずらしようが無いからな・・・
八個の詰め方を考えないと、九個はどうあがいても無理っぽい。

意外と八個の詰め方が回転や、対称移動などを無視して一通りに定まる
っていうところから、証明スタートじゃね?

756 名前:132人目の素数さん :2005/11/28(月) 21:27:51
>>754
どういうこと?

757 名前:132人目の素数さん :2005/11/28(月) 21:29:25
8個も入らない…誰か教えてくれ。

758 名前:132人目の素数さん :2005/11/28(月) 21:32:50
>757
ヒント:風車っぽい形

759 名前:132人目の素数さん :2005/11/28(月) 21:40:58
>>733
2回

760 名前:757 :2005/11/28(月) 21:42:01
>>758
分かりました。べりーさんくすです。

でも、9個は無理だな。('A`)

761 名前:132人目の素数さん :2005/11/28(月) 21:43:25
このような見方はどうかな?

問題文を言い換えて
斜辺1の直角二等辺三角形が何らかの配置で平面上に密に敷き詰められている。この平面に単位円をおくとき、円内に最大いくつの三角形が含まれるか。
って考えると、平面を三角形で敷き詰める方法は(1)一辺1の正方形にして敷き詰める、(2)対辺1の正方形にして敷き詰める、(3)風車型にして敷き詰めるの三通り。
円の対象性からやはり中心と頂点は一致しなきければならない。
最大値は計算すると(1)では4個、(2)では8個、(3)でも8個だから、結局元の問いの最大値も8でいいんじゃない?

762 名前:132人目の素数さん :2005/11/28(月) 21:47:19
>>761
それはあかんやろ。

763 名前:132人目の素数さん :2005/11/28(月) 22:27:12
あかんな

764 名前:132人目の素数さん :2005/11/28(月) 22:31:17
( ´・∀・`)

765 名前:132人目の素数さん :2005/11/28(月) 22:33:30
9個入りそうだ

766 名前:132人目の素数さん :2005/11/28(月) 22:36:18
厚紙で作ってあれこれ動かしてみたらアイデアが浮かぶ気はする

767 名前:132人目の素数さん :2005/11/28(月) 22:52:25
対照性考えたら入る三角形の個数は偶数個になる筈。

768 名前:132人目の素数さん :2005/11/28(月) 22:53:42
9個入りそうだ。
風車型の配置からカメラ絞りを開くように中心に空間を開ける。
三角形の一つを斜辺が円周側になるようにすると、中央にもう一つ三角形が入りそうだ。

769 名前:132人目の素数さん :2005/11/28(月) 22:58:48
>>767
小学校から漢字の勉強をやり直せ!

770 名前:132人目の素数さん :2005/11/28(月) 23:00:35
>768
シャッターみたいな移動って出来るん?風車型から動かすのは(全体として)不可能な気がするよ。

771 名前:132人目の素数さん :2005/11/28(月) 23:12:28
対称性ってなぁ・・・そのまんまじゃ利用できないだろ

772 名前:768 :2005/11/28(月) 23:24:09
>>770
円の中で移動できるかわからない。
しかし、カメラの絞りを開いたような配置は出来る。

773 名前:132人目の素数さん :2005/11/28(月) 23:24:35
対称性を考えてるんじゃなくって勝手に対称なものに限定してるだけ

774 名前:132人目の素数さん :2005/11/28(月) 23:27:34
重なっちゃ駄目なんだっけ?
ちょっと重なっても良いなら9枚入る入れ方見つけたけど、、

775 名前:132人目の素数さん :2005/11/28(月) 23:28:11
>773
確かに。でも行き着くとこはやっぱり対称な入れ方にならないかな?

776 名前:132人目の素数さん :2005/11/28(月) 23:32:23
こういう問題の場合はならないことが多いよ

777 名前:132人目の素数さん :2005/11/28(月) 23:37:00
うんと重なっても良いなら百枚入れる入れ方見つけたけど…。

778 名前:132人目の素数さん :2005/11/28(月) 23:41:29
うそこけ。重なってもイイ場合の最大値は6だ。

779 名前:132人目の素数さん :2005/11/28(月) 23:45:20
じゃあ俺は101枚いれる。

780 名前:132人目の素数さん :2005/11/28(月) 23:55:06
やっぱり8枚が答えだ!

781 名前:132人目の素数さん :2005/11/28(月) 23:56:46
>>780
詳しく!!

782 名前:132人目の素数さん :2005/11/29(火) 00:26:42
結果だけで申し訳ないが解答。
ttp://www.stetson.edu/~efriedma/tanincir/

どうも9個入るらしいな。

783 名前:132人目の素数さん :2005/11/29(火) 00:30:16
次を満たす複素数zをすべて決定せよ。

z^nが有理数になる自然数nが存在する。

784 名前:132人目の素数さん :2005/11/29(火) 00:31:08
すげ

785 名前:132人目の素数さん :2005/11/29(火) 00:32:50
>782
半径が違くない?

786 名前:132人目の素数さん :2005/11/29(火) 00:35:18
半径r = 1.402+内に9個入ってるって事でしょ
1.402<√2


787 名前:132人目の素数さん :2005/11/29(火) 00:36:47
>>785
直角を挟む辺の長さが1の直角二等辺三角形が
指定個数はいるための円の最小(の可能性がある)円の半径が
載っているので r < √2 なやつで最大となると9個かなぁと。

788 名前:132人目の素数さん :2005/11/29(火) 00:37:43
すげー超すげー
6とか7とか9とかすげーよww

789 名前:132人目の素数さん :2005/11/29(火) 00:38:20
よくこんなの見つけてきたなw

790 名前:132人目の素数さん :2005/11/29(火) 00:43:42
すばらしい。証明はどうやってんだろ?

791 名前:132人目の素数さん :2005/11/29(火) 00:44:46
>787
あ、そう書いてあるね。ごめん、勘違いしちゃったよ。英語勉強しよorz

とりあえず乙!ホントよく見つけたなw

792 名前:132人目の素数さん :2005/11/29(火) 00:46:34
>790
証明もかなり激しいんだろうなw 発見されたの最近ってのがまた驚き

793 名前:132人目の素数さん :2005/11/29(火) 00:49:47
>783
aを勝手な有理数として、z=a、ai、a±ai。証明はだるい。

794 名前:132人目の素数さん :2005/11/29(火) 00:53:24
z=√2は?

795 名前:132人目の素数さん :2005/11/29(火) 00:58:37
>>783
rを正の有理数、k,nを自然数としてr^(1/n)(cos(2πk/n)+isin(2πk/n))。

796 名前:795 :2005/11/29(火) 01:00:15
まちごうた。
rを正の有理数、k,nを自然数としてr^(1/n)(cos(πk/n)+isin(πk/n))。

797 名前:132人目の素数さん :2005/11/29(火) 01:19:46
証明?計算だけすれば良いんじゃないの?

798 名前:132人目の素数さん :2005/11/29(火) 01:28:14
k/nも有理数s とおいた方が統一性があっていいんじゃないの?

799 名前:132人目の素数さん :2005/11/29(火) 01:33:53
r^(1/n) も、有理数 t を使って r^t と書けるな

{ exp( r^t + sπi ) | r, s, t は有理数 }

でどうか。

800 名前:132人目の素数さん :2005/11/29(火) 11:38:47
>>792
今年4月にはじまった企画です。みんな参加しる。
ってことではないかと思われ。

この手のは和算なんかで沢山ありそう…

801 名前:132人目の素数さん :2005/11/29(火) 12:29:02
>>743 の問題はかなり昔にどこかで見たような気がする。
たぶん『頭の体操』だったと思う。

802 名前:132人目の素数さん :2005/11/29(火) 12:30:50
1634733 6458092538 4844313388 3865090859 8417836700 3309231218 1110852389 3331001045 0815121211 8167511579
x
1900871 2816648221 1312685157 3935413975 4718967899 6851549366 6638539088 0271038021 0449895719 1261465571
=
310 7418240490 0437213507 5003588856 7930037346 0228427275 4572016194 8823206440 5180815045 5634682967
1723286782 4379162728 3803341547 1073108501 9195485290 0733772482 2783525742 3864540146 9173660247 7652346609

803 名前:132人目の素数さん :2005/11/29(火) 20:03:54
1×1の正方形10個を,重ねずに一つの正方形の中に詰めるには,正方形の一辺はどこまで小さくできるか?
また,正方形11個ではどうか?

804 名前:132人目の素数さん :2005/11/29(火) 22:01:46
>>802素因数分解か?
それ以上の分解は無理か?

805 名前:132人目の素数さん :2005/11/29(火) 22:36:53
>>803
たしか10個のときは3+(1/2)√2
11個のときは5/2 +√2
じゃなかった?

806 名前:132人目の素数さん :2005/11/30(水) 03:24:24
packing problemは面白いね。問題の見た目はかなり穏やかだし答えの予想もある程度はつくんだけど、いざ証明となると難しい。

807 名前:132人目の素数さん :2005/11/30(水) 03:40:37
平面を正n角形で隙間なく敷き詰める。
それが可能なnをすべて決定せよ。

808 名前:132人目の素数さん :2005/11/30(水) 03:50:08
>807
n=3、4、6では成立。

809 名前:132人目の素数さん :2005/11/30(水) 07:37:45
>>804
RSA暗号の素因数だろこれ。

810 名前:132人目の素数さん :2005/11/30(水) 11:10:07
>>782 のページとか見るとなるほどねって思うんだけど、こういうのって、ちゃんと数学として体系化できてるの?
なんというか、たまたまうまく詰めれましたってのは数学ではないし、パズル的ではあっても数学としては…?

811 名前:132人目の素数さん :2005/11/30(水) 12:33:45
任意の△ABCがあります
∠Aの三等分線と∠Bの三等分線で,辺ABに近い線同士の交点をPとします
また,∠Bの三等分線と∠Cの三等分線で,辺BCに近い線同士の交点をQとします
また,∠Cの三等分線と∠Aの三等分線で,辺CAに近い線同士の交点をRとします
△PQRが正三角形になる事を証明して下さい

812 名前:132人目の素数さん :2005/11/30(水) 13:04:04
>>810
いや、「詰めれました」の方はたまたまでもいい
問題は「これ以上はムリス」の方

813 名前:132人目の素数さん :2005/11/30(水) 17:20:06
>>811
F.Morleyの定理だな。自前の証明は持ってない。

814 名前:132人目の素数さん :2005/11/30(水) 19:22:59
詰め込みの問題って明らかに正しそうでも証明はめちゃくちゃハードだからね。
高校化学で習う六法最密充填構造が本当に最密なのかどうかすら
10年位前にやっと証明できたとか言う話だよ。

ちなみに、この問題はケプラー予想って言って、400年前からずっと
未解決だったんだが、計算機によるしらみ潰しの証明で
一応の解決をみたというだけらしい。

815 名前:132人目の素数さん :2005/11/30(水) 21:17:25
そういえば、詰め込みとか被覆のスレって無いなあ。

>>782 の別ページに被覆も取り上げられてるんだけど、おもろいね。
ttp://www.stetson.edu/~efriedma/sqcovcir/
こことか。

9→10 が、ほとんど改善されてなくてワロス

816 名前:132人目の素数さん :2005/11/30(水) 23:17:41
↓の問題お願いします。
http://www10.plala.or.jp/mathcontest/2005r.htm

817 名前:132人目の素数さん :2005/11/30(水) 23:54:04
下の図形を,四等分(合同)して下さい
(×は,ずれない様に書いてるだけだから無視して下さい)

×◯◯◯◯◯◯◯
×◯◯◯◯◯◯◯
×◯◯◯◯◯◯◯
×◯◯◯◯◯◯◯
×◯◯◯◯◯◯×
×◯◯◯◯◯◯×
×◯◯◯◯◯◯×
×◯◯◯◯◯◯×
◯◯◯◯◯◯◯×
◯◯◯◯◯◯◯×
◯◯◯◯◯◯◯×
◯◯◯◯◯◯◯×

818 名前:272 :2005/12/01(木) 00:33:27
272の3つの式から連立方程式以外を使ってabcの値を出す方法が
分かる人はいますか??


819 名前:132人目の素数さん :2005/12/01(木) 00:39:46
あ?

820 名前:132人目の素数さん :2005/12/01(木) 00:44:07
誤爆かな

821 名前:132人目の素数さん :2005/12/01(木) 20:56:47
>>817
グリッドに沿って分けることは無理かな。
梁の部分が6列あるし。

822 名前:132人目の素数さん :2005/12/01(木) 20:58:58
下の連分数はある無理数の値を表しています
それは何でしょうか?


      1
1+──────────
       1
  1+────────
       1
    1+──────
        1
      1+────
         1+…

823 名前:132人目の素数さん :2005/12/01(木) 21:03:54
{sqrt(5)+1}/2に決まってるだろ、はげ。

824 名前:132人目の素数さん :2005/12/01(木) 21:04:09
x=1+1/xの正の解?

825 名前:132人目の素数さん :2005/12/01(木) 21:05:35
答え書こうかと思ったら、書いてるうちに二人に先越されてタ。
連分数は結構知られてるから練らないといい問題にならないね。

826 名前:132人目の素数さん :2005/12/01(木) 22:29:06
>>822
出題者です.答えはメ-ル欄

827 名前:132人目の素数さん :2005/12/01(木) 22:37:18
そっすか(^^;

828 名前:132人目の素数さん :2005/12/01(木) 22:43:36
>>826
良問ですね

829 名前:132人目の素数さん :2005/12/01(木) 22:43:59
えー。

このタイプの連分数は、平方根と加減乗除でかけるって
アーベルかがロアかラグランジュか誰か知らんけど
証明済みだったと思うなぁ。

830 名前:132人目の素数さん :2005/12/01(木) 22:52:07
プゲラ

831 名前:132人目の素数さん :2005/12/01(木) 23:11:11
マジレスすると解法をしりたい

832 名前:132人目の素数さん :2005/12/01(木) 23:25:54
>>831

        1
1+────────
     ┌───┐
     │      │
     │  .A.  │
     │      │
     └───┘

とおけば

A = 1 + 1/A

833 名前:132人目の素数さん :2005/12/01(木) 23:29:35
>>832
>>826
なんで π/2 になるんだよ! ヽ(`Д´)ノ ガロァ!

834 名前:132人目の素数さん :2005/12/01(木) 23:50:03
連分数について詳しいお勧めの本を紹介してたも。

835 名前:132人目の素数さん :2005/12/02(金) 00:03:53
>>834
高木貞治、初等整数論講義、共立

836 名前:132人目の素数さん :2005/12/02(金) 18:29:29
このスレの纏めサイト作っていい?

837 名前:132人目の素数さん :2005/12/02(金) 19:25:39
ぜひおながいします

838 名前:132人目の素数さん :2005/12/02(金) 19:39:08
>>836
さっさと作れ!

839 名前:132人目の素数さん :2005/12/02(金) 19:40:06
>>838はツンデレ。

840 名前:132人目の素数さん :2005/12/02(金) 20:00:06
ツンデレってなんだべ?ツンドラの過去形?

841 名前:132人目の素数さん :2005/12/02(金) 20:06:11
>>840
グラフで言うと指数と対数みたいな関係

842 名前:836 :2005/12/02(金) 20:08:32
俺には無理だという事がわかった

843 名前:132人目の素数さん :2005/12/02(金) 20:09:16
>>840
君も数ヲタなら聞き分けたまえ!
コレで勉強しろ!
http://science4.2ch.net/test/read.cgi/math/1133325852/7-8

844 名前:132人目の素数さん :2005/12/02(金) 20:11:16
>>842
あきらめたら そこで試合終了だよ

  とりあえず、できるところまでやってごらん
   
.       ∧__∧
      ( ´・ω・)∧∧l||l
       /⌒ ,つ⌒ヽ )
       (___  (  __)  
"''"" "'゙''` '゙ ゙゚' ''' '' ''' ゚`

845 名前:836 :2005/12/02(金) 20:28:37
いや。。。。その、試験が近いから・・

846 名前:132人目の素数さん :2005/12/02(金) 20:30:28
試験勉強なんかしてる場合か!?あぁ?どっちが大切か考えてみたか?あぁ?

847 名前:132人目の素数さん :2005/12/02(金) 20:36:14
>>843
こんな用語があったのか。知らんかった。ありがとぉ。これでまたいっぽ数オタに近づいた。

848 名前:132人目の素数さん :2005/12/02(金) 20:39:47
>>845
    三≧=j  ///} } _⌒ヽ_j    `ヽ、
    ー=イ /////}/ノ´         `ヽ、  \
   //ハ_} ´ }/〃 / /  ,      \  ヽ
         /j/ /{  { ハ  /{ { } /     Y´ }
        /´ ゙⌒ヽ、V斗、{ハ,乂{ j、  j   j 「面白い問題教えてーな」 のまとめをすることが
    \、__/        トハ{xメ`  ィ=x y }   /  / 数ヲタ力を高めることに繋がるんだよ
    \\          l八`~   {xメノj/  /!T´  一石二丁じゃないか!
      \`=、      _,ハl|{\0   `~/ /U|  
    '⌒ヽ{ヽ}   /´/ ハj / アこ´ニ=イ{⌒゙iU ささっ、はじめようか!
.     /⌒>く   〈_ノ^)/ ^ ー/xヘ. i|
      /  /    Y   / {   // ハUi

849 名前:132人目の素数さん :2005/12/02(金) 21:59:29
1つのサイコロを振って出た目の数の得点だけもらえるゲームがある。ただし、出た目が気に入らなければ3回まで振り直すことができる。このゲームの期待値は□である

850 名前:132人目の素数さん :2005/12/02(金) 22:04:11
出た目→気に入らないから振り直し→出た鼻→気に入らないから振り直し→出た口→おk

よってこのゲームの期待値は口である

851 名前:132人目の素数さん :2005/12/02(金) 23:18:32
>>849
有名な京大の過去問。
面白いけど、超既出。
無駄無駄無駄…

852 名前:132人目の素数さん :2005/12/03(土) 00:50:32
次の覆面算を解いて下さい

  ABCDE
×     F
━━━━━━━
  EDCBA

853 名前:132人目の素数さん :2005/12/03(土) 01:00:37
たぶん解なし

854 名前:132人目の素数さん :2005/12/03(土) 01:40:52
>>851
面白いね。簡単だけど

855 名前:132人目の素数さん :2005/12/03(土) 01:51:15
プレーヤがアホだと話にならなくなるけどな
ちゃんと最善をつくすならいいんだけど

856 名前:132人目の素数さん :2005/12/03(土) 01:58:08
           n                 
           | |                  
          〈 |〈〈 ヽ    
          〈⊃  }        
   ∩___∩  |   |       こいつとはいつか戦場でやりあう
   | ノ      ヽ !   !
  /  ●   ● |  /
  |    ( _●_)  ミ/ 
 彡、   |∪|  / 
/ __  ヽノ / 
(___)   /  


857 名前:132人目の素数さん :2005/12/03(土) 09:48:35
>>852
ABCDE が左右対称で、F=1 とか?
あ、覆面算は、異なる文字には異なる数字を入れなきゃいけないんだっけ?

858 名前:132人目の素数さん :2005/12/03(土) 14:02:32
ttp://cclemon.ath.cx/nijiura/src/1133586073319.jpg
MD5:D1DA8E13031CF59E98B3F5DFC6B5F9BB
詳細をお願いします

859 名前:132人目の素数さん :2005/12/03(土) 14:03:27
誤爆しました。ごめんなさい

860 名前:132人目の素数さん :2005/12/03(土) 14:06:12
酷いな

861 名前:132人目の素数さん :2005/12/03(土) 15:02:38
>>852
今更だが

  21978
×     4
−−−−−−
  87912


862 名前:132人目の素数さん :2005/12/03(土) 15:18:57
>>822


    1
──────────────
     1
1−────────────
      1
  1+──────────
       2
    1−────────
         2
      1+─────
        1−…

ならπ/2だけど。

863 名前:132人目の素数さん :2005/12/03(土) 17:28:02
822はアフォww

864 名前:132人目の素数さん :2005/12/03(土) 18:55:49
>>849
89/18

865 名前:132人目の素数さん :2005/12/03(土) 22:03:29
一辺の長さが1の、正四面体、正六面体、正八面体、正十二面体、正二十面体を
体積の大きい順番に並べると?

866 名前:132人目の素数さん :2005/12/03(土) 22:27:34
20 12 8 6 4

867 名前:132人目の素数さん :2005/12/04(日) 08:32:02
>>822
1じゃないのか

868 名前:132人目の素数さん :2005/12/04(日) 13:56:52
>>550
x^x^x^x^... なんだけど、値が収束するための x の範囲が
e^(-e) < x < e^(1/x) と計算できるんだけど、どうやるか分かる?
# ちなみに、x が大きいと無限大に発散するし、小さいと振動する。

昔自力で証明したことがあったんだけど、今やったら解けなかった(´・ω・`)
気になるので分かったら教えてください。

869 名前:132人目の素数さん :2005/12/04(日) 14:29:25
>>868
>>550
x^x^x^x^....ってa1=x、a[n+1]=x^a[n]でさだめられる数列の極限で桶?

870 名前:132人目の素数さん :2005/12/04(日) 14:31:26
>>869
OKだと思う。 大昔、数蝉のエレ界で見たことがあった。

871 名前:132人目の素数さん :2005/12/04(日) 15:25:00
>>868
e^(-e) < x < e^(1/e) のとき収束する証明はなんかできた。
x>e^(1/e)、0<x<e^(-e)のとき収束しない証明もなんとかできた。
x=e^(1/e)のときって収束する気がする。なんでだろ?
x=e^(-e)のときがまんどくせー

872 名前:871 :2005/12/04(日) 15:36:46
しまった。0<x<e^(-e)のときの証明まちがってるや。
0<x≦e^(-e)のとき収束しない証明が結構面倒っぽい。
二項づつまとめていけばいけるような気はするんだけど。

873 名前:868 :2005/12/04(日) 16:34:48
ごめん。= 入れ忘れてた。
e^(-e) <= x <= e^(1/e) で収束します。

上限の方は頑張ればできるけど、下限が難しいね……。

874 名前:GiantLeaves ◆6fN.Sojv5w :2005/12/04(日) 16:56:03
talk:>>826 意味が分からないのだが。

875 名前:132人目の素数さん :2005/12/04(日) 17:06:07
遅レス乙

876 名前:132人目の素数さん :2005/12/04(日) 18:28:19
@『変数係数高階方程式、定数変化法、線形空間論、級数法について一度に問える問題を作成せよ。
また、それの解答も記せ。』
A『y=f(x)=√(1-x^2)sin^(-1)xの満たす微分方程式を作り、それを用いてf(x)のテイラー展開を求めよ。
y=√(1-x^2),ysin^(-1)xのテイラー展開の積を計算し、二項係数に関する新たな公式を導け。』
この問題が面白いと思えてここに答えを書けるやつは天才!
ほかの板ではまったく相手いにされなくて、諦めろって言われた問題です。

877 名前:132人目の素数さん :2005/12/04(日) 18:42:23
>>876
>この問題が面白いと思えてここに答えを書けるやつは天才!
 
ここには天才はいないからスレちがい

878 名前:132人目の素数さん :2005/12/04(日) 18:49:56
この問題が面白いと思えてここに答えを書けるやつは天才!
ほかの板ではまったく相手いにされなくて、諦めろって言われた問題です。

この二行つければどんな問題でも書き込んで良いのか
良いこと聞いちゃった

879 名前:876 :2005/12/04(日) 22:50:29
すいません、自己解決しました

880 名前:132人目の素数さん :2005/12/04(日) 23:50:54
>>879
(2)の答えおながいします。

881 名前:132人目の素数さん :2005/12/05(月) 02:16:27
>>876
宿題は質問スレに書け!
それから、教えて欲しいときは 「おねがいします、ロリ数ヲタ様」 だろ?

882 名前:132人目の素数さん :2005/12/05(月) 13:47:38
>>876
最低限のルールも守れない奴は首吊って死ね

883 名前:132人目の素数さん :2005/12/05(月) 19:14:41
円に任意に弦を引く時,弦の長さが内接正三角形の一辺(半径の√3倍)より長くなる確率はいくらか?

884 名前:132人目の素数さん :2005/12/05(月) 20:25:07
なつかしいな。御大どうしてるかな。

885 名前:132人目の素数さん :2005/12/06(火) 01:42:59
>>883
「同様に確からしい」の定義の仕方で値が変わるんだよね、これ。大数の法則から、
長い針金でも用意して1000回くらい投げて測定すれば真の確率が得られそうな気が
するけど、この方法だって、針金の投げ方で何が「同様に確からしい」か変わるから
意味が無い。測定できた確率が0.5±微少量 だったとすると、確率が1/2になるような
「同様に確からしい」条件になる投げ方で針金を投げてることが分かるだけだし。

886 名前:132人目の素数さん :2005/12/06(火) 02:11:24
pを素数とする。An={2^(p^(n+1))−1}/{2^(p^n)−1} (n∈N)とおく。
(1)Anの素因数はpで割ると必ず1余ることを示せ。
(2)AnとAm(n≠m)は互いに素であることを示せ。
(3)pで割ると1余る素数が無数に存在することを示せ。

887 名前:132人目の素数さん :2005/12/06(火) 02:45:04
>>886
これは・・・おもしろそうだけど見るからにノーヒントはきつそう。
ヒントをおながいします。

888 名前:132人目の素数さん :2005/12/06(火) 02:54:10
>>887
だが断るッ!
ヒントなどという奴は、受験数学でも解いてろってこった.

889 名前:132人目の素数さん :2005/12/06(火) 02:58:07
>>888
これノーヒントじゃ数論専攻の学生でもとけないんじゃ・・・プロの数学者がみつけた
テクニックじゃないの?

890 名前:132人目の素数さん :2005/12/06(火) 03:04:49
>>889
わたしは ひんとのせいで さっさと とかれる じょうきょうに あきあきしていました
そこで ひんとを やめたのです

891 名前:889 :2005/12/06(火) 03:05:53
なに かんがえてんだ!

892 名前:132人目の素数さん :2005/12/06(火) 03:07:59
おおくの ものたちが もんだいを とけずに きえていきました。
ねっとすうがくしゃである ろりすうをたが ひっしに かんがえる すがたは
わたしさえも かんどうさせるものがありました。

893 名前:132人目の素数さん :2005/12/06(火) 03:08:51
つ[チェーンソー]

894 名前:889 :2005/12/06(火) 03:09:02
おまえのために もんだいを といてるんじゃねえ!
よくも おれたちを みんなを おもちゃにしてくれたな!

895 名前:132人目の素数さん :2005/12/06(火) 03:11:11
それが どうかしましたか?
すべては だれかが といた もんだいなのです。

>>893
かみに ケンカをうるとは…
どこまでも たのしい ひとたちだ!
どうしても とくつもりですね これも ろりすうをたのサガか……

よろしい もうしばらく ひんとなしで とくと かんがえたまえ!

896 名前:132人目の素数さん :2005/12/06(火) 03:15:00
フェルマーの定理より。


897 名前:132人目の素数さん :2005/12/06(火) 03:17:21
自分で考えた問題でもないのにそんなんで上から人よっていう根性に乾杯!

898 名前:132人目の素数さん :2005/12/06(火) 04:27:47
>>886
>>896のヒントででけた。
 
(1)qをAnの素因数とする。q^e|2^n-1となる最大のe≧1をとる。q-1がpの倍数でないと仮定し
矛盾を導く。フェルマーの小定理より2^(p^(n+1))≡2 (mod p)であるからq=pではありえない。
よってpはq^(e-1)(q-1)と互いに素である。2^(p^(n+1))+q^eZ=(2^(p^n)+q^eZ)^pはZ/q^eZにおいて
1+q^eZに等しいがpが群の位数と互いに素であるから2^(p^n)+q^eZもやはり1+q^eZに等しい。
とくに2^(p^n)-1もq^eで割りきれる。するとAn=(2^(p^(n+1()-1)/(2^(p^n)-1)はqの倍数でない。
これは矛盾である。
(2)Anの素因数qをとる。q^e|2^(p^(n+1))-1となる最大のe≧1をとる。m≧nのとき
q^x|2^(p^(m+1))-1をみたす最大のxはeに等しいことをしめす。それにはm=n+1のときのみ
しめせば十分である。このとき2^(p^(m+1))-1=(2^(p^(n+1))-1)(1+2^(p^(n+1))+2^(2・p^(n+1))+・・・+2^((p-1)・p^(n+1)))
である。整係数多項式P(x)で1+x+・・・+x^(p-1)=(x-1)P(x)+pとなるものをとれば
1+2^(p^(n+1))+2^(2・p^(n+1))+・・・+2^((p-1)・p^(n+1))=(2^(p^(n+1))-1)P(2^(p^(n+1)))+pであるから
1+2^(p^(n+1))+2^(2・p^(n+1))+・・・+2^((p-1)・p^(n+1))と2^(p^(n+1))の最大公約数はpの約数。とくに
前者はqの倍数となりえない。よって主張はしめされた。
このことからとくにm≧nのときにAmはqではわりきれない。以上から(2)の主張が成立する。
(3)(1),(2)より明らか。□
 
こんな方法で示せるのか。おもろいこと聞いた・・・寝よ。

899 名前:132人目の素数さん :2005/12/06(火) 05:33:15
>>897
元ネタ知ってて言ってるのかな?

900 名前:886 :2005/12/06(火) 05:50:30
なんか荒れてる(^^;
とある問題を解こうとしていたときに発見した問題です。一応、私の考えた答えを。

(1)Anの素因数qを任意にとる。An≡0 (mod q)より2^(p^(n+1))≡1 (mod q)…*である。ここで、mod qに
おける2の位数をrとおくと、フェルマーの小定理及び*よりr|(q−1),r|p^(n+1)を得る。もしr=1と
すると2≡1 (mod q)となって矛盾するので、r>1である。よってp|rである。よってp|(q−1)である。

(2)ある素数qでq|An,q|Am (n<m)を満たすものがあるとすると、まず2^(p^(n+1))≡1 (mod q)…**を得る。
**とn+1≦mより、2^(p^m)≡1 (mod q)も成り立つ。すると、0≡Am=Σ[i=0〜p−1]{2^(p^m)}^i≡Σ[i=0〜p−1]1
≡p (mod q) よってq|pが成り立ち、pは素数であるから自動的にp=qとなる。これを**に代入して2^(p^(n+1))≡1
(mod p)ところが、フェルマーの小定理より2^p≡2 (mod p)なので2^(p^(n+1))≡2 (mod p)よって1≡2 (mod p)矛盾。
(3) (1)(2)より成立。

901 名前:886 :2005/12/06(火) 06:14:31
…ということで、(計算の都合上)p=2として、Anの素因数で最小のもの(これも計算の都合上)をqnと
おけば、数列{qn}は各項が全て異なる素数列なので、このqnを計算機で求めさせれば 好きなだけ
新しい素数が手に入るじゃーん!と思ったら大間違いだったorz Anはフェルマー数Fn=2^(2^n)+1
なんかと同じようなオーダーで発散するので すぐに計算機パンクする…

902 名前:132人目の素数さん :2005/12/06(火) 06:25:00
p|(p−1)。


903 名前:132人目の素数さん :2005/12/06(火) 19:58:24



ttp://www10.plala.or.jp/mathcontest/2005r.htm

904 名前:132人目の素数さん :2005/12/06(火) 23:11:10
>>272わかりません…教えてください。

905 名前:132人目の素数さん :2005/12/06(火) 23:17:50
x^2 + y^2 => 1
y^2 + z^2 => 1
z^2 + x^2 => 1

の示す図形の体積を求めて。(円じゃないそうです。)

ちなみに自分は図形を考えるのに3時間もかかりますた

906 名前:132人目の素数さん :2005/12/06(火) 23:30:49
不等号逆な気がする。

907 名前:132人目の素数さん :2005/12/06(火) 23:34:54
なんか色々日本語とかおかしいぞ

明らかに円じゃないわな
球でもない

908 名前:132人目の素数さん :2005/12/06(火) 23:44:57
あるマラソン選手は40kmの距離をちょうど2時間で走る。
このとき、この2時間の間に、彼がちょうど1km走った3分間が存在することを示せ。
今年の信州大学の問題です。

909 名前:132人目の素数さん :2005/12/06(火) 23:51:26
そのまんま平均値の定理じゃね

910 名前:132人目の素数さん :2005/12/07(水) 00:22:21
>>905
宿題は質問スレに書け! クズめ!

911 名前:132人目の素数さん :2005/12/07(水) 00:27:47
なんか最近宿題スレと勘違いしてる奴が居るんだよなあ

912 名前:132人目の素数さん :2005/12/07(水) 02:04:26
>>908
この問題、数オリの練習問題集で解いたことがある。もしかしたら数値まで一緒だったかも…
f(t)=(スタートしてからt分間に走った距離) (km) (0≦t≦120)とおく。f(0)=0,f(120)=40である。
fは連続関数としてよい。g(x)=f(3x+3)−f(x)−1 (0≦x≦39)とすると、
g(0)+g(1)+g(2)+…+g(39)=f(120)−f(0)−40=0…*
よって、ある0≦a<b≦39が存在してg(a)g(b)≦0が成り立つ(*の両辺を2乗してΣg(i)g(j)=−(1/2)Σg(i)^2≦0と
なるので、全ての0≦a<b≦39に対してg(a)g(b)>0だと矛盾するから)。
gもまた連続だから、中間値の定理からa<c<bかつg(c)=0を満たすcが存在する。このcに対してf(3c+3)−f(c)=1
が成り立つので、確かに ちょうど1km走った3分間が存在する。

913 名前:132人目の素数さん :2005/12/07(水) 02:25:59
赤い本だよねw

914 名前:132人目の素数さん :2005/12/07(水) 02:27:04
まあ物理学的には量子マラソン選手だったら成立しないんだけどね
其の場合には走るという概念が不成立なのかな

915 名前:132人目の素数さん :2005/12/07(水) 02:32:15
・距離の単位がなぜかナノ光年とかマイクロ光秒

916 名前:132人目の素数さん :2005/12/07(水) 02:32:43
・田村でも金、谷でも金

917 名前:132人目の素数さん :2005/12/07(水) 02:34:55
ミスったw文脈から分かると思うけど、
【誤】g(x)=f(3x+3)−f(x)−1,f(3c+3)−f(c)=1
【正】g(x)=f(3x+3)−f(3x)−1,f(3c+3)−f(3c)=1

>>913
赤かったかな?一昨年に図書館で借りた本で、もう その本の題名も覚えてない。

918 名前:132人目の素数さん :2005/12/07(水) 13:20:32
□に0〜9までの整数を1つずつ入れて等式を成立させて下さい
ただし時間は1以上,分や秒は59以下です

@
□□分□□秒×□=□時間□□分□□秒

A
□分□□秒×□□=□時間□□分□□秒

919 名前:132人目の素数さん :2005/12/07(水) 18:05:49
>>910
お前その問題解けないだろw

920 名前:132人目の素数さん :2005/12/07(水) 20:18:32
>>905
今年の東大入試に似たような問題が出て瀧がする。

実はその解答も見てないけど、エレガントな解答がある気がしない。
ごりごり計算するだけならつまらない問題だよな。

921 名前:132人目の素数さん :2005/12/07(水) 21:11:59
どっかで平行に切って断面の面積を求めて、、しかない気がするね

ところで平面の場合には\int (1/2)r^2θdθで面積求まるけど
立体のときに似た公式ってあるんだっけ

922 名前:132人目の素数さん :2005/12/07(水) 21:30:41
結構しらないひと多いんだな・・・

923 名前:132人目の素数さん :2005/12/07(水) 21:32:49
一般的にはヤコビアンを使って重積分かな。

三次元の場合は極座標っていっても問題によって
円柱座標とか仰角とるのとかで使い分けたりするね。

924 名前:922 :2005/12/07(水) 21:32:58
me to.

925 名前:132人目の素数さん :2005/12/07(水) 21:38:00
>>920
東大(理系)の6番だね。
不等号の向きが一個反対になってるが、問題としては大差なさそう。
ttp://www.yozemi.ac.jp/nyushi/sokuho/sokuho05/tokyo/zenki/sugaku_ri/mon6.html

926 名前:132人目の素数さん :2005/12/07(水) 21:42:43
>>920は全部≦だったらネットではめちゃめちゃ有名な超既出問題で
2個ならカバリエリの原理の便利さを説明するのによく引き合いに出される超有名問題。
しらない人結構いるんだな。

927 名前:132人目の素数さん :2005/12/07(水) 21:52:02
てか東大こんなネットでは超既出の問題だしたのか。ネットにはまってる香具師には
すげー有利だったろうなw

928 名前:132人目の素数さん :2005/12/07(水) 22:05:39
>>927
所詮、2チャネラーは灯台に受験できるレベルに達していないが

929 名前:132人目の素数さん :2005/12/07(水) 22:28:45
ノイズの集まりだからな

930 名前:132人目の素数さん :2005/12/07(水) 23:41:51
写像f:R→Rは各点で連続で1階微分可能であり、導関数f’:R→Rは各点で不連続である。
この条件を満たすfを1つ求めよ。

…デキナイよぅ。誰か助けて。

931 名前:132人目の素数さん :2005/12/08(木) 00:22:40
>>924
too

932 名前:132人目の素数さん :2005/12/08(木) 02:25:46
問題というか むしろクイズかな。

各項が互いに異なる実数列{an}で、任意のi∈Nに対して、{an}の部分列でaiに収束するものが
取れるようなものは存在するか。

933 名前:132人目の素数さん :2005/12/08(木) 02:52:35
0 以上 1 以下の有理数のうちで
既約に書いたときの分母が k のものを
k が 1 から無限大まで全部並べればいいんじゃないか

{ 0, 1; 1/2; 1/3, 2/3; 1/4, 3/4; 1/5, 2/5, 3/5, 4/5; 1/6, 5/6; 1/7, 2/7, 3/7, 4/7, 5/7, 6/7; ... }

934 名前:132人目の素数さん :2005/12/08(木) 03:04:34
っていうか任意の区間の有理数全体を任意の順番で並べたもので良いような

935 名前:132人目の素数さん :2005/12/08(木) 04:42:38
>>926
> しらない人結構いるんだな。

ゆとり教育の時代ですからね。

936 名前:132人目の素数さん :2005/12/08(木) 05:00:17
昔は昔で、情報源が本しかなかったから
やっぱ知ってる人は少なかったと思われ

937 名前:132人目の素数さん :2005/12/08(木) 05:09:09
おーい、我が家のおりこうさんが意見たれやがったぞ

938 名前:132人目の素数さん :2005/12/08(木) 23:47:57
>>932を満たす数列{an}に対して、Rの通常の位相に関するA={an|n∈N}の閉方はどんな性質を持っているか。
…いや、思いついただけ。

939 名前:132人目の素数さん :2005/12/09(金) 08:48:19
x^2=y^3+17の自然数解の組をすべて。

940 名前:132人目の素数さん :2005/12/09(金) 08:56:51
とりあえず (5, 2)

941 名前:132人目の素数さん :2005/12/09(金) 19:38:38
幅が7000kmもある広大な砂漠の一端にガソリン給油所があります
1台のトラックがこの砂漠を横断したいのですが
ガソリンを満載(荷台に積んだガソリンも含む)しても4200kmしか走れません
勿論砂漠の途中にはガソリン給油所はありません
そこで,ドラム缶に詰めたガソリンを砂漠の所々に置いて(置く場所はどこでも可)
そこでガソリンを補給する事にしました
例えば,スタ-トから1000kmの地点にドラム缶を置くと
往復に2000kmかかるので2200km分のガソリンしか置けません
どこにどの様にガソリンを置くと,最も少ないガソリンで横断できるでしょうか?
また,その時に必要なガソリンの量は何km分でしょうか?


942 名前:132人目の素数さん :2005/12/09(金) 22:45:17
>>941
必要なガソリンは16520km分。
@560km地点に3往復し3080km分×3のガソリンを置いてから3920km分のガソリンを積み560km地点に行く。
A560km地点から1400km地点に2往復し2520km分のガソリンを置いてから1400km地点に行く。
B1400km地点から2800km地点に1往復し1400km分のガソリンを置いてから2800km地点に行く。
C2800km地点から7000kmまで一気に行く。

943 名前:132人目の素数さん :2005/12/09(金) 23:02:13
>>942
それがベストな解だと証明してみせろ!

944 名前:132人目の素数さん :2005/12/10(土) 00:32:12
ふん!
ベストでない解だと証明してみせろ。


945 名前:132人目の素数さん :2005/12/10(土) 06:44:27
>>942 訂正
Aの「2520km分のガソリン」
→「2520km分×2のガソリン」

946 名前:132人目の素数さん :2005/12/10(土) 10:45:19
幅aの道路から、 この道路に直交する幅bの道路へ
長さlの線分を移すとき、線分が角を通過できる最大の長さを求めよ。

947 名前:132人目の素数さん :2005/12/10(土) 10:56:46
こんなの作りましたよ
http://plaza.rakuten.co.jp/mnovice/
皆さんどんどん書き込んでくださいね^^
リク受け付けます^^

948 名前:132人目の素数さん :2005/12/10(土) 11:10:09
>>946
立てて通す!

949 名前:132人目の素数さん :2005/12/10(土) 12:25:40
AB=AD、∠DBC=30°、∠ACB=44°、∠ACD=30°
である四角形ABCDにおいて、∠BACを求めよ。

(ヒント)つ 外接円

950 名前:132人目の素数さん :2005/12/10(土) 13:46:07
数0.123456789101112131415…(規則は分かると思うので書かなくてもいいですよね?)は超越数か?

951 名前:132人目の素数さん :2005/12/10(土) 14:31:32
Yes sir!

952 名前:132人目の素数さん :2005/12/10(土) 20:05:46
>>949
答えを教えろ!

953 名前:132人目の素数さん :2005/12/11(日) 00:54:45
サイコロを5回投げるとき、小さい順に出る確率をエレガントに求めよ。

(いろんな解き方があるだろうから、ワクワクテカテカ…)

954 名前:132人目の素数さん :2005/12/11(日) 09:59:49
>>953
そのような目ので方は6通りある。
なので6/6^6=1/6^5

955 名前:132人目の素数さん :2005/12/11(日) 10:09:54
こんな問題エレガントもへったくれもないぞ

956 名前:132人目の素数さん :2005/12/11(日) 10:25:13
どうい

957 名前:953 :2005/12/11(日) 11:10:19
言っておくが、122334 のようなものを含むのだぞ!

958 名前:132人目の素数さん :2005/12/11(日) 11:17:35
そりは小さい順、ではなくて、非増加、だよん

959 名前:953 :2005/12/11(日) 11:26:17
うるさい、うるさい、うるさい!!小さい順っていったら小さい順なんだよ!!



960 名前:958 :2005/12/11(日) 11:30:36
とは書いてみたものの、「小さい順」なんて表現使ったこと無いから
正しいのか正しくないのかわからなくなってきた。
まぁ、誤解を招く表現なのは確かだ

問題の答えは、
5x6の格子上で(0,0)からスタートして、
右か上にだけ進んでよいというルールの下で x = 5 に到達する、場合の数だから、
5C0 + 6C1 + 7C2 + 8C3 + 9C4 + 10C5 + 11C6
かな。これを 6^5 で割ればよい。

関数 f:{1,..,N} → {1,...,M} のうち、
単調非現象なものの個数はいくつあるか?
と言う風に一般化できるけど、面白くないな。

961 名前:132人目の素数さん :2005/12/11(日) 12:34:39
もうちっと二十面体さいころとかにして
数え上げるのが難しい問題にしないと面白くないよ

962 名前:132人目の素数さん :2005/12/11(日) 12:58:47
6を20にしようが200にしようが、数え上げるのは難しくならんだろう。

963 名前:132人目の素数さん :2005/12/11(日) 13:01:49
正十二面体と正二十面体の体積をエレガントに求めるにはどうしたらいいか?

964 名前:132人目の素数さん :2005/12/11(日) 13:07:03
川村みゆき「多面体の折り紙」でも眺めてみる。

965 名前:132人目の素数さん :2005/12/11(日) 13:08:35
一松信「正多面体を解く」の参考文献にも挙げられている。

966 名前:132人目の素数さん :2005/12/11(日) 18:43:06
平面上に長さ1の曲線が一本ある。
点集合Sをこの曲線上の点から距離1以内にある点の集合だとするとき、
Sの最大値もとめてくれ

967 名前:132人目の素数さん :2005/12/11(日) 18:47:24
点集合の最大値ってどういうこと?

968 名前:132人目の素数さん :2005/12/11(日) 18:49:58
ごめん、Sの面積

969 名前:132人目の素数さん :2005/12/11(日) 19:07:15
>>966
曲線自体も変化するの?なら勘で長さ1の線分のとき。

970 名前:132人目の素数さん :2005/12/11(日) 19:07:33
5.14

971 名前:132人目の素数さん :2005/12/11(日) 19:08:21
>>969
うん、曲線が変化する・・・っていうか、曲線以外何か動かせそうなものあるかな?

972 名前:953 :2005/12/11(日) 20:05:30
>>960
> 5C0 + 6C1 + 7C2 + 8C3 + 9C4 + 10C5 + 11C6

まだまだじゃの。
その程度ですか?


>>959
誰だコイツは? 引き取ってもらえ!

973 名前:132人目の素数さん :2005/12/11(日) 20:28:59
>>966
曲率がいたるところで1以下の曲線なら、いつでも面積一緒のような気がする。

974 名前:953 :2005/12/11(日) 20:39:02
>>972
お前誰だよ?

975 名前:953 :2005/12/11(日) 20:43:05
972のような偽者は置いておいて、これまでとは違うアプローチをしてみたい。

今まで書いてきた事と重複するかもしれないが・・・・・・・ 

976 名前:132人目の素数さん :2005/12/11(日) 21:16:23
953って偽者本物にかかわらず頭おかしいんじゃない?

数学板の不良債権として処理しないとなww

977 名前:132人目の素数さん :2005/12/11(日) 22:07:52
>>966
でけた。以下N(S)={x| |x-y|≦1 ∃y∈S}とする。
・補題 Cが長さlの折れ線のときN(C)の面積はπ+2l以下である。
(証明) 折れる回数に関する帰納法。線分のときはあきらか。Cがn回折れてる折れ線とする。
CをC=C’∪C’’、C’’は線分、C’は折れてる回数がn-1回の線分と分割する。
C’、C’’の長さをl’、l’’とする。帰納法の仮定よりN(C’)の面積はπ+2l’以下。
のこりの部分はC’とC’’の継ぎ目をPとするときにN(C’’)\N(C’)は
N(C’’)\N({P})にふくまれるが後者の面積はちょうど2l’’。よってN(C)の面積はπ+2l以下である。□
これと折れ線近似の理論を用いて長さlの曲線CについてN(C)の面積はπ+2l以下である。
l=1のときは2π+2、と思う。

978 名前:132人目の素数さん :2005/12/11(日) 22:40:03
いきなり2πってなったのなんで??

979 名前:977 :2005/12/12(月) 00:28:38
>>978
書き間違えっすorz。答えはおそらくπ+2。

980 名前:132人目の素数さん :2005/12/12(月) 06:51:16
>>966
Sの面積ってちゃんと定まるの?Sの境界の面積が0でなかったら、面積確定にならない。

981 名前:132人目の素数さん :2005/12/12(月) 07:51:21
>>976
いい問題だとは思うけどね。

982 名前:132人目の素数さん :2005/12/12(月) 11:02:36
新スレ

面白い問題おしえて〜な 11問目
http://science4.2ch.net/test/read.cgi/math/1134352879/

983 名前:132人目の素数さん :2005/12/12(月) 18:26:57
>>966は直線に微分可能とかなんか条件があったほうがいいのかな
一応連続なのは仮定して良いのかな?

「曲線」といった場合不連続なグラフのことをいう場合があるけど
(選択公理がない数学だと、R^2が可算個の曲線の和集合として(りゃ みたいな場合)

984 名前:132人目の素数さん :2005/12/13(火) 18:20:00
(1,2,2,3,3,4)=○|○○|○○|○||。


985 名前: ◆ZELriSYMOE :2005/12/14(水) 10:19:37
test

986 名前:132人目の素数さん :2005/12/14(水) 21:24:31



987 名前:132人目の素数さん :2005/12/14(水) 21:30:04



988 名前:132人目の素数さん :2005/12/14(水) 21:34:46



989 名前:132人目の素数さん :2005/12/15(木) 10:08:13



990 名前:132人目の素数さん :2005/12/15(木) 18:06:27


991 名前:132人目の素数さん :2005/12/15(木) 21:52:41
10!

992 名前:132人目の素数さん :2005/12/15(木) 21:55:57
9!

993 名前:132人目の素数さん :2005/12/15(木) 22:01:03
8!

994 名前:132人目の素数さん :2005/12/15(木) 22:18:16
7!

995 名前:132人目の素数さん :2005/12/15(木) 22:20:24
6!

996 名前:132人目の素数さん :2005/12/15(木) 22:20:25
435 名前:132人目の素数さん[] 投稿日:2005/12/15(木) 11:10:44
誤解してる奴がいるな。
このシリーズの目的を登山に例えると、まだ登山に取り掛かってない。
ザイルとかの道具を準備してるところ。道具なんだから自分で作る
必要はない。出来合いのものを買えばよい。ただ諸君の大勢は金を
そんなに持ってるわけではないだろうから、俺が面倒みてるわけ。
つまり、諸君の負担(予備知識)を軽くしようとしてるんだよ。

997 名前:132人目の素数さん :2005/12/15(木) 22:20:53
5 名前:132人目の素数さん[] 投稿日:2005/11/22(火) 16:36:48
ひとまず礼を言っておこう。有難う。
ただ、せっかく上げてもらって何だけど、このシリーズは類体論まで
いく予定なんで一桁じゃ済まないだろうから、次からはローマ数字
じゃなく普通の数字で「代数的整数論3」などの様にお願いします。

998 名前:132人目の素数さん :2005/12/15(木) 22:22:14
19 名前:132人目の素数さん[] 投稿日:2005/11/22(火) 18:14:05
5 :20B:2005/09/12(月) 17:21:31
このスレでは素人の発言は厳禁。したときは
容赦なくたたくからよく覚えておくように!
おれの怖さは、オイラースレのハンドル198で味わえ。

999 名前:132人目の素数さん :2005/12/15(木) 22:23:29
139 名前:132人目の素数さん[sage] 投稿日:2005/11/30(水) 21:46:58
> 荒らしは黙ってろ!
> ここは208様の神聖なるチラシの裏だ!
> お前ら下賎の者が寝言を書き込めるほど敷居は低くないぞ!
> 落ちこぼれダンボーラー予備軍がぁ!!

1000 名前:132人目の素数さん :2005/12/15(木) 22:23:36
1000!

1001 名前:1001 :Over 1000 Thread
このスレッドは1000を超えました。
もう書けないので、新しいスレッドを立ててくださいです。。。


全部 最新50
DAT2HTML 0.32lpp Converted.